131
Help | Profile | My Organizer | My Bookmarks | Logout Answers and Explanations Test Sections Section 1 Section 2 Section 3 Section 4 Section 6 Section 7 Section 8 Section 9 Section 10 Back to Score Report Essay Online - Practice Test #2 These sample essays were originally handwritten by students but are shown typed here for ease of reading. The essays are displayed exactly as students wrote them, without any corrections to spelling, punctuation, or syntax. One handwritten sample essay is provided to illustrate the need for legible and clear handwriting. Exemplars: Essay Prompt Think carefully about the issue presented in the following excerpt and the assignment below. For a variety of reasons, people often make choices that have negative results. Later, they regret these choices, finding out too late that bad choices can be costly. On the other hand, decisions that seem completely reasonable when they are made may also be the cause of later disappointment and suffering. What looks like a wonderful idea at one time can later seem like the worst decision that could have been made. Good choices, too, can be costly. Are bad choices and good choices equally likely to have negative consequences? Plan and write an essay in which you develop your point of view on this issue. Support your position with reasoning and examples taken from your reading, studies, experience, or observations. Back to Score Report Copyright © 2006 The College Board. All rights reserved. Privacy Policy Terms of Use Contact Us 页码1/1 The Official SAT Online Course 2006-11-12 file://E:\新建文件\b2.htm UnRegistered

SAT Online Course Test 2 Explanations

Embed Size (px)

DESCRIPTION

Sat online course 2 explanation

Citation preview

Page 1: SAT Online Course Test 2 Explanations

严禁用于商业用途!

Help | Profile | My Organizer | My Bookmarks | Logout

Answers and Explanations

Test Sections

Section 1

Section 2

Section 3

Section 4

Section 6

Section 7

Section 8

Section 9

Section 10

Back to Score Report  

Essay     Online - Practice Test #2

These sample essays were originally handwritten by students but are shown typed here for ease of reading. The essays are displayed exactly as students wrote them, without any corrections to spelling, punctuation, or syntax. One handwritten sample essay is provided to illustrate the need for legible and clear handwriting.

Exemplars:

Essay Prompt

Think carefully about the issue presented in the following excerpt and the assignment below.

For a variety of reasons, people often make choices that have negative results. Later, they regret these choices, finding out too late that bad choices can be costly. On the other hand, decisions that seem completely reasonable when they are made may also be the cause of later disappointment and suffering. What looks like a wonderful idea at one time can later seem like the worst decision that could have been made. Good choices, too, can be costly.

Are bad choices and good choices equally likely to have negative consequences? Plan and write an essay in which you develop your point of view on this issue. Support your position with reasoning and examples taken from your reading, studies, experience, or observations.

Back to Score Report  

Copyright © 2006 The College Board. All rights reserved. Privacy Policy Terms of Use Contact Us

 

页码,1/1The Official SAT Online Course

2006-11-12file://E:\新建文件夹\b2.htm

UnRe

gistered

Page 2: SAT Online Course Test 2 Explanations

严禁用于商业用途!

Help | Profile | My Organizer | My Bookmarks | Logout

Answers and Explanations

Test Sections

Section 1

Section 2

Section 3

Section 4

Section 6

Section 7

Section 8

Section 9

Section 10

Back to Score Report  

View Answers and Explanations     Online - Practice Test #2

1If what is the value of when and

ANSWERS AND EXPLANATIONS Explanation for Correct Answer A :  Choice (A) is correct. Substituting the given values for and into the

equation results in the equation which simplifies to

or

Explanation for Incorrect Answer B :  Choice (B) is not correct. If and the equation reads

or which is a false statement.

Explanation for Incorrect Answer C :  Choice (C) is not correct. If and the equation reads

or which is a false statement.

Explanation for Incorrect Answer D :  Choice (D) is not correct. If and the equation reads

or which is a false statement.

Explanation for Incorrect Answer E :  Choice (E) is not correct. If and the equation reads

or which is a false statement.

(A)

(B)

(C)

(D)

(E)

2 If then

ANSWERS AND EXPLANATIONS

(A)

(B)

(C)

(D)

(E)

页码,1/17The Official SAT Online Course

2006-11-12file://E:\新建文件夹\b3.htm

UnRe

gistered

Page 3: SAT Online Course Test 2 Explanations

严禁用于商业用途!

Explanation for Correct Answer C :  Choice (C) is correct. Since it follows that equals

Therefore,

Explanation for Incorrect Answer A :  Choice (A) is not correct. If then and

Explanation for Incorrect Answer B :  Choice (B) is not correct. If then and

Explanation for Incorrect Answer D :  Choice (D) is not correct. If then and

Explanation for Incorrect Answer E :  Choice (E) is not correct. If then and

3

On the number line above, the tick marks are equally spaced. What is the value of

ANSWERS AND EXPLANATIONS Explanation for Correct Answer E :  Choice (E) is correct. Since the tick marks divide the segment between and into four equal parts, the distance between any two adjacent tick

marks must be Therefore corresponds to corresponds to

and the value of is

Explanation for Incorrect Answer A : 

Choice (A) is not correct. is the value of The question asks for the

value of

Explanation for Incorrect Answer B :  Choice (B) is not correct. The tick marks represent quarter units on the

(A)

(B)

(C)

(D)

(E)

页码,2/17The Official SAT Online Course

2006-11-12file://E:\新建文件夹\b3.htm

UnRe

gistered

Page 4: SAT Online Course Test 2 Explanations

严禁用于商业用途!

number line, so must be a whole number multiple of

Explanation for Incorrect Answer C : 

Choice (C) is not correct. is the value of but the question asks for the

difference between and

Explanation for Incorrect Answer D :  Choice (D) is not correct. The tick marks represent quarter units on the

number line, so must be a whole number multiple of

Questions 4-5 refer to the following graph.

4 Which of the following regions in Africa had an elephant population in that was

approximately of its elephant population in

I. Central

II. Eastern

III. Southern

ANSWERS AND EXPLANATIONS Explanation for Correct Answer B :  Choice (B) is correct. By looking at the graph, you can see that in region II

(Eastern Africa) the elephant population in was approximately of

the elephant population in In each of the regions I and III (Central Africa and Southern Africa, respectively), the elephant population in

was greater than of the elephant population in Therefore, only in

region II was the elephant population in approximately of that in

(A) I only

(B) II only

(C) III only

(D) I and III only

(E) I, II, and III

页码,3/17The Official SAT Online Course

2006-11-12file://E:\新建文件夹\b3.htm

UnRe

gistered

Page 5: SAT Online Course Test 2 Explanations

严禁用于商业用途!

Explanation for Incorrect Answer A :  Choice (A) is not correct. In Central Africa, the elephant population in

was greater than of the elephant population in

Explanation for Incorrect Answer C :  Choice (C) is not correct. In Southern Africa, the elephant population in

was greater than of the elephant population in

Explanation for Incorrect Answer D :  Choice (D) is not correct. In each of the regions Central Africa and Southern

Africa, the elephant population in was greater than of the elephant

population in

Explanation for Incorrect Answer E :  Choice (E) is not correct. In Eastern Africa, the elephant population in

was approximately of the elephant population in but in each of the

regions Central Africa and Southern Africa, the elephant population in

was greater than of the elephant population in

5

From to the total elephant population in the four regions of Africa decreased by approximately what percent?

ANSWERS AND EXPLANATIONS Explanation for Correct Answer C :  Choice (C) is correct. In the total elephant population was approximately In

the total elephant population was approximately Since the population

fell by about half, the population decrease was approximately

Explanation for Incorrect Answer A :  Choice (A) is not correct. The total elephant population in was approximately If the total elephant population in the four regions had decreased by approximately from to then the population in would have been approximately

Explanation for Incorrect Answer B :  Choice (B) is not correct. The total elephant population in was approximately If the total elephant population in the four regions had decreased by approximately from to then the population in would have been approximately

Explanation for Incorrect Answer D :  Choice (D) is not correct. The total elephant population in was approximately If the total elephant population in the four regions had decreased by approximately from to then the population in would have been approximately

(A)

(B)

(C)

(D)

(E)

页码,4/17The Official SAT Online Course

2006-11-12file://E:\新建文件夹\b3.htm

UnRe

gistered

Page 6: SAT Online Course Test 2 Explanations

严禁用于商业用途!

Explanation for Incorrect Answer E :  Choice (E) is not correct. The total elephant population in was approximately If the total elephant population in the four regions had decreased by approximately from to then the population in would have been approximately

6 For all numbers the function is defined by Which of the following has a negative value?

ANSWERS AND EXPLANATIONS Explanation for Correct Answer C :  Choice (C) is correct. The value of the function at is

Explanation for Incorrect Answer A :  Choice (A) is not correct. The value of the function at is

This is not a negative value.

Explanation for Incorrect Answer B :  Choice (B) is not correct. The value of the function at is

This is not a negative value.

Explanation for Incorrect Answer D :  Choice (D) is not correct. The value of the function at is

This is not a negative value.

Explanation for Incorrect Answer E :  Choice (E) is not correct. The value of the function at is

This is not a negative value.

(A)

(B)

(C)

(D)

(E)

7

In the figure above, divides square into two rectangles, and bisects

If what is the area of

(A)

(B)

(C)

(D)

页码,5/17The Official SAT Online Course

2006-11-12file://E:\新建文件夹\b3.htm

UnRe

gistered

Page 7: SAT Online Course Test 2 Explanations

严禁用于商业用途!

ANSWERS AND EXPLANATIONS Explanation for Correct Answer B :  Choice (B) is correct. The area of a triangle is given by the formula

where is the length of the base and is the height. In this

case, the length of the base of is because is a

square of side Since divides into two rectangles, it follows

that Since bisects it follows that and the height

of is Therefore, the area of is

Explanation for Incorrect Answer A :  Choice (A) is not correct. The base of is one side of square

so its length is If the area of were its height would be

However, bisects so the height of must also be equal

to the side length of square which is

Explanation for Incorrect Answer C :  Choice (C) is not correct. The base of is one side of square

so its length is If the area of were its height would be

However, bisects so the height of must also be equal

to the side length of square which is

Explanation for Incorrect Answer D :  Choice (D) is not correct. The base of is one side of square

so its length is If the area of were its height would be

However, bisects so the height of must also be equal to

the side length of square which is

Explanation for Incorrect Answer E :  Choice (E) is not correct. The base of is one side of square

so its length is If the area of were its height would be

However, bisects so the height of must also be equal

to the side length of square which is

(E)

8While driving on a -mile trip, Mr. Smith averages miles per hour for the first

hours. In terms of where how many miles remain to be traveled?

ANSWERS AND EXPLANATIONS Explanation for Correct Answer B :  Choice (B) is correct. Since Mr. Smith drives at an average speed of miles per hour for the first hours, he will have traveled miles. Thus,

(A)

(B)

(C)

(D)

(E)

页码,6/17The Official SAT Online Course

2006-11-12file://E:\新建文件夹\b3.htm

UnRe

gistered

Page 8: SAT Online Course Test 2 Explanations

严禁用于商业用途!

there are miles remaining to be traveled.

Explanation for Incorrect Answer A :  Choice (A) is not correct. The number of miles driven, must be subtracted from the total length of the trip,

Explanation for Incorrect Answer C :  Choice (C) is not correct. The total length of the trip is miles, not

miles. Also, after hours, Mr. Smith has driven miles, not miles.

Explanation for Incorrect Answer D :  Choice (D) is not correct. After hours, Mr. Smith has driven miles, not

miles. Therefore, miles, not miles, remain to be

traveled.

Explanation for Incorrect Answer E :  Choice (E) is not correct. This expression represents the ratio

The question asks for number of miles

remaining, not the amount to be traveled relative to the distance traveled so far.

9

In the figure above, the average (arithmetic mean) of the numbers in each column is

If the three circled numbers are moved from the left to the right column, which of the following combinations of numbers can then be moved from the right to the left column so that remains the average of the numbers in each column?

ANSWERS AND EXPLANATIONS Explanation for Correct Answer E :  Choice (E) is correct. Each column has numbers, and the average of each column is so the sum of the numbers in the left column is equal to the sum of the numbers in the right column. When the circled numbers, whose sum is are moved from the left to the right column, a list of numbers with sum needs to be moved from the right column to the left. Of the given options, only the list has a sum of

Explanation for Incorrect Answer A :  Choice (A) is not correct. If these numbers were switched with the circled numbers, then the average of the numbers in the left column would be

but the average of the numbers in the right column would be

(A)

(B)

(C)

(D)

(E)

页码,7/17The Official SAT Online Course

2006-11-12file://E:\新建文件夹\b3.htm

UnRe

gistered

Page 9: SAT Online Course Test 2 Explanations

严禁用于商业用途!

Explanation for Incorrect Answer B :  Choice (B) is not correct. If these numbers were switched with the circled

numbers, then the average of the numbers in the left column would be

but the average of the numbers in the right column would be

Explanation for Incorrect Answer C :  Choice (C) is not correct. If these numbers were switched with the circled

numbers, then the average of the numbers in the left column would be

but the average of the numbers in the right column would be

Explanation for Incorrect Answer D :  Choice (D) is not correct. If these numbers were switched with the circled

numbers, then the average of the numbers in the left column would be

but the average of the numbers in the right column would be

10By P.M., of the junior class had arrived at a school dance. By P.M.,

more juniors had arrived, raising attendance to of the junior class. How many

people are in the junior class?

ANSWERS AND EXPLANATIONS Explanation for Correct Answer D : 

Choice (D) is correct. If the difference between of the junior class and

of the junior class is students, then students make up exactly

of the junior class. Therefore, the total number of students in the

junior class is

Explanation for Incorrect Answer A :  Choice (A) is not correct. If there were only students in the entire junior class, the whole class would have arrived between and which

contradicts the statement in the problem that of the class had arrived by

Explanation for Incorrect Answer B :  Choice (B) is not correct. If there were only people in the junior class,

would have shown up by and when more students had arrived

by the fraction of the class present would have been not

Explanation for Incorrect Answer C :  Choice (C) is not correct. If there were only people in the junior class,

(A)

(B)

(C)

(D)

(E)

页码,8/17The Official SAT Online Course

2006-11-12file://E:\新建文件夹\b3.htm

UnRe

gistered

Page 10: SAT Online Course Test 2 Explanations

严禁用于商业用途!

would have shown up by and when more students had arrived

by the fraction of the class present would have been not

Explanation for Incorrect Answer E :  Choice (E) is not correct. If there were people in the junior class, would have shown up by and when more students had arrived by

the fraction of the class present would have been not

11

In the equilateral triangle above, what is the value of

ANSWERS AND EXPLANATIONS Explanation for Correct Answer D :  Choice (D) is correct. Since triangle is equilateral, the measure of

is thus, and Since the measure of is the three angles of the small triangle on the left measure

and therefore, Solving for gives

Explanation for Incorrect Answer A :  Choice (A) is not correct. Since triangle is equilateral, and the angles in the small triangle on the left are and If

were then and the sum of the angles in the small triangle on the left would be greater than

Explanation for Incorrect Answer B :  Choice (B) is not correct. Since triangle is equilateral, and the angles in the small triangle on the left are and If

were then and the sum of the angles in the small triangle on the left would be greater than

Explanation for Incorrect Answer C :  Choice (C) is not correct. Since triangle is equilateral, and the angles in the small triangle on the left are and If

were then and the sum of the angles in the small triangle on the left would be greater than

Explanation for Incorrect Answer E :  Choice (E) is not correct. Since triangle is equilateral, and the angles in the small triangle on the left are and If

were then and the sum of the angles in the small triangle

(A)

(B)

(C)

(D)

(E)

页码,9/17The Official SAT Online Course

2006-11-12file://E:\新建文件夹\b3.htm

UnRe

gistered

Page 11: SAT Online Course Test 2 Explanations

严禁用于商业用途!

on the left would be less than

12

In the table above, if what is the value of

ANSWERS AND EXPLANATIONS Explanation for Correct Answer D :  Choice (D) is correct. The values in each row of the table must satisfy the equation Substitution of the values from the second row of the

table into the equation yields Substitution of the values from the third row of the table into the equation yields

Explanation for Incorrect Answer A :  Choice (A) is not correct. is the -coordinate of the point with -coordinate

Explanation for Incorrect Answer B :  Choice (B) is not correct. is the value of but the question asks for the value of

Explanation for Incorrect Answer C :  Choice (C) is not correct. but this does not give the value of

Explanation for Incorrect Answer E :  Choice (E) is not correct. but this does not give the value of

(A)

(B)

(C)

(D)

(E)

13

In the figure above, the radius of the circle with center is twice the radius of the circle with center What is the radius of the circle with center

页码,10/17The Official SAT Online Course

2006-11-12file://E:\新建文件夹\b3.htm

UnRe

gistered

Page 12: SAT Online Course Test 2 Explanations

严禁用于商业用途!

ANSWERS AND EXPLANATIONS Explanation for Correct Answer B :  Choice (B) is correct. The radius of the circle with center is equal to

The radius of the circle with center is equal to Since the radius of the circle with center is twice the radius of the circle with center

it follows that This simplifies to the equation

and further to Therefore, the radius of the circle with center is

Explanation for Incorrect Answer A :  Choice (A) is not correct. If the radius of the circle with center were then would equal and the ratio of the radii of the circles would be

Explanation for Incorrect Answer C :  Choice (C) is not correct. If the radius of the circle with center were then would equal and the ratio of the radii of the circles would be

Explanation for Incorrect Answer D :  Choice (D) is not correct. If the radius of the circle with center were then would equal and the ratio of the radii of the circles would be

Explanation for Incorrect Answer E :  Choice (E) is not correct. If the radius of the circle with center were then would equal and the ratio of the radii of the circles would be

(A)

(B)

(C)

(D)

(E)

14

A weather-watch camera is set so that its shutter opens every seconds. If

represents the number of times the camera’s shutter opens in hours, which of the

following defines

ANSWERS AND EXPLANATIONS Explanation for Correct Answer D :  Choice (D) is correct. Since there are seconds in an hour, the number

(A)

(B)

(C)

(D)

(E)

页码,11/17The Official SAT Online Course

2006-11-12file://E:\新建文件夹\b3.htm

UnRe

gistered

Page 13: SAT Online Course Test 2 Explanations

严禁用于商业用途!

of openings per hour is If this goes on for hours, the shutter will

open times. Hence,

Explanation for Incorrect Answer A :  Choice (A) is not correct. This equation corresponds to the situation in which the shutter opens times each hour.

Explanation for Incorrect Answer B :  Choice (B) is not correct. This equation corresponds to the situation in which the shutter opens times each hour, or times per second.

Explanation for Incorrect Answer C :  Choice (C) is not correct. This corresponds to a situation in which the camera shutter opens less than once an hour.

Explanation for Incorrect Answer E :  Choice (E) is not correct. According to this equation, the more time that passes, the fewer times the shutter opens. Since the shutter opens every seconds, this does not make sense.

15       Marble was red.       Marble was not red.       Marble was blue.       Marble was the same color as marble       Marble was the same color as marble A jar contained marbles—some red, some white, and some blue. The information above is about marbles that were drawn from the jar. If is the total number of blue marbles drawn, which of the following statements must be true?

ANSWERS AND EXPLANATIONS Explanation for Correct Answer E :  Choice (E) is correct. Either the five colors, listed in order, were red, white, blue, red, and white (if marbles and were both white) or red, blue, blue, red, and blue (if marbles and were both blue). In the first case, there was only one blue marble, in the second case there were three blue marbles so or

Explanation for Incorrect Answer A :  Choice (A) is not correct. is a possible value for but is also a possible value.

Explanation for Incorrect Answer B :  Choice (B) is not correct. Marbles and were red and was blue. If were two then the given information would be contradicted since marbles and were the same color (either both white or both blue).

Explanation for Incorrect Answer C :  Choice (C) is not correct. is a possible value for but is also a possible value.

Explanation for Incorrect Answer D :  Choice (D) is not correct. Marbles and were red and was blue, so

(A) The only possible value of is

(B) The only possible value of is

(C) The only possible value of is

(D) The only possible values of are and

(E) The only possible values of are and

页码,12/17The Official SAT Online Course

2006-11-12file://E:\新建文件夹\b3.htm

UnRe

gistered

Page 14: SAT Online Course Test 2 Explanations

严禁用于商业用途!

is a possible value. However, is not a possible value because the given information would be contradicted (marbles and were either both white or both blue).

16

In the figure above, If which of the following must be equal to

ANSWERS AND EXPLANATIONS Explanation for Correct Answer E :  Choice (E) is correct. The angles labeled and are supplementary, so

The angles labeled and are corresponding angles, so and Since substitution gives

from which it follows that and The angles labeled and are supplementary, as are the angles

labeled and so and Now that the measures of all the labeled angles have been found, substitution of the correct values in the five options shows that only

Explanation for Incorrect Answer A :  Choice (A) is not correct. but

Explanation for Incorrect Answer B :  Choice (B) is not correct. but

Explanation for Incorrect Answer C :  Choice (C) is not correct. but

Explanation for Incorrect Answer D :  Choice (D) is not correct. but

(A)

(B)

(C)

(D)

(E)

17

If the integer is divided by the remainder is What is the remainder if is divided by

(A)

(B)

(C)

(D)

(E)

页码,13/17The Official SAT Online Course

2006-11-12file://E:\新建文件夹\b3.htm

UnRe

gistered

Page 15: SAT Online Course Test 2 Explanations

严禁用于商业用途!

ANSWERS AND EXPLANATIONS Explanation for Correct Answer C :  Choice (C) is correct. This problem can be solved quickly by choosing a value that satisfies the “if” clause of the first sentence. For example, if is divided by then the remainder is , and when is divided by the remainder is Thus, is a counterexample to options (A), (B), (D), and (E), so option (C) is the only possibility for the correct answer.

More generally, that the remainder must be can be proven as follows. Since there is a remainder of when is divided by it follows that must equal where is some integer. Therefore, the expression

is equal to When is divided by the only part that contributes to the remainder is because goes evenly into

. Since the remainder when is divided by is it follows that the remainder when is divided by is also

Explanation for Incorrect Answer A :  Choice (A) is not correct. If is equal to then the remainder is when

is divided by However, and the remainder when is divided by is not

Explanation for Incorrect Answer B :  Choice (B) is not correct. If is equal to then the remainder is when

is divided by However, and the remainder when is divided by is not

Explanation for Incorrect Answer D :  Choice (D) is not correct. If is equal to then the remainder is when

is divided by However, and the remainder when is divided by is not

Explanation for Incorrect Answer E :  Choice (E) is not correct. If is equal to then the remainder is when

is divided by However, and the remainder when is divided by is not

18The daytime telephone rate between two cities is cents for the first minutes and cents for each additional minute. The total charge is reduced percent on calls made after P.M. The cost, in dollars, of a -minute call made at P.M. between these two cities is

ANSWERS AND EXPLANATIONS Explanation for Correct Answer B :  Choice (B) is correct. First, consider the cost of the phone call without the

reduction. The first minutes cost cents, or dollar.

There are minutes remaining, and each minute costs cents,

or dollars, for a subtotal of dollars. Therefore, the full

cost of the call is dollars.

Now, consider the reduction in price. Subtracting from the

original price yields

(A)

(B)

(C)

(D)

(E)

页码,14/17The Official SAT Online Course

2006-11-12file://E:\新建文件夹\b3.htm

UnRe

gistered

Page 16: SAT Online Course Test 2 Explanations

严禁用于商业用途!

Explanation for Incorrect Answer A :  Choice (A) is not correct. Here, only the cost of the first minutes of the call are reduced by Also, the term should be to account for the expression being in terms of dollars.

Explanation for Incorrect Answer C :  Choice (C) is not correct. Here, the term should be to account for the cost of the last minutes of the call in terms of dollars.

Explanation for Incorrect Answer D :  Choice (D) is not correct. Here, the total cost of the call is of the daytime rate, rather than being reduced by Also, the term should be to account for the expression being in terms of dollars.

Explanation for Incorrect Answer E :  Choice (E) is not correct. Here, the total cost of the call is of the daytime rate, rather than being reduced by Also, minutes are charged at cents per minute, which would make the entire call

minutes.

19 How many positive four-digit integers have as their first digit and or as their last digit?

ANSWERS AND EXPLANATIONS Explanation for Correct Answer C :  Choice (C) is correct. There are ways of choosing the hundreds digit and

ways of choosing the tens digit (any of the digits or may go into either of these positions) There are ways of filling the ones

digit (it can only be either a or a ), and only way to fill the thousands digit since it must be Therefore, the number of integers satisfying the stated conditions is

Explanation for Incorrect Answer A :  Choice (A) is not correct. The list contains numbers of this form but there are more:

Explanation for Incorrect Answer B :  Choice (B) is not correct. This is the number of positive three-digit numbers that have or as their last digit.

Explanation for Incorrect Answer D :  Choice (D) is not correct. This is the number of positive three-digit number that have or as their first digit.

Explanation for Incorrect Answer E :  Choice (E) is not correct. The complete list of numbers of this form is

and contains only numbers.

(A)

(B)

(C)

(D)

(E)

20

页码,15/17The Official SAT Online Course

2006-11-12file://E:\新建文件夹\b3.htm

UnRe

gistered

Page 17: SAT Online Course Test 2 Explanations

严禁用于商业用途!

In the figure above, a square with sides of length units is divided into squares. What is the area of the circle (not shown) that passes through the points

and which are the centers of the four corner squares?

ANSWERS AND EXPLANATIONS Explanation for Correct Answer B :  Choice (B) is correct. Since the sides of the large square have a length of units, each of the small squares has sides of length units. The points and are separated by a full side of a small square, and two half sides of a

small square, or a total of units. Similarly, and

are separated by units, and so on. Moreover, the points and are the vertices of a square that has sides of length units. The

diagonals of square are diameters of the circle that passes through

the points and Since the sides and and the diagonal

of square form a right isosceles triangle, the length of the

diagonal (and the diameter of the circle) is and therefore the radius of

the circle is Therefore, the area of the circle is

square units.

Explanation for Incorrect Answer A :  Choice (A) is not correct. The area of the circle would be square units if

the diameter of the circle, had length units, but the length of

is units.

Explanation for Incorrect Answer C :  Choice (C) is not correct. The area of the circle would be square units if

the diameter of the circle, had length units, but the length of is

units.

Explanation for Incorrect Answer D :  Choice (D) is not correct. The area of the circle would be square units if

the diameter of the circle, had length units, but the length of

is units.

Explanation for Incorrect Answer E :  Choice (E) is not correct. Since the circle is contained inside the large square, the area of the circle must be smaller than the area of the large square, but

square units is greater than square units.

(A) square units

(B) square units

(C) square units

(D) square units

(E) square units

页码,16/17The Official SAT Online Course

2006-11-12file://E:\新建文件夹\b3.htm

UnRe

gistered

Page 18: SAT Online Course Test 2 Explanations

严禁用于商业用途!

    

Back to Score Report  

Copyright © 2006 The College Board. All rights reserved. Privacy Policy Terms of Use Contact Us

 

页码,17/17The Official SAT Online Course

2006-11-12file://E:\新建文件夹\b3.htm

UnRe

gistered

Page 19: SAT Online Course Test 2 Explanations

严禁用于商业用途!

Help | Profile | My Organizer | My Bookmarks | Logout

Answers and Explanations

Test Sections

Section 1

Section 2

Section 3

Section 4

Section 6

Section 7

Section 8

Section 9

Section 10

Back to Score Report  

View Answers and Explanations     Online - Practice Test #2

1 Although visitors initially may find touring the city by subway to be -------, they are pleased to discover that subways are an inexpensive and ------- way to get around.

ANSWERS AND EXPLANATIONS Explanation for Correct Answer B :  Choice (B) is correct. "Daunting" means intimidating. "Efficient" means effective. If one were to insert these terms into the text, the sentence would read "Although visitors initially may find touring the city by subway to be daunting, they are pleased to discover that subways are an inexpensive and efficient way to get around." The first part of the sentence describes how the visitors feel "initially," or at first, about traveling by subway. The second part of the sentence describes a way in which the visitors' views change. It is likely that visitors to a city would at first find its subways intimidating. It is also likely that once visitors discover that subways are an inexpensive and effective way to travel, they would be pleased.

Explanation for Incorrect Answer A :  Choice (A) is incorrect. "Wasteful" means inclined to waste, or uneconomical. "Generous" means giving. If one were to insert these terms into the text, the sentence would read "Although visitors initially may find touring the city by subway to be wasteful, they are pleased to discover that subways are an inexpensive and generous way to get around." It is not likely that visitors to a city would find its subways to be uneconomical, since subways are stated to be inexpensive. In addition, the word "generous" doesn't describe a mode of transportation.

Explanation for Incorrect Answer C :  Choice (C) is incorrect. "Extravagant" means excessive. "Prohibitive" means tending to discourage. If one were to insert these terms into the text, the sentence would read "Although visitors initially may find touring the city by subway to be extravagant, they are pleased to discover that subways are an inexpensive and prohibitive way to get around." It is not likely that visitors to a city would find its subways to be excessive, since subways are said to be inexpensive. It is also not likely that visitors would be pleased by something tending to discourage them.

Explanation for Incorrect Answer D :  Choice (D) is incorrect. "Convenient" means easy or suitable. "Solitary" means alone. If one were to insert these terms into the text, the sentence would read "Although visitors initially may find touring the city by subway to be convenient, they are pleased to discover that subways are an inexpensive and solitary way to get around." It is likely that visitors to a city would find its subways to be "convenient," or an easy means of traveling. However, since the subway is a type of public transportation, travel by subway is, by definition, not "solitary."

Explanation for Incorrect Answer E :  Choice (E) is incorrect. "Enjoyable" means pleasing or satisfying. "Easy" means simple or not difficult. If one were to insert these terms into the text, the sentence would read "Although visitors initially may find touring the city by subway to be enjoyable, they are pleased to discover that subways are an inexpensive and easy way to get around." If visitors initially find touring by subway to be pleasing, then discovering such travel to be both inexpensive and simple doesn't reflect the

(A) wasteful. . generous

(B) daunting. . efficient

(C) extravagant. . prohibitive

(D) convenient. . solitary

(E) enjoyable. . easy

页码,1/22The Official SAT Online Course

2006-11-12file://E:\新建文件夹\b4.htm

UnRe

gistered

Page 20: SAT Online Course Test 2 Explanations

严禁用于商业用途!

contrast implied by the term "although."

2 One critic asserts that modern urban architecture causes sensory deprivation because it fails to provide visual and tactile -------.

ANSWERS AND EXPLANATIONS Explanation for Correct Answer B :  Choice (B) is correct. "Stimulation" means excitement or arousal. If one were to insert this term into the text, the sentence would read "One critic asserts that modern urban architecture causes sensory deprivation because it fails to provide visual and tactile stimulation." In an environment of sensory deprivation, the senses are deprived of "stimulation," or arousal. People in such an environment would have nothing to see (visual arousal) or feel (tactile arousal).

Explanation for Incorrect Answer A :  Choice (A) is incorrect. "Latency" is the state of being dormant or inactive. If one were to insert this term into the text, the sentence would read "One critic asserts that modern urban architecture causes sensory deprivation because it fails to provide visual and tactile latency." Architecture is accused of failing to provide something interesting to the senses, not failing to provide a state of inactivity.

Explanation for Incorrect Answer C :  Choice (C) is incorrect. "Complacence" means self-satisfaction. If one were to insert this term into the text, the sentence would read "One critic asserts that modern urban architecture causes sensory deprivation because it fails to provide visual and tactile complacence." Although people in an environment of sensory deprivation might feel a lack of self-satisfaction, the environment is not necessarily the cause. Architecture is accused of failing to provide something interesting to the senses, not failing to provide self-satisfaction.

Explanation for Incorrect Answer D :  Choice (D) is incorrect. "Confusion" means bewilderment or puzzlement. If one were to insert this term into the text, the sentence would read "One critic asserts that modern urban architecture causes sensory deprivation because it fails to provide visual and tactile confusion." Failing to puzzle has nothing to do with sensory deprivation caused by architecture.

Explanation for Incorrect Answer E :  Choice (E) is incorrect. "Extension" means something additional. If one were to insert this term into the text, the sentence would read "One critic asserts that modern urban architecture causes sensory deprivation because it fails to provide visual and tactile extension." Architecture may cause sensory deprivation, but that has nothing to do with its ability to provide something additional.

(A) latency

(B) stimulation

(C) complacence

(D) confusion

(E) extension

3 Because little rain falls in the district during summer, municipalities are necessarily ------- to ------- water from winter storms.

ANSWERS AND EXPLANATIONS

(A) ready. . squander

(B) reluctant. . retain

(C) free. . absorb

(D) careful. . store

(E) unwilling. . conserve

页码,2/22The Official SAT Online Course

2006-11-12file://E:\新建文件夹\b4.htm

UnRe

gistered

Page 21: SAT Online Course Test 2 Explanations

严禁用于商业用途!

Explanation for Correct Answer D :  Choice (D) is correct. "Careful" means showing concern. "Store" means to put aside for future use. If one were to insert these terms into the text, the sentence would read "Because little rain falls in the district during summer, municipalities are necessarily careful to store water from winter storms." In order to have enough water during the summer, when little rain falls, "municipalities," or cities and towns, must take advantage of water that is available during the winter. After collecting water during the winter, they must show concern to put it aside so it can be available during the dry summer.

Explanation for Incorrect Answer A :  Choice (A) is incorrect. "Ready" means prepared. "Squander" means to waste. If one were to insert these terms into the text, the sentence would read "Because little rain falls in the district during summer, municipalities are necessarily ready to squander water from winter storms." In order to have enough water during the summer, municipalities would have to be prepared to collect water that is available during the winter. Wasting it wouldn't ensure sufficient water during the summer.

Explanation for Incorrect Answer B :  Choice (B) is incorrect. "Reluctant" means hesitant or disinclined. "Retain" means to keep. If one were to insert these terms into the text, the sentence would read "Because little rain falls in the district during summer, municipalities are necessarily reluctant to retain water from winter storms." If the municipalities were hesitant to keep water from winter storms, it is unlikely that they would have water for use in the dry summer.

Explanation for Incorrect Answer C :  Choice (C) is incorrect. "Free" means not controlled. "Absorb" means to take in something. If one were to insert these terms into the text, the sentence would read "Because little rain falls in the district during summer, municipalities are necessarily free to absorb water from winter storms." Although cities and towns may be free to make choices, such as to collect water, it doesn't make sense to speak of cities or towns absorbing water. The word "absorb" describes a physical process of taking in something (for example, the way sponges take in water).

Explanation for Incorrect Answer E :  Choice (E) is incorrect. "Unwilling" means not willing or disinclined. "Conserve" means to save. If one were to insert these terms into the text, the sentence would read "Because little rain falls in the district during summer, municipalities are necessarily unwilling to conserve water from winter storms." It is likely that the lack of water would make municipalities more willing to save water in winter, rather than making them "unwilling" to do so.

4Toni Cade Bambara’s novels are engrossing because the protagonists, in striving to achieve goals, are not simply ------- characters.

ANSWERS AND EXPLANATIONS Explanation for Correct Answer A :  Choice (A) is correct. "Passive" means inactive. If one were to insert this term into the text, the sentence would read "Toni Cade Bambara's novels are engrossing because the protagonists, in striving to achieve goals, are not simply passive characters." This sentence makes sense because inactive characters would not strive to achieve goals and would thus be less "engrossing," or interesting to read about.

Explanation for Incorrect Answer B :  Choice (B) is incorrect. "Tangible" refers to something truly real, something capable

(A) passive

(B) tangible

(C) abandoned

(D) autonomous

(E) redundant

页码,3/22The Official SAT Online Course

2006-11-12file://E:\新建文件夹\b4.htm

UnRe

gistered

Page 22: SAT Online Course Test 2 Explanations

严禁用于商业用途!

of being physically handled or grasped. If one were to insert this term into the text, the sentence would read "Toni Cade Bambara's novels are engrossing because the protagonists, in striving to achieve goals, are not simply tangible characters." An author can create characters who seem real, but it is impossible to make a character in a book physically real.

Explanation for Incorrect Answer C :  Choice (C) is incorrect. "Abandoned" means left behind. If one were to insert this term into the text, the sentence would read "Toni Cade Bambara's novels are engrossing because the protagonists, in striving to achieve goals, are not simply abandoned characters." Characters who have been left behind might be "engrossing," or interesting to read about, but such characters would not necessarily be trying hard to achieve goals.

Explanation for Incorrect Answer D :  Choice (D) is incorrect. An "autonomous" character is self-directing or independent. If one were to insert this term into the text, the sentence would read "Toni Cade Bambara's novels are engrossing because the protagonists, in striving to achieve goals, are not simply autonomous characters." It is unclear why "striving to achieve goals" would show a character to be not independent.

Explanation for Incorrect Answer E :  Choice (E) is incorrect. "Redundant" means unnecessary. If one were to insert this term into the text, the sentence would read "Toni Cade Bambara's novels are engrossing because the protagonists, in striving to achieve goals, are not simply redundant characters." There is no logical connection between "redundant" characters and characters who try hard to achieve goals.

5 Once his integrity had been -------, the mayoral candidate was quick both to ------- these attacks and to issue counterattacks.

ANSWERS AND EXPLANATIONS Explanation for Correct Answer D :  Choice (D) is correct. "Impugned" means attacked by words or arguments. "Repudiate" means to reject as untrue or unjust. If one were to insert these terms into the text, the sentence would read "Once his integrity had been impugned, the mayoral candidate was quick both to repudiate these attacks and to issue counterattacks." The phrase "these attacks" indicates that the first missing term will have a meaning similar to "attacked." When one's "integrity," or honesty, is attacked, it is reasonable to reject the attacks by claiming them to be unjust or in error.

Explanation for Incorrect Answer A :  Choice (A) is incorrect. "Debunked" means to expose something as false. "Buttress" means to strengthen or support. If one were to insert these terms into the text, the sentence would read "Once his integrity had been debunked, the mayoral candidate was quick both to buttress these attacks and to issue counterattacks." People might indeed try to "debunk" a candidate's integrity, or expose his integrity as false. However, the candidate wouldn't want to strengthen the attacks on himself.

Explanation for Incorrect Answer B :  Choice (B) is incorrect. "Restored" means renewed or brought back to an original state. "Recommence" means to begin again. If one were to insert these terms into the text, the sentence would read "Once his integrity had been restored, the mayoral candidate was quick both to recommence these attacks and to issue counterattacks." If the candidate's integrity had just been "restored," then he probably would not want to begin the attacks all over again.

Explanation for Incorrect Answer C :  Choice (C) is incorrect. "Revoked" means taken back. "Relinquish" means to give something up. If one were to insert these terms into the text, the sentence would

(A) debunked . . buttress

(B) restored . . recommence

(C) revoked . . relinquish

(D) impugned . . repudiate

(E) vitiated . . avoid

页码,4/22The Official SAT Online Course

2006-11-12file://E:\新建文件夹\b4.htm

UnRe

gistered

Page 23: SAT Online Course Test 2 Explanations

严禁用于商业用途!

read "Once his integrity had been revoked, the mayoral candidate was quick both to relinquish these attacks and to issue counterattacks." "Integrity," or honesty, can't be taken back. Also, a candidate could not "relinquish," or give up, attacks made on him by another person.

Explanation for Incorrect Answer E :  Choice (E) is incorrect. "Vitiated" means corrupted. "Avoid" means to stay away from something. If one were to insert these terms into the text, the sentence would read "Once his integrity had been vitiated, the mayoral candidate was quick both to avoid these attacks and to issue counterattacks." The candidate might try to stay away from attacks. However, a candidate's "integrity," or honesty, could not be corrupted by attacks, as the sentence implies. Attacks might declare a person to be dishonest, but such attacks cannot make him dishonest.

Passage 1

Passage 2

      Before silent film star Charlie Chaplin (1899-1977)

 came along, tramps and hoboes had long been a part of

 the Anglo-American cartoon and comic strip tradition.

LineBut Chaplin was to raise the tramp figure to heights of

5poetic and mythic power. Chaplin’s famous Tramp is a

 human being down and out on his luck but full of passion

 for life and hope that things will get better. He is complex

 and many-sided, thereby touching most human beings at

 one or more points in our character and makeup. There is

10a good deal in his nature that most of us identify with in

 our secret selves, apart from what we are in the public

  world we inhabit.

      Chaplin was very forthcoming during a 1957 inter-

 view about how much the early comic strips “Weary

15Willie and Tired Tim” influenced his creation of his

 own Tramp character. “There’s been a lot said about

 how I evolved the little tramp character who made my

 name,” said Chaplin. “Deep, psychological stuff has

 been written about how I meant him to be a symbol

页码,5/22The Official SAT Online Course

2006-11-12file://E:\新建文件夹\b4.htm

UnRe

gistered

Page 24: SAT Online Course Test 2 Explanations

严禁用于商业用途!

20of all the class war, of the love-hate concept, the death-

 wish, and what-all. But if you want the simple Chaplin

 truth behind the Chaplin legend, I started the little tramp

 simply to make people laugh and because those other

 tramps, Weary Willie and Tired Tim, had always made

25 me laugh.”

6 Given Chaplin’s statement in lines 22-25 (“I . . . laugh”), he would most likely view Passage 1’s portrayal of the “famous Tramp” (line 5) as

ANSWERS AND EXPLANATIONS Explanation for Correct Answer A :  Choice (A) is correct. Passage 1 makes assertions regarding the Tramp's complex and various human qualities that lines 22-25 in Passage 2 contradict. Passage 2 highlights Chaplin's creative use of the figure of the Tramp to make people laugh. Laughter is never mentioned in Passage 1.

Explanation for Incorrect Answer B :  Choice (B) is incorrect. In Passage 2 Chaplin states that he wanted nothing more than to make people laugh. Chaplin mentions that others have attempted to see his work as offering a social commentary, but he makes it clear that this was not his intention.

Explanation for Incorrect Answer C :  Choice (C) is incorrect. Passage 1 states that "tramps and hoboes had long been a part of the Anglo-American cartoon and comic strip tradition" (lines 2-3), but it does not imply that Chaplin's tramp was "derived from a comic strip."

Explanation for Incorrect Answer D :  Choice (D) is incorrect. Neither passage asserts or denies that Chaplin played other roles in his films.

Explanation for Incorrect Answer E :  Choice (E) is incorrect. Passage 1 suggests that, on the contrary, most people were moved by Chaplin's Tramp.

(A) misleading readers about his creative intention

(B) disregarding his effort to render social commentary through humor

(C) implying that the Tramp was derived from a comic strip

(D) asserting that the Tramp was the only character he portrayed

(E) assuming that few could embrace his ideas

7 Compared to the description of Chaplin’s Tramp in Passage 1, the account of the Tramp in Passage 2 is less

ANSWERS AND EXPLANATIONS

(A) optimistic

(B) ambiguous

(C) sincere

(D) complicated

(E) humorous

页码,6/22The Official SAT Online Course

2006-11-12file://E:\新建文件夹\b4.htm

UnRe

gistered

Page 25: SAT Online Course Test 2 Explanations

严禁用于商业用途!

Explanation for Correct Answer D :  Choice (D) is correct. In Passage 2 Chaplin says the Tramp was solely intended to make people laugh, whereas Passage 1 assigns a variety of characteristics to the Tramp: "he is complex and many-sided" (lines 7-8). Consequently, the account of the Tramp in Passage 2 is less complicated.

Explanation for Incorrect Answer A :  Choice (A) is incorrect. Neither passage is optimistic or pessimistic.

Explanation for Incorrect Answer B :  Choice (B) is incorrect. Both passages make their points clearly, and therefore neither can be labeled ambiguous.

Explanation for Incorrect Answer C :  Choice (C) is incorrect. Both passages appear sincere and honest in their opinions.

Explanation for Incorrect Answer E :  Choice (E) is incorrect. Neither passage relies on humor to make its points.

8 In comparison to Passage 2, the tone of Passage 1 is

ANSWERS AND EXPLANATIONS Explanation for Correct Answer B :  Choice (B) is correct. In Passage 1 the writing is quite "laudatory," or full of praise. According to Passage 1, "Chaplin was to raise the tramp figure to heights of poetic and mythic power" (lines 4-5). Passage 2, on the other hand, consists largely of quotes from Chaplin that show the actor's humility.

Explanation for Incorrect Answer A :  Choice (A) is incorrect. Neither passage contains anything that could be described as defensive; both discuss the Tramp in very positive, open manner.

Explanation for Incorrect Answer C :  Choice (C) is incorrect. Although both passages deal with emotions, neither is "sentimental," or overly emotional.

Explanation for Incorrect Answer D :  Choice (D) is incorrect. Passage 1 attempts to analyze the "poetic and mythic power" (line 5) of the Tramp figure, whereas Passage 2 reveals that he was created "simply to make people laugh" (line 23). Passage 1 should not be described as "less analytical" than Passage 2.

Explanation for Incorrect Answer E :  Choice (E) is incorrect. If anything, Passage 1 is "more pretentious" than Passage 2. Lines such as "there is a good deal in his nature that most of us identify with in our secret selves" (lines 9-11) could certainly be described as "pretentious," or exaggerated.

(A) more defensive

(B) more laudatory

(C) more sentimental

(D) less analytical

(E) less pretentious

9 Which best describes the relationship between Passage 1 and Passage 2?

(A) Passage 1 explains the profound effect of Chaplin’s Tramp on audiences; Passage 2 describes how Chaplin created the Tramp.

页码,7/22The Official SAT Online Course

2006-11-12file://E:\新建文件夹\b4.htm

UnRe

gistered

Page 26: SAT Online Course Test 2 Explanations

严禁用于商业用途!

ANSWERS AND EXPLANATIONS Explanation for Correct Answer E :  Choice (E) is correct. Lines 4-12 of Passage 1 tell how Chaplin enlarged the comic-strip figure of the Tramp and raised the Tramp "to heights of poetic and mythic power." In Passage 2 Chaplin reveals his thinking about the character and what his purpose was in creating the character: "simply to make people laugh" (line 23).

Explanation for Incorrect Answer A :  Choice (A) is incorrect. Passage 2 describes Chaplin's purpose in creating the Tramp. It does not discuss how Chaplin created the Tramp, which would involve information about the first film in which the Tramp appeared. Passage 1 discusses the appeal of the Tramp, but it focuses more on the Tramp's qualities than on the character's effect on audiences.

Explanation for Incorrect Answer B :  Choice (B) is incorrect. Passage 1 does indeed show how Chaplin enlarged the Tramp's character, but there is no discussion of audiences in Passage 2.

Explanation for Incorrect Answer C :  Choice (C) is incorrect. Passage 1 only briefly examines the Tramp's history before Chaplin, and Passage 2 does not analyze how the Tramp changed over time.

Explanation for Incorrect Answer D :  Choice (D) is incorrect. Although Passage 2 does discuss some of Chaplin's favorite comic figures, Passage 1 merely states that Chaplin was a "silent film star" (line 1). Passage 1 does not show how Chaplin gained fame.

(B) Passage 1 explores how Chaplin expanded the Tramp’s character; Passage 2 analyzes the Tramp’s impact on audiences.

(C) Passage 1 examines the origin of the Tramp figure; Passage 2 traces the comedic evolution of the Tramp.

(D) Passage 1 illustrates how Chaplin gained fame as the Tramp; Passage 2 discusses Chaplin’s love of comic figures like the Tramp.

(E) Passage 1 argues that Chaplin added depth to the Tramp; Passage 2 focuses on Chaplin’s purpose in developing the Tramp.

The following passage is from a 1994 novel about a young woman named Sophie who at age eleven had left Haiti to join her mother in New York.

      I was eighteen and going to start college in the fall. We

 moved to a one-family house in a tree-lined neighborhood;

 my mother continued working her two jobs, but she put in

Line even longer hours.

5     Before the move, I had been going to a Haitian Adventist

 school. They guaranteed that they would get me into college

 and they had lived up to their pledge. My mother couldn’t

  have been happier. Her sacrifices had paid off.

      I never told my mother that I hated the Bilingual

10Institution. It was as if I had never left Haiti. All the lessons were in French, except for English

页码,8/22The Official SAT Online Course

2006-11-12file://E:\新建文件夹\b4.htm

UnRe

gistered

Page 27: SAT Online Course Test 2 Explanations

严禁用于商业用途!

  composition   and literature classes.

      When my mother was home, she made me read out

 loud from the English Composition textbooks. The first

15words I read sounded like rocks falling in a stream. Then

 very slowly things began to take on meaning. There were

 words that I heard often. Words that jump out of New York

  Creole 1 conversations, like the last kernel in a cooling pop-

 corn machine. Words like TV, building, and feeling. There

20were other words that helped too, words that looked almost

 the same in French, but were pronounced differently in

 English: nationality, alien, race, enemy. Eventually, I

 began to read better. I answered swiftly when my mother

  asked me a question in English.

25     “Sophie, there is a great responsibility that comes with

 knowledge, to study hard,” my mother would say. I spent

 six years doing nothing but that. School, home, and prayer.

      And then, I fell in love with Joseph. He broke the monot-

 ony of my life when he moved next door. He was the color

30of ground coffee, with a voice like molasses that turned to

  music when he held a saxophone to his lips.

      One day, he came to our door and asked if he could use

 the phone. After his call, he announced that he had gotten a

  job. “I am a musician.”

35     “I know,” I said. “Sometimes I hear you playing at night.”

       “Does it bother you?”       “Non, it’s very pretty.”       “I detect an accent,” he said.

      Oh please, say a small one, I thought. After seven years

40 in this country, I was tired of having people

页码,9/22The Official SAT Online Course

2006-11-12file://E:\新建文件夹\b4.htm

UnRe

gistered

Page 28: SAT Online Course Test 2 Explanations

严禁用于商业用途!

detect my

 accent. I wanted to sound completely American, especially

  for him.       “Where are you from?” he asked.       “Haiti.”45      “Ah, do you speak Creole?”        “Oui, oui,” 2 I ventured, for a laugh.

      “We, we,” he said, pointing to me and him, “We have

 something in common. I speak a form of Creole, too. I am

 from Louisiana. My parents considered themselves what

50 we call Creoles. Is it a small world or what?”

      Later that week, Joseph brought me a sandwich to thank

 me for letting him use the phone. He stayed while I ate.

      “What are you going to study in college?” he asked.

       “I think I am going to be a doctor.”55      “You think? Is this something you like?”       “I suppose so,” I said.

      “You have to have a passion for what you do.”

      “My mother says it’s important for us to have a doctor

  in the family.”60      “What if you don’t want to be a doctor?”

      “There’s a difference between what people want and

  what’s good for them.”

      “You sound like you are quoting someone,” he said.

       “My mother.”65      “What would Sophie like to do?” he asked.

      That was the problem. Sophie really wasn’t sure. I had

  never really dared to dream on my own.

      “It is okay not to have your future on a map,” he said.

 “That way you can flow wherever life takes you.”

70     “That is not Haitian,” I said. “That’s very American.”

       “What is?”       “Being a wanderer. The very idea.”

      “I am not American,” he said. “I am African American.”

页码,10/22The Official SAT Online Course

2006-11-12file://E:\新建文件夹\b4.htm

UnRe

gistered

Page 29: SAT Online Course Test 2 Explanations

严禁用于商业用途!

       “What is the difference?”

75     “The African. Most Haitians are of African descent. So

 you see, it means that you and I, we are already part of each

  other.”

 

1 The French-derived language of Haiti 2 French word for “yes,” pronounced “we”

10 Sophie “hated” (line 9) her school because

ANSWERS AND EXPLANATIONS Explanation for Correct Answer B :  Choice (B) is correct. Although Sophie now lives in an English-speaking country, she receives most of her instruction in French, the language of Haiti. It is this lack of exposure to English that leads Sophie to hate her school.

Explanation for Incorrect Answer A :  Choice (A) is incorrect. Although Sophie is aware of her mother’s two jobs and long hours, she never expresses resentment about her mother’s working to send her to school.

Explanation for Incorrect Answer C :  Choice (C) is incorrect. No mention is made of the neighborhood where the school is located.

Explanation for Incorrect Answer D :  Choice (D) is incorrect. Although Sophie says that “things…[in English class] take on meaning” slowly, she does not indicate that the courses are difficult. On the contrary, she suggests that she would like them to be in English and thus more difficult for her.

Explanation for Incorrect Answer E :  Choice (E) is incorrect. Although we infer from her comments about pronunciation that Sophie makes mistakes, no mention is made of teachers or their expectations.

(A) she resented how hard her mother had to work to send her there

(B) she had little exposure to English

(C) it was in a neighborhood that seemed foreign and unfriendly

(D) the courses were too difficult

(E) the teachers were intolerant of her language errors

11 The comparison in line 15 emphasizes the

ANSWERS AND EXPLANATIONS Explanation for Correct Answer A : 

(A) halting way in which Sophie thought she read

(B) powerful impact of the words Sophie read aloud

(C) feeling Sophie had about her native language

(D) obstacles to Sophie’s writing in a new language

(E) strength of Sophie’s commitment to learn English

页码,11/22The Official SAT Online Course

2006-11-12file://E:\新建文件夹\b4.htm

UnRe

gistered

Page 30: SAT Online Course Test 2 Explanations

严禁用于商业用途!

Choice (A) is correct. Sophie’s notion that her first words sound like "rocks falling in a stream" shows that she hears the words as discrete sounds, unconnected to one another. Consequently, the comparison is used to describe the halting or choppy pace of Sophie's speech.

Explanation for Incorrect Answer B :  Choice (B) is incorrect. Sophie does not feel that her words have impact. It is not until later in the passage that Sophie's words "take on meaning."

Explanation for Incorrect Answer C :  Choice (C) is incorrect. Here Sophie is speaking not French but English, a language still unfamiliar to her.

Explanation for Incorrect Answer D :  Choice (D) is incorrect. Sophie is speaking, not writing, the words that sound like "rocks falling in a stream."

Explanation for Incorrect Answer E :  Choice (E) is incorrect. The comparison to "rocks falling in a stream" suggests Sophie’s hesitant and difficult beginning, not the strength of her commitment.

12 Sophie’s manner of answering her mother (lines 23-24) shows her

ANSWERS AND EXPLANATIONS Explanation for Correct Answer C :  Choice (C) is correct. Because of Sophie’s command of English or progress in English, she is able to swiftly respond to her mother. Sophie also indicates her growing command of English when she says “Eventually, I began to read better.”

Explanation for Incorrect Answer A :  Choice (A) is incorrect. The speed of Sophie’s answers is a sign of growing proficiency, not anger.

Explanation for Incorrect Answer B :  Choice (B) is incorrect. Although Sophie’s mother does “ask [her]…question[s] in English,” there is no indication that her mother is persistent or that Sophie is impatient.

Explanation for Incorrect Answer D :  Choice (D) is incorrect. All of Sophie’s observations in lines 13-24 have to do with speaking English, not French.

Explanation for Incorrect Answer E :  Choice (E) is incorrect. There is no indication that Sophie wants to return to her reading; she is simply answering her mother's questions in English.

(A) struggle with an anger that she has tried to conceal

(B) impatience with her mother’s persistent questioning

(C) growing command of a new language

(D) need to revert to French to express her feelings

(E) eagerness to return to her reading as quickly as possible

13 The use of italics in line 55 serves to emphasize

(A) Joseph’s idealism as contrasted with Sophie’s cynicism

(B) Joseph’s unsuccessful attempt to cheer Sophie

(C) Joseph’s incredulity at Sophie’s approach to her future

the unlikelihood that Sophie will be able to pursue her goal

页码,12/22The Official SAT Online Course

2006-11-12file://E:\新建文件夹\b4.htm

UnRe

gistered

Page 31: SAT Online Course Test 2 Explanations

严禁用于商业用途!

ANSWERS AND EXPLANATIONS Explanation for Correct Answer C :  Choice (C) is correct. By repeating the word “think” with emphasis, Joseph shows his incredulity, or disbelief, at what she has just told him about her thoughts of becoming a doctor. Consequently, the italics indicate that Joseph cannot believe how little Sophie has questioned her plans for the future.

Explanation for Incorrect Answer A :  Choice (A) is incorrect. Joseph expresses no idealism as he repeats the word “think,” nor does Sophie give any evidence of cynicism, or scorn.

Explanation for Incorrect Answer B :  Choice (B) is incorrect. Joseph’s repetition of Sophie’s word choice is not an attempt to cheer her, nor is she in a situation to need cheering. Rather, Joseph is highlighting the tentative quality of her prior claim.

Explanation for Incorrect Answer D :  Choice (D) is incorrect. Joseph does not challenge Sophie’s ability to attain her goal; rather, he underlines the tentative quality of what she has just said.

Explanation for Incorrect Answer E :  Choice (E) is incorrect. Joseph’s emphasis on the word “think” calls attention to Sophie’s tentative statement, not any underestimation of her own talent.

(D)

(E) the extent to which Sophie has underestimated her talents

14

Sophie’s response in line 56 reveals that she

ANSWERS AND EXPLANATIONS Explanation for Correct Answer E :  Choice (E) is correct. In using the tentative phrase “I suppose so,” Sophie indicates that she has not questioned her mother, who “says it’s important…to have a doctor in the family.” Her hesitance suggests that her mother decided she should go to medical school and Sophie never really thought she could question that decision.

Explanation for Incorrect Answer A :  Choice (A) is incorrect. Sophie’s use of the word “suppose” shows a lack of certainty, not a wish to impress.

Explanation for Incorrect Answer B :  Choice (B) is incorrect. Nowhere in the passage does the author provide evidence that Sophie experiences deep fears.

Explanation for Incorrect Answer C :  Choice (C) is incorrect. The hesitant nature of Sophie’s word choice makes clear that she is not strongly dedicated to the idea of becoming a doctor.

Explanation for Incorrect Answer D :  Choice (D) is incorrect. Although Sophie wishes to speak without an accent, she has already told Joseph that she comes from Haiti.

(A) is anxious to impress others

(B) is reluctant to confess her deepest fears

(C) is single-minded in her dedication to a medical career

(D) has apparently decided that she should hide her heritage from Joseph

(E) has rarely questioned the decisions others have made for her

页码,13/22The Official SAT Online Course

2006-11-12file://E:\新建文件夹\b4.htm

UnRe

gistered

Page 32: SAT Online Course Test 2 Explanations

严禁用于商业用途!

15 In line 61, the “difference” is between

ANSWERS AND EXPLANATIONS Explanation for Correct Answer B :  Choice (B) is correct. Sophie is speaking of the contrast between what she might want for herself and what she has been led to believe is good for her. If Sophie follows what she wants, she follows her desire. If she follows “what’s good for [her],” she is being practical.

Explanation for Incorrect Answer A :  Choice (A) is incorrect. Although “what people want” can be seen as selfishness, “what’s good for them” cannot necessarily be seen as altruism, or devotion to the welfare of others.

Explanation for Incorrect Answer C :  Choice (C) is incorrect. Although “what people want” can be seen as intuitive knowledge, “what’s good for [people]” cannot be identified as learned knowledge in this context. This context is not referring to a difference between types of knowledge but a difference between motivations—the difference between motivation according to personal desire and motivation according to practicality.

Explanation for Incorrect Answer D :  Choice (D) is incorrect. For Sophie “what’s good for [her]” may involve a sense of duty to her mother, but nothing in the passage suggests any conflict between this and a love for friends.

Explanation for Incorrect Answer E :  Choice (E) is incorrect. There is no indication that either “what people want” or “what’s good for them” is rooted in either duty to the past or future.

(A) selfishness and altruism

(B) desire and practicality

(C) intuitive knowledge and learned knowledge

(D) love for family and love for friends

(E) duty to the past and fear of the future

16 By using Sophie’s name (line 65) instead of “you,” Joseph is attempting to

ANSWERS AND EXPLANATIONS Explanation for Correct Answer C :  Choice (C) is correct. By referring to her in the third person as “Sophie” rather than in the second person as “you,” Joseph asks her to consider herself from a new point of view, as someone autonomous whose wishes are separate from those of her mother.

Explanation for Incorrect Answer A :  Choice (A) is incorrect. Joseph is asking questions, not posing as any sort of narrator.

Explanation for Incorrect Answer B : 

(A) pose as a narrator of a story

(B) approach a frightening topic gradually

(C) make Sophie consider a new perspective

(D) appear unconcerned about Sophie’s attitude

(E) pretend that he is unaware of Sophie’s presence

页码,14/22The Official SAT Online Course

2006-11-12file://E:\新建文件夹\b4.htm

UnRe

gistered

Page 33: SAT Online Course Test 2 Explanations

严禁用于商业用途!

Choice (B) is incorrect. The narrative provides no evidence that the topic of choice is frightening to Sophie or that using her name is meant to avoid frightening her.

Explanation for Incorrect Answer D :  Choice (D) is incorrect. Joseph shows that he is indeed concerned about Sophie’s submissive approach by persisting in his questions and by asking her to consider herself an independent individual, capable of choice.

Explanation for Incorrect Answer E :  Choice (E) is incorrect. Since he directs the question at Sophie, Joseph clearly is not pretending he does not know she is there.

17 The “problem” (line 66) for Sophie is that

ANSWERS AND EXPLANATIONS Explanation for Correct Answer E :  Choice (E) is correct. Sophie says, "I had never really dared to dream on my own" instead, she had taken cues from her mother. Sophie is clearly stating that the "problem" is that she is not sure what she wants because she has never thought about what she wants for her future.

Explanation for Incorrect Answer A :  Choice (A) is incorrect. Although what her mother wants is clear, Sophie is not sure what she wants, nor has she ever "dared to dream on [her] own."

Explanation for Incorrect Answer B :  Choice (B) is incorrect. The mother's financial sacrifices are mentioned earlier in the passage but not in this conversation. In addition, it is the mother who wants Sophie to become a doctor.

Explanation for Incorrect Answer C :  Choice (C) is incorrect. Joseph has already been identified as a musician. He is not trying to get Sophie to join him, but to make her own decisions.

Explanation for Incorrect Answer D :  Choice (D) is incorrect. Sophie does not address the length of time it would take to complete medical training.

(A) what she wants and what her mother wants are radically different

(B) medical school would require her mother to make even more financial sacrifices

(C) Joseph expects her to follow his dreams instead of her own

(D) she is uncomfortable with the long years of schooling that becoming a doctor entails

(E) she never considered her own needs as important

18 Joseph’s statement in lines 68-69 (“It is . . . you”) primarily shows him to be

ANSWERS AND EXPLANATIONS Explanation for Correct Answer B :  Choice (B) is correct. By saying it's "okay not to have your future on a map," Joseph is urging Sophie to see her future in a less restricted way than she has previously seen it. Joseph’s suggestions that Sophie’s future does not need to be mapped and that she can “flow wherever life takes [her]” indicate his ability

(A) more tenacious than Sophie’s mother is

(B) more tolerant of ambiguity than Sophie is

(C) more cynical about the future than Sophie is

(D) unsentimental about family and heritage

(E) incapable of making commitments

页码,15/22The Official SAT Online Course

2006-11-12file://E:\新建文件夹\b4.htm

UnRe

gistered

Page 34: SAT Online Course Test 2 Explanations

严禁用于商业用途!

to handle uncertainty, or ambiguity.

Explanation for Incorrect Answer A :  Choice (A) is incorrect. By indicating that one’s future need not be mapped out, Joseph shows himself to be less tenacious, or single-minded, than Sophie’s mother seems to be.

Explanation for Incorrect Answer C :  Choice (C) is incorrect. Nothing in Joseph’s comments about the flexibility of the future suggests cynicism.

Explanation for Incorrect Answer D :  Choice (D) is incorrect. Joseph addresses only the ways Sophie might manage her future. He does not discuss or allude to family or heritage.

Explanation for Incorrect Answer E :  Choice (E) is incorrect. Joseph does not suggest that he dislikes commitments. He simply indicates that the future need not be rigidly planned.

The following passage is from an essay written in 1991 about fences in suburban culture.

      In the United States, the traditional view embraced

 by society is that fences are European, out of place in the

 American landscape. This notion turns up repeatedly in

Linenineteenth-century American writing about the landscape.

5One author after another denounces “the Englishman’s

 insultingly inhospitable brick wall, topped with broken

 bottles.” Frank J. Scott, an early landscape architect who

 had a large impact on the look of America’s first suburbs,

 worked tirelessly to rid the landscape of fences, which he

10derided as a feudal holdover from Britain. Writing in 1870,

 he held that “to narrow our own or our neighbor’s views of

 the free graces of Nature” was selfish and undemocratic. To

 drive through virtually any American suburb today, where

 every lawn steps right up to the street in a gesture of open-

15ness and welcome, is to see how completely such views

 have triumphed. After a visit to the United States, British

页码,16/22The Official SAT Online Course

2006-11-12file://E:\新建文件夹\b4.htm

UnRe

gistered

Page 35: SAT Online Course Test 2 Explanations

严禁用于商业用途!

 novelist Vita Sackville-West decided that “Americans . . .

  have no sense of private enclosure.”

      In many American suburbs such as the one where I grew

20up, a fence or a hedge along the street meant one thing: the

 family who lived behind it was antisocial, perhaps even had

 something to hide. Fences and hedges said: Ogres within;

 skip this place on Halloween. Except for these few dubious

 addresses, each little plot in our development was landscaped

25like a miniature estate, the puniest “expanse” of unhedged

 lawn was made to look like a public park. Any enjoyment

 of this space was sacrificed to the conceit of wide-open land,

 for without a fence or hedge, front yards were much too

 public to spend time in. Families crammed their activities

30into microscopic back-yards, the one place where the use-

 fulness of fences and hedges seemed to outweigh their

  undemocratic connotations.

      But the American prejudice against fences predates the

 suburban development. Fences have always seemed to us

35somehow un-American. Europeans built walled gardens;

 Americans from the start distrusted the hortus

conclusus * .

 If the space within the wall was a garden, then what was

 that outside the wall? To the Puritans the whole American

 landscape was a promised land and to draw lines around

40sections of it was to throw this paramount idea into question.

 When Anne Bradstreet, the Massachusetts colony’s first

 poet, set about writing a traditional English garden ode, she

  tore down the conventional garden wall—or (it

页码,17/22The Official SAT Online Course

2006-11-12file://E:\新建文件夹\b4.htm

UnRe

gistered

Page 36: SAT Online Course Test 2 Explanations

严禁用于商业用途!

comes to

 the same thing) made it capacious enough to take in the

45 whole of America.

      The nineteenth-century transcendentalists, too, considered

 the American landscape “God’s second book” and they

 taught us to read it for moral instruction. Residues of this

 idea persist, of course; we still regard and write about nature

50with high moral purpose (an approach that still produces a

 great deal of pious prose). And though, in our own nature

 writing, guilt seems to have taken the rhetorical place of

 nineteenth-century ecstasy, the essential religiosity remains.

 We may no longer spell it out, but most of us still believe

55the landscape is somehow sacred, and to meddle with it

 sacrilegious. And to set up hierarchies within it—to set off

 a garden from the surrounding countryside—well, that

  makes no sense at all.

 

* A Latin phrase that means “confined garden”

19 In line 1, “embraced” most nearly means

ANSWERS AND EXPLANATIONS Explanation for Correct Answer B :  Choice (B) is correct. "Adopted" means taken. In line 1, the author states, "the traditional view embraced by society is that fences are European." In this context, "embraced" signifies "taken" or "adopted."

Explanation for Incorrect Answer A :  Choice (A) is incorrect. "Caressed" means hugged. It is not possible to hug a point of view.

Explanation for Incorrect Answer C :  Choice (C) is incorrect. "Enfolded" means enclosed. It is not possible to enclose

(A) caressed

(B) adopted

(C) enfolded

(D) included

(E) encircled

页码,18/22The Official SAT Online Course

2006-11-12file://E:\新建文件夹\b4.htm

UnRe

gistered

Page 37: SAT Online Course Test 2 Explanations

严禁用于商业用途!

something non-material like a point of view.

Explanation for Incorrect Answer D :  Choice (D) is incorrect. "Included" means involved. In line 1, the author states, "the traditional view embraced by society is that fences are European." "Included" does not mean the same thing as "embraced" in this context.

Explanation for Incorrect Answer E :  Choice (E) is incorrect. "Encircled" means surrounded. “Encircled” does not refer to taking in or assuming an idea as one’s own.

20 In lines 10-12, Frank J. Scott’s observation implies that nature

ANSWERS AND EXPLANATIONS Explanation for Correct Answer B :  Choice (B) is correct. In these lines, Scott describes erecting fences as "selfish and undemocratic." In addition, he describes the presence of fences as something that would "narrow our own or our neighbor's views of the free graces of Nature." Clearly, Scott believes that nature "should be available for all to enjoy without hindrance."

Explanation for Incorrect Answer A :  Choice (A) is incorrect. Scott writes only about inhabited areas, where people may choose to erect fences. In addition, his observations imply that nature can be graceful and abundant in areas that are inhabited.

Explanation for Incorrect Answer C :  Choice (C) is incorrect. Although the passage does state that Scott had an impact on the design of early suburbs, lines 10-12 address only his dislike of the way fences obstruct one's view.

Explanation for Incorrect Answer D :  Choice (D) is incorrect. Although Scott does compare the use of fences in Britain and America, he does not compare the effects of nature on the inhabitants of the two countries.

Explanation for Incorrect Answer E :  Choice (E) is incorrect. In lines 10-12, Scott does not compare the presence of nature in American suburbs and the British countryside.

(A) is graceful and beautiful only in areas uninhabited by humans

(B) should be available for all to enjoy without hindrance

(C) must be incorporated into the design of American suburbs

(D) exerts a more powerful effect on the British than on Americans

(E) is less evident in American suburbs than in the British countryside

21 In lines 12-15, “To drive . . . welcome” suggests that suburban lawns

ANSWERS AND EXPLANATIONS Explanation for Correct Answer C :  Choice (C) is correct. The author suggests that the "openness and welcome" of the lawns reflect the attitude of the people who own them. This attitude is described as characteristically American.

(A) represent the American preoccupation with appearances

(B) epitomize the values of capitalism

(C) reflect a particular American attitude

(D) emulate the inviting character of formal British gardens

(E) reveal the distinctive personality of a homeowner

页码,19/22The Official SAT Online Course

2006-11-12file://E:\新建文件夹\b4.htm

UnRe

gistered

Page 38: SAT Online Course Test 2 Explanations

严禁用于商业用途!

Explanation for Incorrect Answer A :  Choice (A) is incorrect. The author calls attention to "a gesture of openness and welcome," not to any "preoccupation," or concern, with appearance.

Explanation for Incorrect Answer B :  Choice (B) is incorrect. The author does not mention capitalism or its values.

Explanation for Incorrect Answer D :  Choice (D) is incorrect. The passage makes it clear that British gardens are not open, welcoming, or inviting.

Explanation for Incorrect Answer E :  Choice (E) is incorrect. The statement refers to the lawns in "virtually any American suburb," linking them in a common "gesture of openness and welcome." The author does not mention individual homeowners or their distinctive personalities.

22 In line 27, “conceit” most nearly means

ANSWERS AND EXPLANATIONS Explanation for Correct Answer A :  Choice (A) is correct. "Conceit" means idea. A "grandiose ideal," an ideal more elaborate than necessary, is a fitting way to describe the "conceit" of tiny parcels of lawn pretending to look like "wide-open land."

Explanation for Incorrect Answer B :  Choice (B) is incorrect. In this context "conceit" does not refer to "vanity," or an overly favorable opinion of one’s own ability.

Explanation for Incorrect Answer C :  Choice (C) is incorrect. It would be odd to speak of an "expression" of wide-open land, or to describe maintaining wide-open land as "ingenious," or inventive.

Explanation for Incorrect Answer D :  Choice (D) is incorrect. The passage is about an idea, not a material thing. In addition, the ideal of wide-open land with fences is the opposite of "ornate," or elaborate.

Explanation for Incorrect Answer E :  Choice (E) is incorrect. An edifice is a building, not "wide-open land."

(A) grandiose ideal

(B) extreme vanity

(C) ingenious expression

(D) ornate article

(E) extravagant edifice

23

The discussion in lines 41-45 (“When Anne . . . America”) implies that Anne Bradstreet’s garden poetry

(A) reflected her disapproval of America’s rapid development

(B) followed the pattern set by more innovative British poets

(C) espoused the Puritans’ belief in religious freedom

(D) promoted greater enthusiasm for gardening in America

(E) focused on the vast American landscape rather than on individual gardens

页码,20/22The Official SAT Online Course

2006-11-12file://E:\新建文件夹\b4.htm

UnRe

gistered

Page 39: SAT Online Course Test 2 Explanations

严禁用于商业用途!

ANSWERS AND EXPLANATIONS Explanation for Correct Answer E :  Choice (E) is correct. In lines 41-45, the author's discussion centers on Anne Bradstreet's transformation of the traditional garden ode. Bradstreet changed the focus of the traditional garden ode by "[tearing] down the conventional garden wall." Removing the garden wall eliminates the separation between individual gardens. This allows "the garden" in Bradstreet's work to stand for the "vast American landscape."

Explanation for Incorrect Answer A :  Choice (A) is incorrect. The discussion of Bradstreet does not mention disapproval of America's rapid development.

Explanation for Incorrect Answer B :  Choice (B) is incorrect. In this passage, it is Bradstreet who appears innovative ("she tore down the conventional garden wall") as she alters the traditional garden ode.

Explanation for Incorrect Answer C :  Choice (C) is incorrect. This section of the passage does not discuss Puritan religious beliefs.

Explanation for Incorrect Answer D :  Choice (D) is incorrect. Bradstreet does not "promote," or advance, the practice of gardening in this passage. Further, the gardens and garden wall of Bradstreet’s poetry are figurative, not literal.

24

In line 56, the “hierarchies” most closely represent

ANSWERS AND EXPLANATIONS Explanation for Correct Answer C :  Choice (C) is correct. "Hierarchies" means a graded or ranked series. The passage describes "set[ting] off a garden from the surrounding countryside," thus creating a hierarchy or ranked series. The author clearly views the act of “set[ting] off a garden from the surrounding countryside" as both "sacrilegious" and as a misguided division of "nature's sacred space."

Explanation for Incorrect Answer A :  Choice (A) is incorrect. Although setting off a garden from the area around it is undesirable, this section of the passage does not present it as practical. In fact, the author states that it "makes no sense at all."

Explanation for Incorrect Answer B :  Choice (B) is incorrect. The author does not mention any territorial claims. Instead; emphasis is placed on the "sacrilegious" nature of "meddl[ing] with" the landscape.

Explanation for Incorrect Answer D :  Choice (D) is incorrect. Although landscape is referred to as "somehow sacred," it is not described as "pristine," or unspoiled, and development is not mentioned.

Explanation for Incorrect Answer E :  Choice (E) is incorrect. The author clearly does not see fences as an American legacy, but rather as a British one.

(A) a practical but undesirable consequence of urban life

(B) a crucial method of determining territorial claims

(C) a misguided division of nature’s sacred space

(D) an effort to protect pristine land from development

(E) an unfortunate legacy of nineteenth-century America

页码,21/22The Official SAT Online Course

2006-11-12file://E:\新建文件夹\b4.htm

UnRe

gistered

Page 40: SAT Online Course Test 2 Explanations

严禁用于商业用途!

    

Back to Score Report  

Copyright © 2006 The College Board. All rights reserved. Privacy Policy Terms of Use Contact Us

 

页码,22/22The Official SAT Online Course

2006-11-12file://E:\新建文件夹\b4.htm

UnRe

gistered

Page 41: SAT Online Course Test 2 Explanations

严禁用于商业用途!

Help | Profile | My Organizer | My Bookmarks | Logout

Answers and Explanations

Test Sections

Section 1

Section 2

Section 3

Section 4

Section 6

Section 7

Section 8

Section 9

Section 10

Back to Score Report  

View Answers and Explanations     Online - Practice Test #2

1 If then

ANSWERS AND EXPLANATIONS Explanation for Correct Answer D : 

Choice (D) is correct. Since

Explanation for Incorrect Answer A :  Choice (A) is not correct. When 0.0001 is substituted for in the equation, the result is which is not equal to

Explanation for Incorrect Answer B :  Choice (B) is not correct. When is substituted for in the equation, the result is which is not equal to

Explanation for Incorrect Answer C :  Choice (C) is not correct. When is substituted for in the equation, the result

is which is not equal to

Explanation for Incorrect Answer E :  Choice (E) is not correct. When is substituted for in the equation, the

result is which is not equal to

(A)

(B)

(C)

(D)

(E)

2 If is less than and is more than what is the value of when

ANSWERS AND EXPLANATIONS Explanation for Correct Answer A :  Choice (A) is correct. If is less than and is more than then

and When and

(A)

(B)

(C)

(D)

(E)

页码,1/11The Official SAT Online Course

2006-11-12file://E:\新建文件夹\b5.htm

UnRe

gistered

Page 42: SAT Online Course Test 2 Explanations

严禁用于商业用途!

Explanation for Incorrect Answer B :  Choice (B) is not correct. If the value of were then so the value of

would be If then and the value of would be not

Explanation for Incorrect Answer C :  Choice (C) is not correct. If the value of were then so the value of

would be If then and the value of would be not

Explanation for Incorrect Answer D :  Choice (D) is not correct. If the value of were then so the value of

would be If then and the value of would be not

Explanation for Incorrect Answer E :  Choice (E) is not correct. If the value of were then so the value of

would be If then and the value of would be not

3

The pictograph above shows the results of a survey in which people were asked to indicate which of four hot beverages they drink. How many more responses did Coffee receive than Hot cider?

ANSWERS AND EXPLANATIONS Explanation for Correct Answer D :  Choice (D) is correct. In the pictograph, the responses for Coffee are shown by a row of full cups, whereas the responses for Hot cider are shown by a row of only

cups. Since each cup corresponds to responses, Coffee received

responses, while Hot cider received responses, so Coffee received more responses than Hot cider.

Explanation for Incorrect Answer A :  Choice (A) is not correct. is the number of responses that Hot cider received, but the question asks for the difference between the number of responses Coffee received and the number of responses Hot cider received.

Explanation for Incorrect Answer B :  Choice (B) is not correct. is the number of responses that Hot chocolate received, but the question asks for the difference between the number of responses Coffee received and the number of responses Hot cider received.

Explanation for Incorrect Answer C : 

(A)

(B)

(C)

(D)

(E)

页码,2/11The Official SAT Online Course

2006-11-12file://E:\新建文件夹\b5.htm

UnRe

gistered

Page 43: SAT Online Course Test 2 Explanations

严禁用于商业用途!

Choice (C) is not correct. There are four and one-half more full cups in the Coffee row than the in Hot cider row and the half-cup represents responses.

Explanation for Incorrect Answer E :  Choice (E) is not correct. is the number of responses that Coffee received, but the question asks for the difference between the number of responses Coffee received and the number of responses Hot cider received.

4 If is a positive even integer, then could equal which of the

following?

ANSWERS AND EXPLANATIONS Explanation for Correct Answer C : 

Choice (C) is correct. Since is positive and even, is a positive odd

number, and is a positive even number, one more than is the

only number among the options that is the product of consecutive integers of which the smaller is odd. That is

Explanation for Incorrect Answer A :  Choice (A) is not correct. The factors and are consecutive positive

integers. The only ways to write as the product of two positive integers and where are as and In each of the products, the integers

are not consecutive.

Explanation for Incorrect Answer B :  Choice (B) is not correct. The factors and are consecutive positive

integers. The only ways to write as the product of two positive integers and where are as and In the first two products, the

integers are not consecutive. The last product corresponds to which is not an even integer.

Explanation for Incorrect Answer D :  Choice (D) is not correct. The factors and are consecutive positive

integers. The only ways to write as the product of two positive integers and where are as and In each of the

products, the integers are not consecutive.

Explanation for Incorrect Answer E :  Choice (E) is not correct. The factors and are consecutive positive

integers. The only ways to write as the product of two positive integers and where are as and In each of the products, the

integers are not consecutive.

(A)

(B)

(C)

(D)

(E)

5

页码,3/11The Official SAT Online Course

2006-11-12file://E:\新建文件夹\b5.htm

UnRe

gistered

Page 44: SAT Online Course Test 2 Explanations

严禁用于商业用途!

In the figure above, circular region represents shirts with pockets, circular region

represents shirts with buttons, and circular region represents shirts with collars. What is represented by the shaded region?

ANSWERS AND EXPLANATIONS Explanation for Correct Answer C : 

Choice (C) is correct. The shaded region covers all of the area where and

overlap, so every shirt represented by this region must have both pockets and buttons. Some of the shaded region is in so some of these shirts have collars.

Explanation for Incorrect Answer A :  Choice (A) is not correct. Shirts with pockets, buttons, and collars would be

represented by only the small region in the center where all three regions

and overlap. The shaded region includes but is larger than this small center region.

Explanation for Incorrect Answer B :  Choice (B) is not correct. If none of these shirts have collars, then none of the shaded region would be in

Explanation for Incorrect Answer D :  Choice (D) is not correct. These shirts would be represented by the region where

and overlap.

Explanation for Incorrect Answer E :  Choice (E) is not correct. These shirts would be represented by the region where

and overlap.

(A) Shirts with pockets, buttons, and collars

(B) Shirts with pockets and buttons, but without collars

(C) Shirts with pockets and buttons (some possibly with collars)

(D) Shirts with pockets and collars (some possibly with buttons)

(E) Shirts with buttons and collars (some possibly with pockets)

6

页码,4/11The Official SAT Online Course

2006-11-12file://E:\新建文件夹\b5.htm

UnRe

gistered

Page 45: SAT Online Course Test 2 Explanations

严禁用于商业用途!

In the figure above, the slope of line is What is the value of

ANSWERS AND EXPLANATIONS Explanation for Correct Answer E :  Choice (E) is correct. The slope is the change in divided by the change in so

This simplifies to

Explanation for Incorrect Answer A :  Choice (A) is not correct. If the value of were then the slope of line would

be not

Explanation for Incorrect Answer B : 

Choice (B) is not correct. If the value of were then the slope of line would

be not

Explanation for Incorrect Answer C : 

Choice (C) is not correct. If the value of were then the slope of line would

be not

Explanation for Incorrect Answer D :  Choice (D) is not correct. If the value of were then the slope of line would

be not

(A)

(B)

(C)

(D)

(E)

7If and where which of the following must be equal to

ANSWERS AND EXPLANATIONS

(A)

(B)

(C)

(D)

(E)

页码,5/11The Official SAT Online Course

2006-11-12file://E:\新建文件夹\b5.htm

UnRe

gistered

Page 46: SAT Online Course Test 2 Explanations

严禁用于商业用途!

Explanation for Correct Answer A : 

Choice (A) is correct. Multiplying both sides of the equation by gives the

equation Since it follows that and therefore

Explanation for Incorrect Answer B : 

Choice (B) is not correct. If were equal to then would be equal to

Multiplying both sides of the equation by gives the equation so

would also equal However, is equal to

Explanation for Incorrect Answer C : 

Choice (C) is not correct. If and have values and

respectively, then and However, but Therefore,

does not equal in this case.

Explanation for Incorrect Answer D : 

Choice (D) is not correct. If and have values and

respectively, then and However, but Therefore,

does not equal in this case.

Explanation for Incorrect Answer E : 

Choice (E) is not correct. If and have values and

respectively, then and However, but Therefore,

does not equal in this case.

8

In the figure above, lies on In terms of which of the following must be

equivalent to

(A)

(B)

(C)

(D)

(E)

页码,6/11The Official SAT Online Course

2006-11-12file://E:\新建文件夹\b5.htm

UnRe

gistered

Page 47: SAT Online Course Test 2 Explanations

严禁用于商业用途!

ANSWERS AND EXPLANATIONS Explanation for Correct Answer B : 

Choice (B) is correct. The measure of is given by the expression

This simplifies to The sum of the measures

of angle angle and is Hence, is equivalent

to which simplifies to Thus, must be equivalent to

Explanation for Incorrect Answer A :  Choice (A) is not correct. If were equivalent to then

would equal which would lead to the false

statement

Explanation for Incorrect Answer C :  Choice (C) is not correct. If were equivalent to then

would equal which would lead to the

statement which is false for the given figure because is an angle of

positive measure.

Explanation for Incorrect Answer D :  Choice (D) is not correct. If were equivalent to then

would equal which would lead to the

statement Therefore, is equivalent to only if which

is not necessarily true for the given figure. The question asks for an expression that must be equivalent to

Alternatively, a close examination of the figure (which is drawn to scale) shows that

the measure of which equals is less than so

Explanation for Incorrect Answer E :  Choice (E) is not correct. If were equivalent to then

would equal which would lead to the

statement Therefore, is equivalent to only if which is not necessarily true for the given figure. The question asks for an expression that must be equivalent to

In fact, a close examination of the figure (which is drawn to scale) shows that the

measure of , which equals is less than so cannot be

equivalent to

9If what is the value of

Your Response:  

Correct Response(s):  5/2, 2.5

Explanation:  

The correct answer is or . Since the equation also holds,

so This answer can also be expressed in decimal form as .

10A recipe for making loaves of bread requires cups of flour and tablespoons of baking powder. If the proportions in this recipe are to be used to make loaves of

页码,7/11The Official SAT Online Course

2006-11-12file://E:\新建文件夹\b5.htm

UnRe

gistered

Page 48: SAT Online Course Test 2 Explanations

严禁用于商业用途!

bread, how many cups of flour will be needed? (Do not round your answer.)

Your Response:  

Correct Response(s):  36/5, 7.2

Explanation:  

The correct answer is or .

If the same proportions of the recipe are used to make loaves of bread and

loaves of bread, then Therefore,

so Alternatively, cups of flour are required for each

of the loaves, so cups of flour would be needed to make

loaves of bread.

This answer can also be expressed as

11

In the figure above, and intersect at If and bisects

what is the value of

Your Response:  

Correct Response(s):  40

Explanation:  

The correct answer is . Since and intersect at the measure of

which is is the same as the measure of Since bisects

the measure of which is is half of So

12The length and width of a rectangle have integer values. If the area of the rectangle is what is one possible value for the perimeter of the rectangle?

Your Response:  

Correct Response(s):  40 or 56 or 152

Explanation:  

The possible correct answers are and The area of the rectangle is so where is the length and is the width. Since and

must be integers, the only possible dimensions of this rectangle are and The perimeter of the rectangle is so the possible

页码,8/11The Official SAT Online Course

2006-11-12file://E:\新建文件夹\b5.htm

UnRe

gistered

Page 49: SAT Online Course Test 2 Explanations

严禁用于商业用途!

values of the perimeter are and

13

A sequence is formed by repeating the numbers above in the same order indefinitely. What is the sum of the first terms of the sequence?

Your Response:  

Correct Response(s):  14

Explanation:  

The correct answer is . Since the first terms of the sequence repeat, the sum of the first terms of the sequence is equivalent to the sum of the first terms

of the sequence multiplied by So the sum is

14

In a survey, people responded to the following question: “How many weeks of vacation did you take last year?” Their responses to the question are summarized in the chart above, where equals the number of vacation weeks indicated. How many respondents took more than weeks of vacation last year?

Your Response:  

Correct Response(s):  2380

Explanation:  

The correct answer is . According to the graph, of the

respondents took weeks or less of vacation. So, of the respondents

took more than weeks of vacation. Therefore, of the

respondents took more than weeks vacation last year.

15

If for all values of where

and are constants, what is the value of

Your Response:  

Correct Response(s):  103

Explanation:  

页码,9/11The Official SAT Online Course

2006-11-12file://E:\新建文件夹\b5.htm

UnRe

gistered

Page 50: SAT Online Course Test 2 Explanations

严禁用于商业用途!

The correct answer is . Since the equation is true for all values of then it

must be true when So, if

and, if

so

16

In the -plane above, the area of is What is the value of

Your Response:  

Correct Response(s):  4

Explanation:  

The correct answer is . The formula for the area of a triangle is where

is the length of the base and is the height. The height of is the distance

from to the -axis, which is The length of the base is the distance from

to which is also So, the area of is Therefore,

and

17 The total cost of a taxicab ride is the sum of (1) a basic fixed charge for using the taxicab, and

(2) an additional charge for each of a mile that is traveled.

If the total cost to ride mile is and the total cost to ride miles is

what is the total cost, in dollars, of a -mile ride? (Disregard the $ sign when gridding your answer. If, for example, your answer is

grid )

Your Response:  

Correct Response(s):  8.50, 17/2

Explanation:  

The correct answer is . The formula to find the total cost of a taxicab ride is

where is the basic fixed charge, is the charge for each of a mile

traveled, and is the number of miles traveled. The cost to ride of a mile is

页码,10/11The Official SAT Online Course

2006-11-12file://E:\新建文件夹\b5.htm

UnRe

gistered

Page 51: SAT Online Course Test 2 Explanations

严禁用于商业用途!

so The cost to ride miles is so

. The solution to this system of equations is and

Therefore, the total cost to ride miles is .

This answer can be gridded in decimal form as or in fraction form as

18 Let the function be defined so that is the area of a semicircle with diameter If what is the value of

Your Response:  

Correct Response(s):  10

Explanation:  

The correct answer is . If the function is defined to be the area of a

semicircle with diameter then So, the equation

can be written as Solving for yields

    

Back to Score Report  

Copyright © 2006 The College Board. All rights reserved. Privacy Policy Terms of Use Contact Us

 

页码,11/11The Official SAT Online Course

2006-11-12file://E:\新建文件夹\b5.htm

UnRe

gistered

Page 52: SAT Online Course Test 2 Explanations

严禁用于商业用途!

Help | Profile | My Organizer | My Bookmarks | Logout

Answers and Explanations

Test Sections

Section 1

Section 2

Section 3

Section 4

Section 6

Section 7

Section 8

Section 9

Section 10

Back to Score Report  

View Answers and Explanations     Online - Practice Test #2

1 A stranger, the students were surprised to see him enter the classroom carrying a bowling ball.

ANSWERS AND EXPLANATIONS Explanation for Correct Answer D :  Choice (D) is correct. It avoids the error of the original by placing the subject, "the students," at the beginning and deleting the repetitive pronoun "him."

Explanation for Incorrect Answer A :  Choice (A) involves illogical word order. "A stranger," the object of the verb phrase "were surprised to see," is placed before the subject, "the students"; it would be more logical to place it just after the verb phrase and delete the repetitive pronoun "him."

Explanation for Incorrect Answer B :  Choice (B) results in illogical word order. The phrase "A stranger carrying a bowling ball" is placed at the beginning of the sentence; it would be more logical to replace the pronoun "him" with that phrase.

Explanation for Incorrect Answer C :  Choice (C) involves improper coordination. It provides a coordinate clause, "and he carried a bowling ball," instead of the modifying phrase that is needed for clarity and conciseness.

Explanation for Incorrect Answer E :  Choice (E) results in a sentence fragment. The relative pronoun "who" creates a subordinate clause ("were surprised . . . bowling ball"), leaving an incomplete main clause.

(A) A stranger, the students were surprised to see him enter the classroom carrying a bowling ball.

(B) A stranger carrying a bowling ball, the students were surprised to see him entering the classroom.

(C) The students were surprised to see a stranger enter the classroom, and he carried a bowling ball.

(D) The students were surprised to see a stranger carrying a bowling ball enter the classroom.

(E) The students, who were surprised to see a stranger enter the classroom carrying a bowling ball.

2

Several of Frank Stella’s paintings were inspired by the shapes of waves and whales, titled after chapter headings from Moby-Dick.

(A) paintings were inspired by the shapes of waves and whales, titled

(B) paintings had their inspiration from the shapes of waves and whales with titles

(C) paintings, inspired by the shapes of waves and whales, are titled

(D) paintings, which were inspired by the shapes of waves and whales and which are titled

(E) paintings, being inspired by the shapes of waves and whales, titled

页码,1/21The Official SAT Online Course

2006-11-12file://E:\新建文件夹\b6.htm

UnRe

gistered

Page 53: SAT Online Course Test 2 Explanations

严禁用于商业用途!

ANSWERS AND EXPLANATIONS Explanation for Correct Answer C :  Choice (C) is correct. It avoids the error of the original by placing the verbal phrase, "inspired by the shapes of waves and whales," immediately after the noun it modifies, "paintings," and by making "several" the subject of the new verb "are titled."

Explanation for Incorrect Answer A :  Choice (A) displays improper modification and word order. The verbal phrase beginning with "titled" cannot logically modify the nouns immediately before it, "waves and whales."

Explanation for Incorrect Answer B :  Choice (B) exhibits improper modification. The prepositional phrase "with titles" cannot logically modify the noun immediately before it, "whales."

Explanation for Incorrect Answer D :  Choice (D) creates a sentence fragment. It contains verbs in dependent clauses ("were inspired " and "are titled") but no independent verb that can complete a statement.

Explanation for Incorrect Answer E :  Choice (E) produces a sentence fragment. Since it contains no verb (only the verbals "being inspired" and "titled"), it does not make a complete statement.

3 The mayor claimed that a majority of the property owners would have favored her proposal if put to the vote.

ANSWERS AND EXPLANATIONS Explanation for Correct Answer B :  Choice (B) is correct. It avoids the error of the original by making clear that it is the "proposal" that will be "put to the vote."

Explanation for Incorrect Answer A :  Choice (A) involves improper modification. The phrase "put to the vote" could refer either to "the property owners" or to the "proposal."

Explanation for Incorrect Answer C :  Choice (C) involves an illogical sequence of verb tenses. The verb "favored" indicates an event that did happen, but "would have been put" indicates that the conditions for that event have not happened.

Explanation for Incorrect Answer D :  Choice (D) results in improper modification. The phrase "put to the vote" could refer either to "the property owners" or to the "proposal."

Explanation for Incorrect Answer E :  Choice (E) involves improper modification. The plural pronoun "they" should be the singular "it" to agree with the singular "proposal."

(A) would have favored her proposal if put

(B) would have favored her proposal if it had been put

(C) favored her proposal if it would have been put

(D) favored her proposal if put

(E) favored her proposal if they were put

4The psychologist states that most people want the same things: interesting and

页码,2/21The Official SAT Online Course

2006-11-12file://E:\新建文件夹\b6.htm

UnRe

gistered

Page 54: SAT Online Course Test 2 Explanations

严禁用于商业用途!

meaningful work, respect, and to have them be loved for themselves alone.

ANSWERS AND EXPLANATIONS Explanation for Correct Answer E :  Choice (E) is correct. It avoids the error of the original by providing three parallel phrases, introduced by infinitives, to describe the things most people want.

Explanation for Incorrect Answer A :  Choice (A) involves a lack of parallelism. The expression "to have them be loved . . . alone" is an infinitive phrase. It thus breaks the pattern of successive noun phrases ("interesting and meaningful work" and "respect") established earlier in the sentence.

Explanation for Incorrect Answer B :  Choice (B) results in a lack of parallelism. The three items in the list are an infinitive phrase ("to have . . . work"), a noun ("respect"), and another infinitive phrase ("loved . . . alone"), instead of the three grammatically similar phrases that are needed.

Explanation for Incorrect Answer C :  Choice (C) is unsatisfactory because it results in a lack of parallelism. The expression "work . . . meaningful" does not match the pattern established by the pair of infinitive phrases that follow it.

Explanation for Incorrect Answer D :  Choice (D) involves a problem with diction. The phrase "their own love" has a meaning much different from "to be loved for themselves alone."

(A) interesting and meaningful work, respect, and to have them be loved for themselves alone

(B) to have interesting and meaningful work, respect, and loved for themselves alone

(C) work that has interest and is meaningful, to have respect, and to be beloved for themselves alone

(D) interesting and meaningful work, respect, and their own love

(E) to have interesting and meaningful work, to be respected, and to be loved for themselves alone

5By employing exotic harmonies and making unusual use of instruments, Mahler was a pathfinder from romanticism to modern music.

ANSWERS AND EXPLANATIONS Explanation for Correct Answer C :  Choice (C) is correct. It avoids the error of the original by using the verb "created" and the noun "path" to form the appropriate idiom.

Explanation for Incorrect Answer A :  Choice (A) results in an improper idiom. The verb "was" and the predicate noun "pathfinder" precede the preposition "from," where it would be more idiomatic to use the verb "created" and the direct object "a path."

Explanation for Incorrect Answer B :  Choice (B) creates improper modification. The introductory phrase, "By employing . . . instruments," refers improperly to "a path" rather than to "Mahler."

(A) Mahler was a pathfinder

(B) a path was created by Mahler

(C) Mahler created a path

(D) Mahler was the creator of a path

(E) was how Mahler created a path

页码,3/21The Official SAT Online Course

2006-11-12file://E:\新建文件夹\b6.htm

UnRe

gistered

Page 55: SAT Online Course Test 2 Explanations

严禁用于商业用途!

Explanation for Incorrect Answer D :  Choice (D) results in wordiness. The phrase "was the creator of a path" could more simply be expressed as "created a path."

Explanation for Incorrect Answer E :  Choice (E) involves improper modification. The introductory phrase "By employing . . . instruments" is improperly placed next to the verb "was" rather than next to the noun "Mahler," which it modifies.

6 In the past, many famous painters meticulously ground their own colors, an attention to detail that is noteworthy.

ANSWERS AND EXPLANATIONS Explanation for Correct Answer A :  Choice (A) is correct. It uses a concise noun phrase ("an attention to detail that is noteworthy") to embed appropriate information within a single clause.

Explanation for Incorrect Answer B :  Choice (B) uses improper coordination. It links two complete thoughts ("In the past . . . ground their own colors" and "inasmuch . . . it is noteworthy") with only a comma.

Explanation for Incorrect Answer C :  Choice (C) displays improper coordination. It uses only a comma to link two complete thoughts ("In the past . . . ground their own colors" and "this makes it . . . to detail").

Explanation for Incorrect Answer D :  Choice (D) includes excess words. With a slight change in word order ("an attention to detail that is noteworthy"), the words "idea" and "in showing their" become unnecessary.

Explanation for Incorrect Answer E :  Choice (E) uses vague pronouns. The pronouns "which" and "it" may allude to the act of grinding colors, but the sentence contains no noun to which they can refer.

(A) an attention to detail that is noteworthy

(B) inasmuch as they showed attention to detail, it is noteworthy

(C) this makes it noteworthy in showing their attention to detail

(D) an idea that is noteworthy in showing their attention to detail

(E) which is noteworthy and it shows an attention to detail

7

By including pieces of cloth, newspaper, wallpaper, and other materials in his work, Picasso’s innovation had an important influence on twentieth-century art.

ANSWERS AND EXPLANATIONS Explanation for Correct Answer E :  Choice (E) is correct. It avoids the error of the original by providing an appropriate subject, "the innovative Picasso," for the modifying phrase "By including . . . work."

(A) Picasso’s innovation had an important influence on

(B) this innovation of Picasso’s was important in its influence over

(C) Picasso’s important innovative influence was on

(D) Picasso was influential, with his innovation, over

(E) the innovative Picasso was an important influence on

页码,4/21The Official SAT Online Course

2006-11-12file://E:\新建文件夹\b6.htm

UnRe

gistered

Page 56: SAT Online Course Test 2 Explanations

严禁用于商业用途!

Explanation for Incorrect Answer A :  Choice (A) involves improper modification. It improperly makes "Picasso's innovation" the subject of the modifying phrase "By including . . . in his work."

Explanation for Incorrect Answer B :  Choice (B) is unsatisfactory because it results in improper modification. The phrase "By including . . . in his work" cannot logically modify "this innovation."

Explanation for Incorrect Answer C :  Choice (C) results in an illogical sentence because of improper modification. The phrase "By including . . . in his work" cannot logically modify "Picasso's . . . influence."

Explanation for Incorrect Answer D :  Choice (D) exhibits awkward phrasing. It is strange to say that Picasso was "influential, with his innovation, over twentieth-century art."

8 Once American films looked slick and commercial compared to European imports; now, almost the reverse is true.

ANSWERS AND EXPLANATIONS Explanation for Correct Answer A :  Choice (A) is correct. The adverb "now" appropriately introduces the comparison completed by "almost the reverse is true."

Explanation for Incorrect Answer B :  Choice (B) results in improper use of a pronoun. The pronoun "they" is ambiguous, since it can refer to either "American films" or "European imports."

Explanation for Incorrect Answer C :  Choice (C) results in a connective error. The use of the adverb "instead" improperly suggests that the two clauses express alternatives rather than opposition.

Explanation for Incorrect Answer D :  Choice (D) involves wordiness. Neither the phrase "it is" nor the phrase "that is" is necessary to the clause.

Explanation for Incorrect Answer E :  Choice (E) results in improper pronoun use. The singular pronoun "it" has no singular antecedent in the sentence.

(A) now, almost the reverse is true

(B) now they are almost the reverse

(C) instead, there is almost a reversal now

(D) now it is almost the reverse that is true

(E) it has now been almost reversed

9

Although known primarily as a poet, the paintings of Lawrence Ferlinghetti have recently been receiving public attention.

(A) Although known primarily as a poet, the paintings of Lawrence Ferlinghetti

(B) Although known primarily for his poetry, Lawrence Ferlinghetti’s paintings

(C) Although his poetry is primarily what he is known for, Lawrence Ferlinghetti’s paintings

(D) Although Lawrence Ferlinghetti is known primarily as a poet and his paintings

(E) Although Lawrence Ferlinghetti is known primarily for his poetry, his paintings

页码,5/21The Official SAT Online Course

2006-11-12file://E:\新建文件夹\b6.htm

UnRe

gistered

Page 57: SAT Online Course Test 2 Explanations

严禁用于商业用途!

ANSWERS AND EXPLANATIONS Explanation for Correct Answer E :  Choice (E) is correct. It avoids the error of the original by supplying the introductory clause with a subject and verb (“Lawrence Ferlinghetti is”), thereby making it clear that Ferlinghetti is known as a poet.

Explanation for Incorrect Answer A :  Choice (A) results in improper modification. The introductory adjective phrase (“Although . . . Ferlinghetti") refers improperly to “the paintings,” when it should refer to Ferlinghetti.

Explanation for Incorrect Answer B :  Choice (B) creates improper modification. The opening adjective phrase (“Although . . . poetry”) improperly modifies Ferlinghetti’s “paintings” rather than Ferlinghetti.

Explanation for Incorrect Answer C :  Choice (C) involves ambiguous reference. The sentence does not contain a noun antecedent for the pronoun “his.”

Explanation for Incorrect Answer D :  Choice (D) results in improper coordination. The subordinate clause “Although . . . poetry” is improperly joined to an independent clause “his paintings . . . attention” with a coordinating conjunction rather than with a comma.

10 For many a brilliant architect, being free to innovate is more important than being well paid.

ANSWERS AND EXPLANATIONS Explanation for Correct Answer A :  Choice (A) is correct. It uses parallel verbal phrases ("being free" and "being well paid") to establish a clear comparison.

Explanation for Incorrect Answer B :  Choice (B) has a flaw in parallelism. The verbal phrase "having freedom" is not parallel with the later verbal phrase "being well paid."

Explanation for Incorrect Answer C :  Choice (C) violates parallelism. The independent clause "there is more importance . . . to innovate" is not parallel with the later verbal phrase "being well paid."

Explanation for Incorrect Answer D :  Choice (D) fails to maintain parallelism. The noun "freedom" is not parallel with the later verbal "being."

Explanation for Incorrect Answer E :  Choice (E) displays a flaw in parallelism. The verbal phrase "to have the freedom" is not parallel with the later verbal phrase "being well paid."

(A) being free to innovate is more important than

(B) having freedom of innovation is more important than

(C) there is more importance in the freedom to innovate than

(D) freedom to innovate has more importance than

(E) to have the freedom to innovate is more important than

11 What was not achieved in last year’s county voter registration drive was more than compensated for by this year, which registered over three thousand new voters.

页码,6/21The Official SAT Online Course

2006-11-12file://E:\新建文件夹\b6.htm

UnRe

gistered

Page 58: SAT Online Course Test 2 Explanations

严禁用于商业用途!

ANSWERS AND EXPLANATIONS Explanation for Correct Answer C :  Choice (C) is correct. It avoids the error of the original by comparing two parallel nouns, “last year’s . . . drive” and “this year’s drive.”

Explanation for Incorrect Answer A :  Choice (A) involves a lack of parallelism. A voter registration drive, “last year’s . . . drive,” is not parallel with and cannot be compared to a year, “this year.”

Explanation for Incorrect Answer B :  Choice (B) creates improper modification. The adjective phrase “having . . . registered” refers improperly to a year, “this year,” rather than to a registration drive.

Explanation for Incorrect Answer D :  Choice (D) results in improper use of a pronoun. The pronoun “they” is ambiguous, since the sentence does not contain a noun to which it can refer.

Explanation for Incorrect Answer E :  Choice (E) creates improper pronoun use. The pronoun “they” is ambiguous because it does not refer to any noun in the sentence.

(A) by this year, which registered over three thousand new voters

(B) by this year, having over three thousand new voters registered

(C) by this year’s drive, which registered over three thousand new voters

(D) when they registered three thousand new voters this year

(E) this year, when they registered over three thousand new voters

12

the cyclist exploring Ireland’s western cliffs, every road leading the town

of Clifden offers   set of wonders. 

ANSWERS AND EXPLANATIONS Corrected Sentence: For the cyclist exploring Ireland’s western cliffs, every road leading out of the town of Clifden offers its own set of wonders.

Explanation for Correct Answer C :  The error in this sentence occurs at (C), where the number of the pronoun is incorrect. The plural pronoun "their" does not agree with the singular noun to which it refers, "road."

Explanation for Incorrect Answer A :  There is no error at (A). The preposition "for" is appropriate to introduce a phrase identifying who might take the road.

Explanation for Incorrect Answer B :  There is no error at (B). The phrasal preposition "out of" appropriately links the noun "road" with the later noun "town."

Explanation for Incorrect Answer D :  There is no error at (D). The adjective "own" correctly modifies the noun immediately after it, "set."

Explanation for Incorrect Answer E :  There is an error in the sentence.

For out of

their own No error

页码,7/21The Official SAT Online Course

2006-11-12file://E:\新建文件夹\b6.htm

UnRe

gistered

Page 59: SAT Online Course Test 2 Explanations

严禁用于商业用途!

13

The construction of a waterway linking the Atlantic and Pacific Oceans

in 1524, the Panama Canal opened in 1914 did

become . 

ANSWERS AND EXPLANATIONS Corrected Sentence: 

Explanation for Correct Answer E :  There is no error in this sentence.

Explanation for Incorrect Answer A :  There is no error at (A). The singular verb phrase "was . . . proposed" agrees with its singular subject, "construction," and the adjective "first" is appropriately placed.

Explanation for Incorrect Answer B :  There is no error at (B). The word "but" provides a link between the two clauses; the phrase "not until" combines with the verb phrase "did . . . become" to form an appropriate idiom.

Explanation for Incorrect Answer C :  There is no error at (C). The phrase "such a project" is appropriately used to refer to the phrase "The construction . . . Oceans."

Explanation for Incorrect Answer D :  There is no error at (D). The noun phrase "a reality" is properly used to refer to the phrase "such a project."

14

The charm Lofting’s book the humorous reversal roles—the animals

guide, assist, and generally care of the helpless humans. 

ANSWERS AND EXPLANATIONS Corrected Sentence: The charm of Lofting's book lies in the humorous reversal of roles— the animals guide, assist, and generally take care of the helpless humans.

Explanation for Correct Answer D :  The error in this sentence occurs at (D), where there is a lack of parallelism. The pronoun "they" introduces a clause, breaking the pattern of successive verbs that precedes it.

Explanation for Incorrect Answer A :  There is no error at (A). The preposition "of" is properly used in combination with the noun phrase "Lofting's book" to form a possessive.

Explanation for Incorrect Answer B :  There is no error at (B). The verb "lies" agrees with its singular subject "charm" and combines with the preposition "in" to form an appropriate idiom.

was first proposed but not until

such a project a reality No error

of lies in of

they take No error

页码,8/21The Official SAT Online Course

2006-11-12file://E:\新建文件夹\b6.htm

UnRe

gistered

Page 60: SAT Online Course Test 2 Explanations

严禁用于商业用途!

Explanation for Incorrect Answer C :  There is no error at (C). The preposition "of" combines with the nouns "reversal" and "roles" to form an appropriate idiom.

Explanation for Incorrect Answer E :  There is an error in the sentence.

15

People were trained one tiny part of one process in one department of one

industry, and so no sense the process . 

ANSWERS AND EXPLANATIONS Corrected Sentence: People were trained to perform one tiny part of one process in one department of one industry, and so they had no sense of the process as a whole.

Explanation for Correct Answer B :  The error in this sentence occurs at (B), where a failure to maintain parallelism leaves the phrase after the comma incomplete. Since the verbal "having" has no subject, it is not parallel with the earlier clause "People were trained."

Explanation for Incorrect Answer A :  There is no error at (A). The infinitive "to perform" correctly begins a phrase describing how people were trained.

Explanation for Incorrect Answer C :  There is no error at (C). The preposition "of" functions properly to link its object, the noun "process," with the earlier noun "sense."

Explanation for Incorrect Answer D :  There is no error at (D). The prepositional phrase "as a whole" correctly modifies the noun "process."

Explanation for Incorrect Answer E :  There is an error in the sentence.

16

The Stegosaurus, with bony plates and tail spikes,

was Colorado. 

ANSWERS AND EXPLANATIONS Corrected Sentence: The Stegosaurus, a plant-eating dinosaur with protective bony plates and tail spikes, was once common in what is now Colorado.

Explanation for Correct Answer A :  The error in this sentence occurs at (A), where there is noun-noun disagreement and subject-verb disagreement. The plural noun "dinosaurs" is an improper appositive for the singular subject, "the Stegosaurus," and it does not agree with the singular verb "was."

Explanation for Incorrect Answer B : 

to perform

having of as a whole No error

plant-eating dinosaurs protective

once common in what is now No error

页码,9/21The Official SAT Online Course

2006-11-12file://E:\新建文件夹\b6.htm

UnRe

gistered

Page 61: SAT Online Course Test 2 Explanations

严禁用于商业用途!

There is no error at (B). "Protective" is an appropriate adjective to modify the noun "plates."

Explanation for Incorrect Answer C :  There is no error at (C). The adverb "once" functions properly to tell when something happened, the adjective "common" is used correctly after the linking verb "was" to describe "the Stegosaurus," and the preposition "in" is appropriate to introduce a phrase telling where.

Explanation for Incorrect Answer D :  There is no error at (D). The noun clause "what is now" serves appropriately as the object of the preposition "in."

Explanation for Incorrect Answer E :  There is an error in the sentence.

17

Some plants use chemical signals insects, and signals help to

put neighboring plants on alert   . 

ANSWERS AND EXPLANATIONS Corrected Sentence: Some plants use chemical signals that repel insects, and these signals help to put neighboring plants on alert so they can mount their own defenses.

Explanation for Correct Answer B :  The error in this sentence occurs at (B), where the adverb "also," in addition to the conjunction "and," results in wordiness.

Explanation for Incorrect Answer A :  There is no error at (A). The relative pronoun "that" and the verb "repel" are appropriately combined to begin a relative clause modifying "signals."

Explanation for Incorrect Answer C :  There is no error at (C). The conjunction "so" is properly joined to the subject "they" and the verb "can" to begin a clause. The plural pronoun "they" agrees with the plural noun "plants," to which it refers.

Explanation for Incorrect Answer D :  There is no error at (D). The verb "mount" correctly precedes the noun phrase "their own defenses" to form an appropriate idiom.

Explanation for Incorrect Answer E :  There is an error in the sentence.

18

Innovative use of computers in the classroom allows students projects

that   to be analytical and intellectually adventurous. 

ANSWERS AND EXPLANATIONS Corrected Sentence: Innovative use of computers in the classroom allows students to undertake projects that encourage them to be both analytical and intellectually adventurous.

Explanation for Correct Answer B : 

that repel also, these

so they can mount their own defenses No error

to undertake

encourages them both No error

页码,10/21The Official SAT Online Course

2006-11-12file://E:\新建文件夹\b6.htm

UnRe

gistered

Page 62: SAT Online Course Test 2 Explanations

严禁用于商业用途!

The error in this sentence occurs at (B), where there is subject-verb disagreement. The singular verb "encourages" does not agree with its plural subject, the relative pronoun "that," which refers to the plural noun "projects."

Explanation for Incorrect Answer A :  There is no error at (A). The infinitive "to undertake" properly introduces the noun phrase that operates as the direct object of the verb "allows."

Explanation for Incorrect Answer C :  There is no error at (C). The plural pronoun "them" agrees with the plural noun "students," to which it refers.

Explanation for Incorrect Answer D :  There is no error at (D). The conjunction "both" combines with "and" to link the two qualities that students are encouraged to have.

Explanation for Incorrect Answer E :  There is an error in the sentence.

19

When one the customs of a community, must its

history and observe its people going about ordinary activities.  

ANSWERS AND EXPLANATIONS Corrected Sentence: When one is researching the customs of a community, one must learn about its history and observe its people going about their ordinary activities.

Explanation for Correct Answer B :  The error in this sentence occurs at (B), where there is an improper pronoun shift. The person who “is researching” is also the person who “must learn,” and therefore should be referred to by the pronoun “one” rather than by “you.”

Explanation for Incorrect Answer A :  There is no error at (A). The singular verb “is researching” agrees with its singular subject “one.”

Explanation for Incorrect Answer C :  There is no error at (C). The verb “learn” and the preposition “about” combine to form an appropriate idiom.

Explanation for Incorrect Answer D :  There is no error at (D). The plural pronoun “their” agrees with its plural antecedent “people.”

Explanation for Incorrect Answer E :  There is an error in the sentence.

20

consummate skill, Picasso sketched a the youthful

experienced dancer who was him. 

is researching you learn about

their No error

Working with portrait of but

posing for No error

页码,11/21The Official SAT Online Course

2006-11-12file://E:\新建文件夹\b6.htm

UnRe

gistered

Page 63: SAT Online Course Test 2 Explanations

严禁用于商业用途!

ANSWERS AND EXPLANATIONS Corrected Sentence: 

Explanation for Correct Answer E :  There is no error in this sentence.

Explanation for Incorrect Answer A :  There is no error at (A). The participle "Working" and the preposition "with" are appropriately used to introduce a modifying phrase.

Explanation for Incorrect Answer B :  There is no error at (B). The noun "portrait" and the preposition "of" combine to form an appropriate idiom.

Explanation for Incorrect Answer C :  There is no error at (C). The word "but" is appropriately used to link two contrasting adjectives describing the dancer.

Explanation for Incorrect Answer D :  There is no error at (D). The participle "posing" combines with the verb "was" to form an appropriate verb form.

21

1576 Santa Elena, now an in South Carolina, was the

capital of Spanish Florida; , it an English settlement by

1735. 

ANSWERS AND EXPLANATIONS Corrected Sentence: From 1566 until 1576, Santa Elena, now an excavation site in South Carolina, was the capital of Spanish Florida; however, it had become an English settlement by 1735.

Explanation for Correct Answer D :  The error in this sentence occurs at (D), where the present perfect verb phrase "has become" is used instead of the pluperfect that is needed to describe an action that had taken place "by 1735."

Explanation for Incorrect Answer A :  There is no error at (A). The words "From" and "until" are properly used to describe the decade during which Santa Elena was the capital of Spanish Florida.

Explanation for Incorrect Answer B :  There is no error at (B). The compound noun "excavation site" is properly used in the phrase enclosed by commas that describes what Santa Elena is now.

Explanation for Incorrect Answer C :  There is no error at (C). The conjunctive adverb "however" is properly used to introduce a clause.

Explanation for Incorrect Answer E :  There is an error in the sentence.

22

From 1566 until excavation site

however has become

No error

页码,12/21The Official SAT Online Course

2006-11-12file://E:\新建文件夹\b6.htm

UnRe

gistered

Page 64: SAT Online Course Test 2 Explanations

严禁用于商业用途!

the attorney’s moving plea, the judge the juvenile offender

probation for an period. 

ANSWERS AND EXPLANATIONS Corrected Sentence: Despite the attorney's moving plea, the judge placed the juvenile offender on probation for an indefinite period.

Explanation for Correct Answer D :  The error in this sentence occurs at (D), where there is improper diction. The adjective "indecisive" is used where "indefinite" is needed.

Explanation for Incorrect Answer A :  There is no error at (A). The preposition "Despite" is properly used to introduce a prepositional phrase.

Explanation for Incorrect Answer B :  There is no error at (B). The verb "placed," which can be singular or plural, agrees with its singular subject, "judge."

Explanation for Incorrect Answer C :  There is no error at (C). The preposition "on" combines with the noun "probation" to form an appropriate prepositional phrase.

Explanation for Incorrect Answer E :  There is an error in the sentence.

23

a art form of the Americas, the art world of the

1920’s to the three popular Mexican mural artists

ANSWERS AND EXPLANATIONS Corrected Sentence: 

Explanation for Correct Answer E :  There is no error in this sentence.

Explanation for Incorrect Answer A :  There is no error at (A). The participle "yearning" correctly introduces a phrase describing "the art world," and the preposition "for" functions properly within that phrase to link the verbal "yearning" with the noun "form."

Explanation for Incorrect Answer B :  There is no error at (B). The adverb "truly" is used correctly to modify the adjective "representative," and "representative" appropriately describes the noun "form."

Explanation for Incorrect Answer C :  There is no error at (C). The verb in past tense, "looked," correctly describes a completed action, and the adverb "hopefully" (telling how) correctly modifies the verb.

Despite placed on

indecisive No error

Yearning for truly representative

looked hopefully

of the day No error

页码,13/21The Official SAT Online Course

2006-11-12file://E:\新建文件夹\b6.htm

UnRe

gistered

Page 65: SAT Online Course Test 2 Explanations

严禁用于商业用途!

Explanation for Incorrect Answer D :  There is no error at (D). The prepositional phrase "of the day" serves appropriately as an adjective describing the noun "artists."

24

There has always been a friction between   have

opposing political views we are very vocal. 

ANSWERS AND EXPLANATIONS Corrected Sentence: There has always been a great deal of friction between Joan and me because we have opposing political views about which we are very vocal.

Explanation for Correct Answer B :  The error in this sentence occurs at (B), where there is an improper pronoun case. The nominative case of the first-person pronoun, "I," is used where the objective case, "me," is needed.

Explanation for Incorrect Answer A :  There is no error at (A). The phrase "a great deal of" is an appropriate idiom.

Explanation for Incorrect Answer C :  There is no error at (C). The subordinating conjunction "because" is appropriately used to introduce a dependent adverbial clause; the nominative case of the pronoun "we" is appropriately used, since it is the subject of the clause.

Explanation for Incorrect Answer D :  There is no error at (D). The preposition "about" is appropriately used to introduce the relative pronoun "which," which refers appropriately to the plural noun "views."

Explanation for Incorrect Answer E :  There is an error in the sentence.

25

M. R. Harrington, an archaeologist from the Museum of the American Indian,

the ruins the Pueblo Grande de Nevada, he unearthed

artifacts a 500-year occupation by indigenous peoples. 

ANSWERS AND EXPLANATIONS Corrected Sentence: 

Explanation for Correct Answer E :  There is no error in this sentence.

Explanation for Incorrect Answer A :  There is no error at (A). The subordinating conjunction "When" properly begins the dependent adverbial clause "When . . . Nevada."

Explanation for Incorrect Answer B :  There is no error at (B). The verb "began" properly describes an action in the past

great deal of Joan and I because we

about which No error

When

began to excavate he named

indicating No error

页码,14/21The Official SAT Online Course

2006-11-12file://E:\新建文件夹\b6.htm

UnRe

gistered

Page 66: SAT Online Course Test 2 Explanations

严禁用于商业用途!

and joins with the infinitive phrase "to excavate" to form an appropriate idiom.

Explanation for Incorrect Answer C :  There is no error at (C). The singular pronoun "he," referring to "M.R. Harrington," agrees with the singular verb "named."

Explanation for Incorrect Answer D :  There is no error at (D). The participle "indicating" properly modifies the noun "artifacts."

26

The supervisor cited three workers, each of   win a prize

cost-effective changes the factory. 

ANSWERS AND EXPLANATIONS Corrected Sentence: The supervisor cited three workers, each of whom is likely to win a prize for having suggested cost-effective changes at the factory.

Explanation for Correct Answer A :  The error in this sentence occurs at (A), where there is an improper use of pronoun. In the phrase "each of which," the pronoun "which" incorrectly refers to a person (one of the "three workers") and should instead be "whom."

Explanation for Incorrect Answer B :  There is no error at (B). The singular verb "is" agrees with the singular subject "each" and combines with the phrase "likely to" to form an appropriate idiom.

Explanation for Incorrect Answer C :  There is no error at (C). The preposition "for" and the phrase "having suggested" together form an appropriate idiom.

Explanation for Incorrect Answer D :  There is no error at (D). The preposition "at" properly introduces the adjective phrase modifying "changes."

Explanation for Incorrect Answer E :  There is an error in the sentence.

27

Freedom of action and expression at the foundation our system of

government but also of our human relations levels of

society. 

ANSWERS AND EXPLANATIONS Corrected Sentence: Freedom of action and expression is at the foundation not only of our system of government but also of our expectations concerning human relations at all levels of society.

Explanation for Correct Answer A :  The error in this sentence occurs at (A), where there is subject-verb disagreement. The plural verb "are" does not agree with the singular subject "freedom."

which is likely to

for having suggested at No error

are not only of

expectations concerning at all

No error

页码,15/21The Official SAT Online Course

2006-11-12file://E:\新建文件夹\b6.htm

UnRe

gistered

Page 67: SAT Online Course Test 2 Explanations

严禁用于商业用途!

Explanation for Incorrect Answer B :  There is no error at (B). The phrase "not only" is joined with the preposition "of" to form an appropriate idiom that is paralleled later in the sentence with "but also of."

Explanation for Incorrect Answer C :  There is no error at (C). The noun "expectations" and the participle "concerning" combine to form an appropriate idiom.

Explanation for Incorrect Answer D :  There is no error at (D). The preposition "at" and the adjective "all" together form an appropriate idiom.

Explanation for Incorrect Answer E :  There is an error in the sentence.

28

While both disaster rescue workers and news reporters physical danger,

can usually control exposure to risk, rescue workers

often cannot. 

ANSWERS AND EXPLANATIONS Corrected Sentence: While both disaster rescue workers and news reporters may face physical danger, the latter can usually control their exposure to risk, whereas rescue workers often cannot.

Explanation for Correct Answer C :  The error in this sentence occurs at (C), where there is noun-pronoun disagreement. Because the pronouns "his" and "her" are both singular, neither properly refers to the plural noun "the latter" ("news reporters").

Explanation for Incorrect Answer A :  There is no error at (A). The plural verb "may face" agrees with its plural compound subjects "workers" and "reporters."

Explanation for Incorrect Answer B :  There is no error at (B). The phrase "the latter" properly refers to "news reporters."

Explanation for Incorrect Answer D :  There is no error at (D). The subordinating conjunction "whereas" properly begins the dependent adverbial clause "whereas . . . cannot."

Explanation for Incorrect Answer E :  There is an error in the sentence.

29

The refusal of the management     family leave an

uproar among employees.  

ANSWERS AND EXPLANATIONS

may face

the latter his or her whereas

No error

to revise their policy on caused

No error

页码,16/21The Official SAT Online Course

2006-11-12file://E:\新建文件夹\b6.htm

UnRe

gistered

Page 68: SAT Online Course Test 2 Explanations

严禁用于商业用途!

Corrected Sentence: The refusal of management to revise its policy on family leave caused an uproar among employees.

Explanation for Correct Answer B :  The error in this sentence occurs at (B), where an improper pronoun is used. The plural pronoun "their" refers incorrectly to the singular noun "management."

Explanation for Incorrect Answer A :  There is no error at (A). The infinitive phrase "to revise" operates as an nominative adjective describing "refusal."

Explanation for Incorrect Answer C :  There is no error at (C). The noun "policy" and the preposition "on" combine to form an appropriate idiom.

Explanation for Incorrect Answer D :  There is no error at (D). The verb "caused" idiomatically introduces the effect of the management's action.

Explanation for Incorrect Answer E :  There is an error in the sentence.

(1) Some of the world’s greatest scientists have been women, and most people still tend to think of science as a “man’s game.” (2) There are probably many reasons that more men than women had fame as scientists. (3) Unequal access to educational opportunities is certainly one. 

(4) But sometimes the reason is plain old-fashioned dishonesty. (5) James Watson, Francis Crick, and Maurice Wilkins were awarded the Nobel Prize for the discovery by them of the double helix structure of the DNA molecule. (6) The discovery is seen by most as one of the greatest contributions to the modern history of biology. 

(7) One of the most important pieces of evidence used by Watson and Crick to figure out this structure was an x-ray diffraction photograph that had been taken by a woman, Rosalind Franklin. (8) Scientists often build on the work of other scientists, but they usually do so openly. (9)  Franklin’s photograph was secretly shown to Watson by her colleague Maurice Wilkins. (10) Who never told her what he had done. (11) And then Watson, Crick, and Wilkins gave Nobel Prize lectures that contained 98 references to the work of other scientists, not citing a single one of Franklin’s papers.  (12) Of them only Wilkins in his speech making even a casual reference to her when he said she made some “very valuable contributions to the x-ray analysis.” 

30

In context, which of the following is the best change to make to sentence 1 ?

ANSWERS AND EXPLANATIONS Explanation for Correct Answer D :  Choice (D) is correct. The addition of the adverb “yet” establishes the necessary contrast between clauses.

Explanation for Incorrect Answer A :  Choice (A) is unsatisfactory because the first sentence of the passage does not require the addition of such a transitional phrase.

(A) Insert “As one can see” at the beginning.

(B) Insert “In the field of genetics” at the beginning.

(C) Insert “of course” after “and”.

(D) Insert “yet” after “and”.

(E) Delete the quotation marks.

页码,17/21The Official SAT Online Course

2006-11-12file://E:\新建文件夹\b6.htm

UnRe

gistered

Page 69: SAT Online Course Test 2 Explanations

严禁用于商业用途!

Explanation for Incorrect Answer B :  Choice (B) is unsatisfactory because it changes the meaning of the original sentence, narrowing the role of women from all sciences to the field of genetics.

Explanation for Incorrect Answer C :  Choice (C) is unsatisfactory because it does not provide the necessary contrast between clauses and because it adds an unnecessary phrase.

Explanation for Incorrect Answer E :  Choice (E) is unsatisfactory because the phrase “a man’s game” is colloquial speech, quoted by the writer.

31 What is the best way to deal with sentence 2?

ANSWERS AND EXPLANATIONS Explanation for Correct Answer E :  Choice (E) is correct. It replaces the simple past tense verb, “had,” with the completed past tense, “have achieved.”

Explanation for Incorrect Answer A :  Choice (A) is unsatisfactory. Sentence 2 begins the discussion of a series of reasons (continued in sentences 3 and 4) that women have not achieved as much fame in science as men have.

Explanation for Incorrect Answer B :  Choice (B) is unsatisfactory because sentence 1 presents a set of circumstances, and sentences 2, 3, and 4 offer possible reasons for those circumstances.

Explanation for Incorrect Answer C :  Choice (C) is unsatisfactory because “Definitely” contradicts the word “probably” later in the sentence.

Explanation for Incorrect Answer D :  Choice (D) is unsatisfactory because it introduces a colloquial phrase more general than the original adjective.

(A) Omit it.

(B) Switch it with sentence 1.

(C) Insert “Definitely” at the beginning.

(D) Change “many” to “lots of”.

(E) Change “had” to “have achieved”.

32 Which of the following sentences is best inserted after sentence 3?

ANSWERS AND EXPLANATIONS Explanation for Correct Answer B :  Choice (B) is correct. The sentence extends the discussion of “educational opportunities” in sentence 2 and provides an additional reason for men’s greater

(A) They think of science as a field in which men have been traditionally encouraged to participate.

(B) The failure of the educational system to nurture young girls’ interest in science is certainly another.

(C) Some of the best-known names in science are those of men such as Galileo and Einstein.

(D) The girls in my school are not given the same opportunities to study scientific subjects as the boys are.

(E) Yet Rosalind Franklin was a woman who made a number of extremely significant contributions to modern science.

页码,18/21The Official SAT Online Course

2006-11-12file://E:\新建文件夹\b6.htm

UnRe

gistered

Page 70: SAT Online Course Test 2 Explanations

严禁用于商业用途!

fame in science.

Explanation for Incorrect Answer A :  Choice (A) is unsatisfactory because the pronoun “They” is ambiguous and because the sentence returns to the concerns of sentence 1 rather than building on sentence 3.

Explanation for Incorrect Answer C :  Choice (C) is unsatisfactory because it makes an illogical leap, moving from a mention of “access to educational opportunities” in sentence 3 to a sudden presentation of the names of great scientists.

Explanation for Incorrect Answer D :  Choice (D) is unsatisfactory because it moves to a discussion of present conditions in school instead of providing background information about conditions hindering female scientists from achieving fame.

Explanation for Incorrect Answer E :  Choice (E) is unsatisfactory because it moves illogically to defend the contributions of Rosalind Franklin, though the writer has not yet introduced her.

33 In context, which of the following is the best way to express the underlined portion of sentence 5 (reproduced below) ? James Watson, Francis Crick, and Maurice Wilkins were awarded the Nobel Prize for the discovery by them of the double helix structure of the DNA molecule.

ANSWERS AND EXPLANATIONS Explanation for Correct Answer D :  Choice (D) is correct. It replaces the wordy phrase "the discovery by them" with the more concise "their discovery."

Explanation for Incorrect Answer A :  Choice (A) is unsatisfactory because the phrase "by them" is repetitive and unnecessary.

Explanation for Incorrect Answer B :  Choice (B) is unsatisfactory because the three scientists have not been named up to this point in the essay; hence, there is no one to whom "They" could logically refer.

Explanation for Incorrect Answer C :  Choice (C) is unsatisfactory. Since Watson, Crick, and Wilkins have not been named previously, their first names should be given; also, the phrase "by them" is repetitive.

Explanation for Incorrect Answer E :  Choice (E) is unsatisfactory because the word "But" indicates that what follows will contrast with what came before. Instead, sentence 5 introduces an example illustrating the idea expressed in sentence 4.

(A) (As it is now)

(B) They were awarded the Nobel Prize for the discovery

(C) Watson, Crick, and Wilkins were awarded the Nobel Prize for the discovery by them

(D) James Watson, Francis Crick, and Maurice Wilkins were awarded the Nobel Prize for their discovery

(E) But the Nobel Prize was awarded to Watson, Crick, and Wilkins for the discovery

34In context, which of the following is the best version of sentences 9 and 10

页码,19/21The Official SAT Online Course

2006-11-12file://E:\新建文件夹\b6.htm

UnRe

gistered

Page 71: SAT Online Course Test 2 Explanations

严禁用于商业用途!

(reproduced below) ? Franklin’s photograph was secretly shown to Watson by her colleague Maurice Wilkins. Who never told her what he had done.

ANSWERS AND EXPLANATIONS Explanation for Correct Answer E :  Choice (E) is correct. The phrase "In this case" alerts the reader that what follows may be an exception to the general rule that has just been described in sentence 8.

Explanation for Incorrect Answer A :  Choice (A) is unsatisfactory because sentence 10 is a fragment: it is grammatically incomplete because it is a subordinate clause.

Explanation for Incorrect Answer B :  Choice (B) is unsatisfactory because the word "Similarly" indicates that what follows will agree with what has just been said in sentence 8; in fact, the writer is describing an exception to the rule stated in sentence 8.

Explanation for Incorrect Answer C :  Choice (C) is unsatisfactory because it illogically places the emphasis on Franklin's misfortune rather than on Wilkins' dishonesty, which is the overall subject of the passage.

Explanation for Incorrect Answer D :  Choice (D) is unsatisfactory because the sequence of events is confusing. It describes Wilkins' failure to tell Franklin before mentioning that he showed her photograph to Watson, but it would be more logical to describe showing the photograph first.

(A) (As it is now)

(B) Similarly, her colleague Maurice Wilkins showed Franklin’s photograph to Watson, and he never told her about it.

(C) Unfortunately, she did not know that her colleague Maurice Wilkins had secretly shown this photograph to Watson.

(D) In fact, Franklin’s colleague Maurice Wilkins never told her that he had shown Watson the photograph.

(E) In this case, Franklin’s colleague Maurice Wilkins secretly showed Watson her photograph without telling her.

35 Which of the following is the best way to express the underlined portion of sentence 12 (reprinted below) ? Of them only Wilkins in his speech making even a casual reference to her when he said she made some “very valuable contributions to the x-ray analysis.”

ANSWERS AND EXPLANATIONS Explanation for Correct Answer C :  Choice (C) is correct. The phrase "Of the three" provides a concise link to the previous sentence; also, the sentence has an appropriate main verb, "made."

Explanation for Incorrect Answer A : 

(A) (As it is now)

(B) Having made his speech, only Wilkins had made even

(C) Of the three, only Wilkins made even

(D) Only his speech contained even

(E) In the same manner, Wilkins made

页码,20/21The Official SAT Online Course

2006-11-12file://E:\新建文件夹\b6.htm

UnRe

gistered

Page 72: SAT Online Course Test 2 Explanations

严禁用于商业用途!

Choice (A) is unsatisfactory because it is a fragment: the sentence has no main verb. Also, the pronoun "them" is ambiguous; it is not clear to which of the many plural nouns in the previous sentence it refers.

Explanation for Incorrect Answer B :  Choice (B) is unsatisfactory because the phrase "Having made his speech" unnecessarily repeats information from sentence 11; also, the pluperfect tense of the verb, "had made," would be more logical if it were changed to the past tense, "made."

Explanation for Incorrect Answer D :  Choice (D) is unsatisfactory because it is not clear whether the pronoun "his" refers to Watson, Crick, or Wilkins.

Explanation for Incorrect Answer E :  Choice (E) is unsatisfactory because the phrase "In the same manner" is illogical; the sentence is describing something in Wilkins's speech that was different from the other two speeches.

    

Back to Score Report  

Copyright © 2006 The College Board. All rights reserved. Privacy Policy Terms of Use Contact Us

 

页码,21/21The Official SAT Online Course

2006-11-12file://E:\新建文件夹\b6.htm

UnRe

gistered

Page 73: SAT Online Course Test 2 Explanations

严禁用于商业用途!

Help | Profile | My Organizer | My Bookmarks | Logout

Answers and Explanations

Test Sections

Section 1

Section 2

Section 3

Section 4

Section 6

Section 7

Section 8

Section 9

Section 10

Back to Score Report  

View Answers and Explanations     Online - Practice Test #2

1 New data measuring the ------- of land beneath the oceans permit accurate generalizations about the topography of the seafloor.

ANSWERS AND EXPLANATIONS Explanation for Correct Answer B :  Choice (B) is correct. "Contours" are the outlines or shapes of curving or irregular surfaces. If one were to insert this term into the text, the sentence would read "New data measuring the contours of land beneath the oceans permit accurate generalizations about the topography of the seafloor." The missing term has a meaning similar to "topography," or the surface features of a place or region. The "contours of land" are the outlines or shape of the land's surface.

Explanation for Incorrect Answer A :  Choice (A) is incorrect. "Models" are miniature representations of something. If one were to insert this term into the text, the sentence would read "New data measuring the models of land beneath the oceans permit accurate generalizations about the topography of the seafloor." The actual land beneath the oceans, not miniature representations of the land, must be measured in order to make accurate generalizations about the seafloor.

Explanation for Incorrect Answer C :  Choice (C) is incorrect. "Remnants" are remains or things left over. If one were to insert this term into the text, the sentence would read "New data measuring the remnants of land beneath the oceans permit accurate generalizations about the topography of the seafloor." In this context, it does not make sense to speak of remains of land on the ocean floor.

Explanation for Incorrect Answer D :  Choice (D) is incorrect. A "population" is the total number of people or organisms in a given area. If one were to insert this term into the text, the sentence would read "New data measuring the populations of land beneath the oceans permit accurate generalizations about the topography of the seafloor." The land beneath the oceans may have a "population" of organisms, but knowing the number of these organisms would not assist in making generalizations about the seafloor's "topography," or surface features.

Explanation for Incorrect Answer E :  Choice (E) is incorrect. "Pigments" are substances used for coloring. If one were to insert this term into the text, the sentence would read "New data measuring the pigments of land beneath the oceans permit accurate generalizations about the topography of the seafloor." Colors are not among the surface features typically referred to by the term "topography."

(A) models

(B) contours

(C) remnants

(D) population

(E) pigments

2 Excessive secrecy tends to ------- excessive curiosity and thus serves to ------- the very impulses against which it guards.

页码,1/21The Official SAT Online Course

2006-11-12file://E:\新建文件夹\b7.htm

UnRe

gistered

Page 74: SAT Online Course Test 2 Explanations

严禁用于商业用途!

ANSWERS AND EXPLANATIONS Explanation for Correct Answer E :  Choice (E) is correct. "To invite" means to encourage. "To provoke" means to give rise to. If one were to insert these terms into the text, the sentence would read "Excessive secrecy tends to invite excessive curiosity and thus serves to provoke the very impulses against which it guards." This sentence makes sense because excessive secrecy encourages excessive curiosity. Secrecy thus gives rise to "the very impulses" that the overly secretive seek to avoid--namely, people's desires to find out what is going on.

Explanation for Incorrect Answer A :  Choice (A) is incorrect. "To inhibit" means to prevent. "To protect" means to guard. If one were to insert these terms into the text, the sentence would read "Excessive secrecy tends to inhibit excessive curiosity and thus serves to protect the very impulses against which it guards." Excessive secrecy does not prevent excessive curiosity. In fact, it does exactly the opposite.

Explanation for Incorrect Answer B :  Choice (B) is incorrect. "To disguise" means to conceal an identity. "To supplant" means to oust and take the place of. If one were to insert these terms into the text, the sentence would read "Excessive secrecy tends to disguise excessive curiosity and thus serves to supplant the very impulses against which it guards." Excessive secrecy does not conceal the identity of excessive curiosity. Neither does excessive secrecy oust and take the place of curious interest.

Explanation for Incorrect Answer C :  Choice (C) is incorrect. "To satisfy" means to meet expectations. "To limit" means to restrict. If one were to insert these terms into the text, the sentence would read "Excessive secrecy tends to satisfy excessive curiosity and thus serves to limit the very impulses against which it guards." Excessive secrecy does not meet the expectations of excessive curiosity, nor does it restrict curious interest.

Explanation for Incorrect Answer D :  Choice (D) is incorrect. "To compel" means to force. "To deride" means to mock. If one were to insert these terms into the text, the sentence would read "Excessive secrecy tends to compel excessive curiosity and thus serves to deride the very impulses against which it guards." Excessive secrecy may encourage excessive curiosity, but it does not force it.

(A) inhibit . . protect

(B) disguise . . supplant

(C) satisfy . . limit

(D) compel . . deride

(E) invite . . provoke

3In frigid regions a layer of permafrost under the soil surface prevents water from sinking deep into the soil, and so the water ------- the land, helping to create bog and ------- conditions.

ANSWERS AND EXPLANATIONS Explanation for Correct Answer E :  Choice (E) is correct. "Floods" means covers with water. A "swamp" is a water-soaked patch of land. If one were to insert these terms into the text, the sentence would read "In frigid regions a layer of permafrost under the soil surface prevents water from sinking deep into the soil, and so the water floods the land, helping to

(A) freezes . . tropical

(B) parches . . marsh

(C) inundates . . desert

(D) aerates . . jungle

(E) floods . . swamp

页码,2/21The Official SAT Online Course

2006-11-12file://E:\新建文件夹\b7.htm

UnRe

gistered

Page 75: SAT Online Course Test 2 Explanations

严禁用于商业用途!

create bog and swamp conditions." The first missing term describes what water does when it cannot sink into soil, and the second missing term describes a consequence of that situation. A piece of land "floods" when water cannot sink into it, and this creates watery areas, or "swamps."

Explanation for Incorrect Answer A :  Choice (A) is incorrect. "Freezes" means turns to ice. "Tropical" refers to a hot region or hot conditions. If one were to insert these terms into the text, the sentence would read "In frigid regions a layer of permafrost under the soil surface prevents water from sinking deep into the soil, and so the water freezes the land, helping to create bog and tropical conditions." It is possible that water might turn to ice atop the cold region's permafrost, but this would not cause hot, "tropical" conditions.

Explanation for Incorrect Answer B :  Choice (B) is incorrect. "Parches" means dries out. A "marsh" is a patch of watery ground. If one were to insert these terms into the text, the sentence would read "In frigid regions a layer of permafrost under the soil surface prevents water from sinking deep into the soil, and so the water parches the land, helping to create bog and marsh conditions." A piece of land gets very wet when water cannot sink into it. This land could not be called "parched." Furthermore, dried out land could not be considered a "marsh," which is a patch of watery ground.

Explanation for Incorrect Answer C :  Choice (C) is incorrect. "Inundates" means covers with water. A "desert" is an extremely dry region. If one were to insert these terms into the text, the sentence would read "In frigid regions a layer of permafrost under the soil surface prevents water from sinking deep into the soil, and so the water inundates the land, helping to create bog and desert conditions." A piece of land gets very wet when water cannot sink through it. This is the opposite of "desert conditions."

Explanation for Incorrect Answer D :  Choice (D) is incorrect. "Aerates" means adds air to. A "jungle" is an overgrown forest. If one were to insert these terms into the text, the sentence would read "In frigid regions a layer of permafrost under the soil surface prevents water from sinking deep into the soil, and so the water aerates the land, helping to create bog and jungle conditions." A piece of land gets very wet when water cannot sink through it. Water does not "aerate" land, or expose land to air. Furthermore, an overgrown forest would probably not occur in "frigid," or cold, climates.

4Although the bystander’s account of the car accident at first seemed -------, the police officer was surprised, on further investigation, to find that it was -------.

ANSWERS AND EXPLANATIONS Explanation for Correct Answer C :  Choice (C) is correct. "Implausible" means hard to believe. "Correct" means accurate. If one were to insert these words into the text, the sentence would read "Although the bystander's account of the car accident at first seemed implausible, the police officer was surprised, on further investigation, to find that it was correct." The word "although" signals that the first missing term will contrast with the second missing term. The police officer would be surprised if a story that at first seemed hard to believe later proved to be correct.

Explanation for Incorrect Answer A :  Choice (A) is incorrect. "Dubious" means doubtful. "Erroneous" means full of error. If one were to insert these words into the text, the sentence would read "Although the bystander's account of the car accident at first seemed dubious, the police

(A) dubious . . erroneous

(B) incongruous . . inconsistent

(C) implausible . . correct

(D) logical . . pertinent

(E) probable . . coherent

页码,3/21The Official SAT Online Course

2006-11-12file://E:\新建文件夹\b7.htm

UnRe

gistered

Page 76: SAT Online Course Test 2 Explanations

严禁用于商业用途!

officer was surprised, on further investigation, to find that it was erroneous." The officer would hardly have been surprised had a doubtful account turned out to be full of errors. The sentence is illogical because the contrast signaled by the word "although" does not occur.

Explanation for Incorrect Answer B :  Choice (B) is incorrect. "Incongruous" means illogical. "Inconsistent" means contradictory. If one were to insert these words into the text, the sentence would read "Although the bystander's account of the car accident at first seemed incongruous, the police officer was surprised, on further investigation, to find that it was inconsistent." The police officer would not be surprised if a story that at first seemed illogical later proved to be contradictory. The contrast signaled by the word "although" does not occur, and, as a result, the sentence does not make sense.

Explanation for Incorrect Answer D :  Choice (D) is incorrect. "Logical" means rational or making sense. "Pertinent" means relevant. If one were to insert these words into the text, the sentence would read "Although the bystander's account of the car accident at first seemed logical, the police officer was surprised, on further investigation, to find that it was pertinent." The sentence does not make sense because there is no connection between a story being rational and a story being relevant.

Explanation for Incorrect Answer E :  Choice (E) is incorrect. "Probable" means likely. "Coherent" means logical. If one were to insert these words into the text, the sentence would read "Although the bystander's account of the car accident at first seemed probable, the police officer was surprised, on further investigation, to find that it was coherent." The police officer would not be surprised if a story that at first seemed likely later proved to be logical. The sentence is illogical because the contrast signaled by the word "although" does not occur.

5 The legislation facing Congress was so ------- that it threatened to shatter the governing body’s fragile bipartisanship.

ANSWERS AND EXPLANATIONS Explanation for Correct Answer A :  Choice (A) is correct. "Divisive" means creating division. If one were to insert this term into the text, the sentence would read "The legislation facing Congress was so divisive that it threatened to shatter the governing body's fragile bipartisanship." The missing term describes the nature of a piece of legislation that threatened to break up a relationship between two political parties. Therefore, by definition, "divisive legislation" will tend to divide people and disrupt bipartisan political action.

Explanation for Incorrect Answer B :  Choice (B) is incorrect. "Transparent" means easily understood. If one were to insert this term into the text, the sentence would read "The legislation facing Congress was so transparent that it threatened to shatter the governing body's fragile bipartisanship." Legislation that can be easily understood does not necessarily threaten a relationship between political parties.

Explanation for Incorrect Answer C :  Choice (C) is incorrect. "Concordant" means harmonious or agreeing. If one were to insert this term into the text, the sentence would read "The legislation facing Congress was so concordant that it threatened to shatter the governing body's fragile bipartisanship." Legislation that is harmonious would not suggest a threat to a relationship between political parties.

Explanation for Incorrect Answer D :  Choice (D) is incorrect. "Repetitive" means repeating unnecessarily. If one were to insert this term into the text, the sentence would read "The legislation facing Congress was so repetitive that it threatened to shatter the governing body's fragile

(A) divisive

(B) transparent

(C) concordant

(D) repetitive

(E) rhetorical

页码,4/21The Official SAT Online Course

2006-11-12file://E:\新建文件夹\b7.htm

UnRe

gistered

Page 77: SAT Online Course Test 2 Explanations

严禁用于商业用途!

bipartisanship." "Repetitive" legislation would not necessarily affect the governing body's bipartisanship.

Explanation for Incorrect Answer E :  Choice (E) is incorrect. "Rhetorical" refers to saying or asking something for persuasive effect. If one were to insert this term into the text, the sentence would read "The legislation facing Congress was so rhetorical that it threatened to shatter the governing body's fragile bipartisanship." Legislation that states something for effect would not necessarily suggest a threat to a relationship between political parties.

6 In All God’s Children Need Traveling Shoes, author Maya Angelou uses -------, brief descriptive sketches, to provide ------- view of Ghana that clearly details the land and its people.

ANSWERS AND EXPLANATIONS Explanation for Correct Answer C :  Choice (C) is correct. "Vignettes" are short, descriptive literary sketches. "Vivid" means lively or intensely clear. If one were to insert these terms into the text, the sentence would read "In All God's Children Need Traveling Shoes, author Maya Angelou uses vignettes, brief descriptive sketches, to provide a vivid view of Ghana that clearly details the land and its people." This sentence makes sense because "vignettes" are brief descriptive sketches, and "vivid" sketches clearly describe a subject.

Explanation for Incorrect Answer A :  Choice (A) is incorrect. "Missives" are letters. "Illusory" means deceptive. If one were to insert these terms into the text, the sentence would read "In All God's Children Need Traveling Shoes, author Maya Angelou uses missives, brief descriptive sketches, to provide an illusory view of Ghana that clearly details the land and its people." A letter is not a brief descriptive sketch. Also, a deceptive view of Ghana would not clearly describe the land and its people.

Explanation for Incorrect Answer B :  Choice (B) is incorrect. "Themes" are main ideas. "Thorough" means complete. If one were to insert these terms into the text, the sentence would read "In All God's Children Need Traveling Shoes, author Maya Angelou uses themes, brief descriptive sketches, to provide a thorough view of Ghana that clearly details the land and its people." Since all books have main ideas, one does not need to specify that author Maya Angelou uses them.

Explanation for Incorrect Answer D :  Choice (D) is incorrect. "Treatises" are typically lengthy, complex, and often analytical pieces of writing. "Authentic" means real or factually correct. If one were to insert these terms into the text, the sentence would read "In All God's Children Need Traveling Shoes, author Maya Angelou uses treatises, brief descriptive sketches, to provide an authentic view of Ghana that clearly details the land and its people." A "treatise" is not a brief sketch.

Explanation for Incorrect Answer E :  Choice (E) is incorrect. "Abstracts" are summaries of main points. "Ambiguous" means uncertain or indistinct. If one were to insert these terms into the text, the sentence would read "In All God's Children Need Traveling Shoes, author Maya Angelou uses abstracts, brief descriptive sketches, to provide an ambiguous view of Ghana that clearly details the land and its people." An "ambiguous" view of Ghana would not offer clear details about the country's land and people.

(A) missives. . an illusory

(B) themes. . a thorough

(C) vignettes. . a vivid

(D) treatises. . an authentic

(E) abstracts. . an ambiguous

7Because an older horse is more ------- than a younger one, it is safer for a novice

页码,5/21The Official SAT Online Course

2006-11-12file://E:\新建文件夹\b7.htm

UnRe

gistered

Page 78: SAT Online Course Test 2 Explanations

严禁用于商业用途!

rider.

ANSWERS AND EXPLANATIONS Explanation for Correct Answer D :  Choice (D) is correct. "Tractable" means easily handled or controlled. If one were to insert this term into the text, the sentence would read "Because an older horse is more tractable than a younger one, it is safer for a novice rider." The missing term refers to the quality of a horse that will make it safe for inexperienced riders. Most likely, an older horse is easier to handle or control than a younger horse and is therefore a good choice for a "novice," or inexperienced, rider.

Explanation for Incorrect Answer A :  Choice (A) is incorrect. "Frolicsome" means playful. If one were to insert this term into the text, the sentence would read "Because an older horse is more frolicsome than a younger one, it is safer for a novice rider." An older horse is usually less, not more, playful than a younger horse. In addition, the energy of a playful horse might be unsafe for a "novice," or inexperienced, rider.

Explanation for Incorrect Answer B :  Choice (B) is incorrect. "Cantankerous" means difficult to deal with. If one were to insert this term into the text, the sentence would read "Because an older horse is more cantankerous than a younger one, it is safer for a novice rider." A horse that is difficult to deal with might be dangerous for a "novice," or inexperienced, rider.

Explanation for Incorrect Answer C :  Choice (C) is incorrect. "Gargantuan" means extremely large. If one were to insert this term into the text, the sentence would read "Because an older horse is more gargantuan than a younger one, it is safer for a novice rider." This would not make sense because the size of a horse is not always correlated with its age. Furthermore, an extremely large horse would not necessarily be safer than a smaller horse.

Explanation for Incorrect Answer E :  Choice (E) is incorrect. "Precipitate" means being hasty and impulsive. If one were to insert this term into the text, the sentence would read "Because an older horse is more precipitate than a younger one, it is safer for a novice rider." A horse that is impulsive would be extremely unsafe for a "novice," or inexperienced, rider.

(A) frolicsome

(B) cantankerous

(C) gargantuan

(D) tractable

(E) precipitate

8The library’s collection is a ------- of Asian American historical documents, including rare materials about race relations.

ANSWERS AND EXPLANATIONS Explanation for Correct Answer D :  Choice (D) is correct. A "trove" is a valuable collection. If one were to insert this term into the text, the sentence would read "The library's collection is a trove of Asian American historical documents, including rare materials about race relations." The missing term is characterized by the phrase that follows the comma. A library collection containing rare materials is a valuable collection.

(A) summary

(B) fabrication

(C) consensus

(D) trove

(E) replication

页码,6/21The Official SAT Online Course

2006-11-12file://E:\新建文件夹\b7.htm

UnRe

gistered

Page 79: SAT Online Course Test 2 Explanations

严禁用于商业用途!

Explanation for Incorrect Answer A :  Choice (A) is incorrect. A "summary" is a review of the main points of something. If one were to insert this term into the text, the sentence would read "The library's collection is a summary of Asian American historical documents, including rare materials about race relations." A library collection, which consists of many diverse documents, is not a "summary."

Explanation for Incorrect Answer B :  Choice (B) is incorrect. A "fabrication" is something made up. If one were to insert this term into the text, the sentence would read "The library's collection is a fabrication of Asian American historical documents, including rare materials about race relations." A library collection includes real historical documents and thus is not a "fabrication."

Explanation for Incorrect Answer C :  Choice (C) is incorrect. "Consensus" mean general agreement. If one were to insert this term into the text, the sentence would read "The library's collection is a consensus of Asian American historical documents, including rare materials about race relations." The term "consensus" does not make sense in this context because a library collection cannot be described as a "general agreement."

Explanation for Incorrect Answer E :  Choice (E) is incorrect. A "replication" is a copy. If one were to insert this term into the text, the sentence would read "The library's collection is a replication of Asian American historical documents, including rare materials about race relations." These historical documents would be originals, not copies. A "replication" is a copy of an original, not a collection of rare historical documents.

      There you are, driving along the freeway, nobody in

 sight, just you, daydreaming, when RIGHT BEHIND YOU

 there’s a blast of sound, as of some eighteen-wheeler out

Lineof control, howling in rage . . . . WHERE IS IT? Without

5thinking, you see all directions at once: the empty road

 ahead and behind, the lack of vehicles on all sides. . . .

 Slowly, you come to understand that the sound is not

 from the road but from a highballing freight train travelling

  parallel to the freeway.

10     How long to go from full alert to anticlimax? Less than a

 minute. It was a chemical reaction, pure and simple, and it

  crippled time in your mind.

9

The capital letters in lines 2 and 4 primarily serve to evoke a sense of

(A) anger

(B) alarm

(C) impatience

(D) disbelief

(E) embarrassment

页码,7/21The Official SAT Online Course

2006-11-12file://E:\新建文件夹\b7.htm

UnRe

gistered

Page 80: SAT Online Course Test 2 Explanations

严禁用于商业用途!

ANSWERS AND EXPLANATIONS Explanation for Correct Answer B :  Choice (B) is correct. The capitalized words are intended to suggest the surprising suddenness of the noise and convey the driver's sense of urgency in finding where the noise is coming from. The driver is clearly startled and "alarmed" by the blast of sound.

Explanation for Incorrect Answer A :  Choice (A) is incorrect. The passage does not mention the driver showing anger. Rather the capitalized letters indicate the driver's alarm, or concern, over the blast of sound.

Explanation for Incorrect Answer C :  Choice (C) is incorrect. The capitalized words relate to the driver's experience of a loud, unknown sound. The driver is probably impatient to find where the sound is coming from, but "impatience" is not a strong enough feeling to be expressed in capital letters.

Explanation for Incorrect Answer D :  Choice (D) is incorrect. There is no question about the existence of the blast of sound, so disbelief cannot apply.

Explanation for Incorrect Answer E :  Choice (E) is incorrect. The driver is not embarrassed by the sound, but startled and worried.

10

If the passage were to continue, the next paragraph would most likely discuss

ANSWERS AND EXPLANATIONS Explanation for Correct Answer D :  Choice (D) is correct. The last sentence of the passage is "it was a chemical reaction, pure and simple, and it crippled time in your mind" (lines 11-12). The logical next step for the passage would be to provide more detail about this chemical reaction.

Explanation for Incorrect Answer A :  Choice (A) is incorrect. As is suggested by the last two sentences, the subject of the passage is the nature of the driver's reaction. It is not trains or other loud vehicles.

Explanation for Incorrect Answer B :  Choice (B) is incorrect. The passage does not discuss the repeated honking of horns.

Explanation for Incorrect Answer C :  Choice (C) is incorrect. Where the driver was going is irrelevant, and the passage does not indicate that the driver was in a hurry.

Explanation for Incorrect Answer E :  Choice (E) is incorrect. Since the "chemical reaction" does not make distinctions between types of noises, it is unlikely that the passage would go on to discuss those distinctions.

(A) other qualities of freight trains that often cause anxiety in humans

(B) the chemical reaction that occurs when drivers repeatedly honk their horns in traffic

(C) the destination of the driver and the reason for the driver’s haste

(D) the nature of the chemical reaction that occurs when people become scared

(E) important differences between the effects of truck horns and train whistles on humans

页码,8/21The Official SAT Online Course

2006-11-12file://E:\新建文件夹\b7.htm

UnRe

gistered

Page 81: SAT Online Course Test 2 Explanations

严禁用于商业用途!

      When Dr. Mae C. Jemison blasted into orbit aboard the

 space shuttle Endeavour on September 12, 1992, she also

 blasted into history as the first woman of color to go into

Linespace. A chemical engineer, scientist, physician, teacher,

5and astronaut, Jemison has been undaunted by a lack of

 role models or by roadblocks to women and minority

 people. “I had to learn very early not to limit myself

 due to others’ limited imagination,” says Dr. Jemison.

 An advocate for science and technology, Jemison main-

10tains, “we need to change the image of who does science.

 That’s important not only for folks who want to go into

  science, but for the folks who fund science.”

11 The primary purpose of the passage is to

ANSWERS AND EXPLANATIONS Explanation for Correct Answer D :  Choice (D) is correct. The passage's first sentence explains Jemison's primary achievement: she "blasted into history as the first woman of color to go into space" (lines 3-4). The remainder of the passage sums up Jemison's background and her goals for science.

Explanation for Incorrect Answer A :  Choice (A) is incorrect. This passage describes the achievements and goals of a particular individual who was undaunted by the obstacles before her. The primary purpose of the passage is not to note the "obstacles facing women in science."

Explanation for Incorrect Answer B :  Choice (B) is incorrect. Although the passage does indicate that Jemison is an advocate for science and technology, this is not the passage's central focus. In addition, the passage does not explain how she "fosters," or encourages, interest in science.

Explanation for Incorrect Answer C :  Choice (C) is incorrect. The idea of fame is not discussed in this passage.

Explanation for Incorrect Answer E :  Choice (E) is incorrect. The passage does not mention how Jemison first became

(A) note obstacles facing women in science

(B) explain how Jemison fosters interest in science

(C) discuss how Jemison rose to fame

(D) provide a sketch of Jemison and her goals

(E) describe Jemison’s introduction to science

页码,9/21The Official SAT Online Course

2006-11-12file://E:\新建文件夹\b7.htm

UnRe

gistered

Page 82: SAT Online Course Test 2 Explanations

严禁用于商业用途!

involved in science.

12 In lines 9-12, Jemison is represented as

ANSWERS AND EXPLANATIONS Explanation for Correct Answer E :  Choice (E) is correct. The passage calls Jemison an "advocate for science and technology" and quotes her as saying, "we need to change the image of who does science" (lines 9-10). These lines indicate that Jemison would like to "reform," or change, common perceptions about who can or should participate in science.

Explanation for Incorrect Answer A :  Choice (A) is incorrect. Jemison is concerned not with politics but with science.

Explanation for Incorrect Answer B :  Choice (B) is incorrect. Jemison shows no interest in "arbitrating," or refereeing, an argument. Rather, she wants to expand the public's understanding of science and technology, and change common perceptions people have of scientists.

Explanation for Incorrect Answer C :  Choice (C) is incorrect. Jemison would no doubt make a fine mentor, but the focus of this passage is on her and her ideas about science.

Explanation for Incorrect Answer D :  Choice (D) is incorrect. Although Jemison clearly has many gifts, she is not portrayed as having unusual personal magnetism or as being an idealist.

(A) a political ideologue

(B) an arbitrator of disputes

(C) a mentor to women

(D) a charismatic visionary

(E) a champion of reform

Mark Twain is the pseudonym of Samuel Clemens (1835-1910), who is best known for his stories about life along the Mississippi River. He also lectured and read from his writings in many countries around the world. The following passages are adapted from two essays about Clemens published while Clemens was still alive.

Passage 1

      While Mark Twain has declared that humor is a

 “subject which has never had much interest” for him, it is

 as a humorist that the world persists in regarding him. It is

Linecertain that Mark Twain is the greatest genius evolved by

5natural selection out of the ranks of American journalism.

 Crude, rudimentary, and often coarse as much of his writing

 was, it bore upon it the fresh stamp of contemporary

  actuality. American humor, neither

页码,10/21The Official SAT Online Course

2006-11-12file://E:\新建文件夹\b7.htm

UnRe

gistered

Page 83: SAT Online Course Test 2 Explanations

严禁用于商业用途!

unfathomably absurd

 like the Irish, nor sharp and sensible and full of the realities

10of life like the Scottish, is simply the humor of imagination.

 It consists in piling towers on towers and mountains on

 mountains; of heaping a joke up to the stars and extending

  it to the end of the world.

      Humor, it must be remembered, is a function of

15nationality. The same joke, as related by an American,

 a Scotsman, or an Irishman, carries with it a distinctive

 ethnic flavor and individuality of approach. Indeed, it is

 open to question whether most humor does not require

 some specialized knowledge of some particular locality.

20The secret of Mark Twain’s worldwide popularity as a

 humorist is not to be attributed to any tricks of style, to

 any breadth of knowledge, or even to any depth of intel-

 lectuality. His humor has international range because it is

 constructed out of a deep comprehension of human nature

25and a profound sympathy for human relationships and

 human failings; thus, it successfully surmounts the diffi-

 culties of translation into alien tongues. Above all, he has

 sympathized with and admired the citizens of every nation,

 seeking beneath the surface veneer the universal traits of

30that nation’s humanity. It is a matter of fact that he has

 made far more damaging admissions concerning America

 than concerning any other nation. “My secret, if there is a

 secret,” Twain has said, “is to create humor independent of

 local conditions. Through studying humanity as exhibited

页码,11/21The Official SAT Online Course

2006-11-12file://E:\新建文件夹\b7.htm

UnRe

gistered

Page 84: SAT Online Course Test 2 Explanations

严禁用于商业用途!

Passage 2

35in the people and localities I best knew and understood, I

 have sought to winnow out the encumbrance of the local.

  Humor, like morality, has its eternal verities.”

      Humor as a solid quality and a lucrative trade is of

 modern invention. The great men who dared to laugh in

40an earlier age than ours laughed in moderation and with

 a wise purpose. Aristophanes, Shakespeare, and Chaucer

 are the true humorists of our world. They did not jest out

 of season. Their humor is precious on account of its parsi-

 mony. They do not at every turn slap their readers on the

45back and assure them that there is nothing congruous in

 the visible world. Of the irreverence that turns whatever

 is beautiful or noble into a stupid jest they knew nothing.

 They kept their humor in its proper place; they used it for a

 wise purpose; they did not degrade it to catch an easy round

50of applause; and, fortunately for them, they are today refused

 the august title of humorist, which sits so aptly upon the

  shoulders of Mark Twain.

      The essence of humor is that it should be unexpected.

 The modern humorist is never unexpected. He beats the

55drum from the moment at which he appears upon the stage.

 Mark Twain brings whatever time has honored down to

 the level of a Yankee drummer. He finds every custom

 ridiculous that does not conform with the standard of the

 United States. He holds his sides when he thinks of the

60old masters. Nor does he understand that there are certain

页码,12/21The Official SAT Online Course

2006-11-12file://E:\新建文件夹\b7.htm

UnRe

gistered

Page 85: SAT Online Course Test 2 Explanations

严禁用于商业用途!

 manifestations of genius which should be sacred even for

 the jester. In other words, Mark Twain the humorist is a

 bull in the china shop of ideas. When, as in A Connecticut

 Yankee in King Arthur’s Court, he gave full rein to his

65fancy, he achieved such a masterpiece of vulgarity as the

 world has never seen. His book gives you the same sort

 of impression which you might receive from a beautiful

 picture over which a poisonous slug had crawled. The hint

 of magnificence is there, pitilessly deformed and defaced.

70And it is the more pitiful because he has a talent which

 stands in need of no folly for its embellishment. Had he

 never cut a joke, had he refrained always from grinning at

 grave and beautiful things, how brilliant a fame would have

  been his!

13Which statement best describes how the authors of the two passages differ in their views of Twain’s humor?

ANSWERS AND EXPLANATIONS Explanation for Correct Answer D :  Choice (D) is correct. The author of Passage 1 claims that Twain respects citizens from every nation and describes "the universal traits of ... that nation's humanity." The author of Passage 2 accuses Twain of a lack of discrimination, or good judgment and taste, saying that Twain "brings whatever time has honored down to the level of a Yankee drummer."

Explanation for Incorrect Answer A :  Choice (A) is incorrect. Although the author of Passage 2 does state that Twain thinks the customs of other countries are ridiculous, there is no indication that the author of Passage 1 finds Twain's humor offensive.

Explanation for Incorrect Answer B :  Choice (B) is incorrect. Although the author of Passage 2 finds Twain's humor highly distasteful, the author of Passage 1, far from viewing Twain's humor as mediocre,

(A) The author of Passage 1 criticizes its offensive style, whereas the author of Passage 2 deplores its American bias.

(B) The author of Passage 1 views it as mediocre, whereas the author of Passage 2 views it as intolerable.

(C) The author of Passage 1 concludes that it is trivial, whereas the author of Passage 2 concludes that it is harmless.

(D) The author of Passage 1 praises its universality, whereas the author of Passage 2 disparages its lack of discrimination.

(E) The author of Passage 1 admires its vigor, whereas the author of Passage 2 considers it understated.

页码,13/21The Official SAT Online Course

2006-11-12file://E:\新建文件夹\b7.htm

UnRe

gistered

Page 86: SAT Online Course Test 2 Explanations

严禁用于商业用途!

calls Twain "the greatest genius" that emerged out of American journalism.

Explanation for Incorrect Answer C :  Choice (C) is incorrect. The author of Passage 1 calls Twain a "genius" and mentions his "worldwide popularity," so clearly this author would not describe Twain's humor as trivial. The author of Passage 2 calls Twain "a bull in the china shop of ideas," suggesting that Twain is far from harmless.

Explanation for Incorrect Answer E :  Choice (E) is incorrect. The author of Passage 1 does appear to admire the vigor, or liveliness, of Twain's humor. However, the author of Passage 2 contrasts Twain's humor with the "moderation" of writers like Aristophanes, Shakespeare, and Chaucer, suggesting that Twain's humor is the opposite of understated, or restrained.

14 In line 23, Passage 1, “range” most nearly means

ANSWERS AND EXPLANATIONS Explanation for Correct Answer A :  Choice (A) is correct. The term "range" is used to express the vast amount, or scope, of people throughout the world who appreciate Twain's humor.

Explanation for Incorrect Answer B :  Choice (B) is incorrect. "Distance" gives the sense of how far two points are from each other. The use of the term "range" here is not to describe the distance between Twain and the people his humor touches, but to indicate the vast amount of different people who understand his jokes.

Explanation for Incorrect Answer C :  Choice (C) is incorrect. Although it could make sense to say that Twain's humor "has international variation," the end of the sentence suggests that the humor has a universal quality that does not depend on variations among people.

Explanation for Incorrect Answer D :  Choice (D) is incorrect. The reason given for the humor's "international range" does not suggest that the humor has been ranked, or judged in relation to others.

Explanation for Incorrect Answer E :  Choice (E) is incorrect. Although the author does suggest that Twain's humor could have international value, the term "range" refers to vast amount of people who value Twain's humor, not the value itself.

(A) scope

(B) distance

(C) variation

(D) ranking

(E) value

15 In lines 23-27 (“His humor . . . tongues”), the author of Passage 1 attributes Twain’s international popularity to his

ANSWERS AND EXPLANATIONS Explanation for Correct Answer C : 

(A) knowledge of comedic style

(B) intellectual breadth

(C) understanding of people

(D) mastery of foreign languages

(E) reputation for appealing to ethnic humor

页码,14/21The Official SAT Online Course

2006-11-12file://E:\新建文件夹\b7.htm

UnRe

gistered

Page 87: SAT Online Course Test 2 Explanations

严禁用于商业用途!

Choice (C) is correct. The passage states that Twain's humor is based on "a deep comprehension of human nature," or a profound understanding of what it means to be human.

Explanation for Incorrect Answer A :  Choice (A) is incorrect. According to the sentence in lines 20-23, Twain's popularity, or widespread public acclaim, is not the result of a tricky or adept use of literary style.

Explanation for Incorrect Answer B :  Choice (B) is incorrect. In lines 20 to 21, the author denies that Twain's popularity, or widespread public acclaim, results from "any breadth of knowledge, or . . . depth of intellectuality."

Explanation for Incorrect Answer D :  Choice (D) is incorrect. In lines 26-27, the author claims that Twain's humor can be translated easily into other languages, not that Twain himself was familiar with those languages.

Explanation for Incorrect Answer E :  Choice (E) is incorrect. The author argues that Twain's humor is not ethnic or local, but universal.

16 Which of the following most nearly captures the meaning of “winnow . . . local” (line 36, Passage 1) ?

ANSWERS AND EXPLANATIONS Explanation for Correct Answer B :  Choice (B) is correct. Twain is saying that in order to "create humor independent of local conditions," or to create a more universal humor, he removes the obstacles to universal humor that local ideas present.

Explanation for Incorrect Answer A :  Choice (A) is incorrect. To "winnow out" means to get rid of something, not to mix it with something else.

Explanation for Incorrect Answer C :  Choice (C) is incorrect. Instead of placing the specific, or local, in context, according to the sentence, Twain seeks to eliminate the local altogether.

Explanation for Incorrect Answer D :  Choice (D) is incorrect. Twain is speaking of removing obstacles, not expressing truth.

Explanation for Incorrect Answer E :  Choice (E) is incorrect. By the expression "winnow out," Twain means that he ignores local customs when they hinder understanding universal truths, not that he reaps the benefits of the local customs.

(A) Intermix local details with universal truths

(B) Take out that which prevents one from recognizing the universal

(C) Use the universal as a way to place the specific in context

(D) Devise new ways to express old truths

(E) Reap the benefits of local customs so as to understand the universal

17 Which of the following, if true, would most seriously undermine the main argument presented in Passage 1?

(A) Humor is intrinsically connected to a sense of morality.

页码,15/21The Official SAT Online Course

2006-11-12file://E:\新建文件夹\b7.htm

UnRe

gistered

Page 88: SAT Online Course Test 2 Explanations

严禁用于商业用途!

ANSWERS AND EXPLANATIONS Explanation for Correct Answer B :  Choice (B) is correct. If some cultural differences could not be transcended, or risen above, then the author's argument that looking beyond cultural differences is the basis of Twain's humor would be seriously undermined.

Explanation for Incorrect Answer A :  Choice (A) is incorrect. The discovery of a link between humor and a sense of morality would not necessarily undermine the argument that Twain's humor is based on universal rather than local characteristics.

Explanation for Incorrect Answer C :  Choice (C) is incorrect. The combination of imagination and humor is precisely the way the author defines American humor. If this were proven true, the argument would be strengthened, not undermined.

Explanation for Incorrect Answer D :  Choice (D) is incorrect. The passage quotes Twain as saying that his experience and knowledge of particular localities has enabled him to find universal humor, so the idea that a humorist must have such knowledge would not undermine the author's argument.

Explanation for Incorrect Answer E :  Choice (E) is incorrect. The author quotes Twain as agreeing that "[h]umor, like morality, has its eternal verities," or perpetual truths. If this were proven true, the argument would be strengthened, not undermined.

(B) Certain cultural differences are so powerful that it is impossible for them to be transcended.

(C) Humor is a function of imagination coupled with exaggeration.

(D) In order to be successful as a humorist, one must have specialized knowledge of local customs.

(E) Humor is based on perpetual truths.

18 Twain’s style of humor is criticized in Passage 2 mainly because the author believes that Twain

ANSWERS AND EXPLANATIONS Explanation for Correct Answer A :  Choice (A) is correct. Throughout all of Passage 2, the author argues that, unlike "the true humorists," Aristophanes, Shakespeare, and Chaucer, Twain exhibits no sense of restraint or subtlety in his humor. According to the author, because Twain continually jokes at everything, keeping nothing sacred, his humor lacks the subtle unexpected quality of the masters. The literary masters "laughed in moderation and with a wise purpose," while Twain cannot refrain from "grinning at grave and beautiful things."

Explanation for Incorrect Answer B :  Choice (B) is incorrect. The author of Passage 2, in fact, criticizes Twain for not ignoring local customs. According to the author, Twain mocks any custom that does not conform to the culture of the United States.

Explanation for Incorrect Answer C : 

(A) lacks the subtlety of the literary masters

(B) ignores local customs

(C) confuses the standards of different nations

(D) attempts too hard to appear refined

(E) is less amusing as a public speaker than as a writer

页码,16/21The Official SAT Online Course

2006-11-12file://E:\新建文件夹\b7.htm

UnRe

gistered

Page 89: SAT Online Course Test 2 Explanations

严禁用于商业用途!

Choice (C) is incorrect. The author of Passage 2 accuses Twain of mocking all nations other than the United States. The author does not claim that Twain confuses the standards, rather that he inappropriately thinks that the standards of all nations should conform to those of the United States.

Explanation for Incorrect Answer D :  Choice (D) is incorrect. Far from suggesting that Twain tries to seem refined, Passage 2 implies that Twain's humor "turns whatever is beautiful or noble into a stupid jest."

Explanation for Incorrect Answer E :  Choice (E) is incorrect. There is no indication in the passage that the author distinguishes between Twain's performance as a public speaker and his performance as a writer.

19 Which of the following terms would the author of Passage 2 most likely use to describe Twain?

ANSWERS AND EXPLANATIONS Explanation for Correct Answer E :  Choice (E) is correct. The statement in Passage 2 that Twain "finds every custom ridiculous that does not conform with the standard of the United States" suggests that Twain is a chauvinist, one who shows an extreme preference for a particular group or place.

Explanation for Incorrect Answer A :  Choice (A) is incorrect. The negative view of Twain's humor, for example, calling one of Twain's works "a masterpiece of vulgarity," suggests that the author of Passage 2 would not describe Twain as having the refined manners of an aristocrat.

Explanation for Incorrect Answer B :  Choice (B) is incorrect. There is no evidence in the passage that the author would describe Twain as a reformer, one who wishes to correct whatever is defective, corrupt, or evil.

Explanation for Incorrect Answer C :  Choice (C) is not correct. The passage suggests that rather that being an apologist, one who defends someone or something, Twain uses humor to attack and undermine "whatever is beautiful or noble."

Explanation for Incorrect Answer D :  Choice (D) is incorrect. There is nothing in the passage that indicates whether the author would describe Twain as a dreamer or a visionary, one whose ideas are idealistic or impractical.

(A) Aristocrat

(B) Reformer

(C) Apologist

(D) Visionary

(E) Chauvinist

20

In the last sentence of Passage 2 (lines 71-74), the author indicates that Mark Twain

(A) would have enjoyed fame despite his vulgarity and crudeness

(B) would have been a better writer if he had not attempted humor

(C) would have enjoyed a brilliant career if he had perfected his comedic technique

(D) was an amateur and a dilettante whose interest in humor was superficial

(E) was destined for failure as a result of his insensitivity to his audience

页码,17/21The Official SAT Online Course

2006-11-12file://E:\新建文件夹\b7.htm

UnRe

gistered

Page 90: SAT Online Course Test 2 Explanations

严禁用于商业用途!

ANSWERS AND EXPLANATIONS Explanation for Correct Answer B :  Choice (B) is correct. According to the last sentence in the passage, if Twain had "never cut a joke," that is, if Twain had not tried to be humorous, he would have achieved a "brilliant" fame as a writer.

Explanation for Incorrect Answer A :  Choice (A) is incorrect. Although the author of Passage 2 criticizes Twain for his vulgarity earlier in the passage, the subject of the last sentence in the passage is the effect the author believes that Twain's attempts at humor had on his work.

Explanation for Incorrect Answer C :  Choice (C) is incorrect. The last sentence in the passage indicates that the author believes that Twain would have had to avoid comedy altogether in order to have "brilliant" fame.

Explanation for Incorrect Answer D :  Choice (D) is incorrect. There is no indication in the sentence that the author saw Twain as an amateur or a dilettante, one who has a superficial interest in the arts.

Explanation for Incorrect Answer E :  Choice (E) is incorrect. There is no indication that the author of Passage 2 thinks of Twain as likely to be a failure; nor is there any suggestion that Twain's own audience thinks of him as insensitive.

21Which of the following, if true, would best support the main argument presented in Passage 2 ?

ANSWERS AND EXPLANATIONS Explanation for Correct Answer B :  Choice (B) is correct. The idea that certain subjects are not easily satirized, or held up to ridicule, supports the argument that certain subjects should not be ridiculed. As the author of Passage 2 argues, "there are certain manifestations of genius which should be sacred even for the jester" but that modern humor, like that of Twain, treats every subject with ridicule.

Explanation for Incorrect Answer A :  Choice (A) is incorrect. The idea that any sort of greatness can be subjected to offensive satire is inconsistent with the argument that certain subjects should not be ridiculed.

Explanation for Incorrect Answer C :  Choice (C) is incorrect. Whether intelligent people appreciate certain kinds of humor is irrelevant to the argument that certain subjects should not be ridiculed.

Explanation for Incorrect Answer D :  Choice (D) is incorrect. The reception humorists are likely to receive is irrelevant to the argument that certain subjects should not be ridiculed.

Explanation for Incorrect Answer E :  Choice (E) is incorrect. Although the author of Passage 2 does suggest that contemporary humorists are less insightful than past writers, this idea is irrelevant

(A) Literary greatness can sometimes be the subject of offensive satire.

(B) Certain subjects are not easily satirized.

(C) Intelligent men and women appreciate a good joke at their own expense every now and then.

(D) Humorists are likely to be well received when they undertake to ridicule respected writers of the past.

(E) Contemporary humorists are never as insightful as the great comic writers of the past.

页码,18/21The Official SAT Online Course

2006-11-12file://E:\新建文件夹\b7.htm

UnRe

gistered

Page 91: SAT Online Course Test 2 Explanations

严禁用于商业用途!

to the argument that certain subjects should not be ridiculed.

22 How would the author of Passage 1 most likely respond to the assertion in Passage 2 that Twain ridicules everything “that does not conform with the standard of the United States” (lines 58-59) ?

ANSWERS AND EXPLANATIONS Explanation for Correct Answer A :  Choice (A) is correct. The author of Passage 1 claims that, in addition to showing respect for others by having "sympathized with and admired the citizens of every nation," Twain "has made far more damaging admissions concerning America than concerning any other nation." It is clear that Twain's sympathy and respect for other nations, combined with his willingness to criticize America, is evidence that he reserves, or keeps, his harshest criticism for America.

Explanation for Incorrect Answer B :  Choice (B) is incorrect. The author of Passage 1 does imply that Twain is a great journalist, but this statement is not a response to the statement about Twain's use of ridicule.

Explanation for Incorrect Answer C :  Choice (C) is incorrect. There is no indication in Passage 1 that the author believes that Twain's popularity was due to his being American.

Explanation for Incorrect Answer D :  Choice (D) is incorrect. Unlike the author of Passage 2, the author of Passage 1 suggests that Twain's humor is not uniquely American but is based on "a deep sympathy for human relationships and human failings."

Explanation for Incorrect Answer E :  Choice (E) is incorrect. The idea that Twain is fascinated by American culture is not suggested in Passage 1.

(A) Twain reserves his harshest criticism for America.

(B) Twain is a gifted American journalist.

(C) Twain achieved international success because he is American.

(D) Twain attempts to create a uniquely American style of humor.

(E) Twain is captivated by American culture.

23

Which of the following pairs best represents each author’s attitude toward the term “humorist,” as used in Passage 1 (line 3) and Passage 2 (line 62) ?       Passage 1 / Passage 2

ANSWERS AND EXPLANATIONS Explanation for Correct Answer E :  Choice (E) is correct. In Passage 1, the term "humorist" is applied to Twain by others though Twain himself claims to be unconcerned with humor. In Passage 2, the author portrays Twain the humorist as "a bull in the china shop of ideas," indicating a scornful view of Twain's humor. The respect felt by the author of Passage 1 is reflected in the description of Twain as a "genius." The disdain, or contempt, of the author of Passage 2 can be inferred from the negative remarks made in comparing Twain with the earlier literay masters.

(A) Amusement / Anger

(B) Envy / Mockery

(C) Pride / Embarrassment

(D) Approval / Smugness

(E) Respect / Disdain

页码,19/21The Official SAT Online Course

2006-11-12file://E:\新建文件夹\b7.htm

UnRe

gistered

Page 92: SAT Online Course Test 2 Explanations

严禁用于商业用途!

Explanation for Incorrect Answer A :  Choice (A) is incorrect. There is no evidence in Passage 1 that the author is amused by the term. Although the author of Passage 2 is critical of Twain as a humorist, "anger" would be too strong a word to describe the author's tone.

Explanation for Incorrect Answer B :  Choice (B) is incorrect. Although the tone of the discussion in Passage 2 could be seen as mocking, there is no indication that the author of Passage 1 is envious of Twain or other humorists.

Explanation for Incorrect Answer C :  Choice (C) is incorrect. There is no indication that the author of Passage 1 feels pride with regard to the term, or that the author of Passage 2 feels embarrassment.

Explanation for Incorrect Answer D :  Choice (D) is incorrect. Though the author of Passage 1 seems to express approval of Twain as a humorist, the author of Passage 2 expresses contempt for modern humorists rather than the self-satisfaction that smugness suggests.

24 How would the author of Passage 2 most likely respond to the claim made by the author of Passage 1 that Twain “admired the citizens of every nation, seeking beneath the surface veneer the universal traits of that nation’s humanity” (lines 28-30)?

ANSWERS AND EXPLANATIONS Explanation for Correct Answer C :  Choice (C) is correct. The author of Passage 2 claims that Twain "finds every custom ridiculous that does not conform with the standard of the United States." This suggests that Twain's ability to understand other people and their customs is limited by his American perspective, or point of view.

Explanation for Incorrect Answer A :  Choice (A) is incorrect. The author of Passage 2 claims that "The modern humorist [such as Twain] is never unexpected."

Explanation for Incorrect Answer B :  Choice (B) is incorrect. The author of Passage 2 suggests that Twain does not share any characteristics with great humorists of the past.

Explanation for Incorrect Answer D :  Choice (D) is incorrect. The author of Passage 2 argues that Twain's humor has no wisdom at all, and that it is largely characterized by irreverence, or disrespect.

Explanation for Incorrect Answer E :  Choice (E) is incorrect. The author of Passage 2 does not claim that Twain is not funny, but rather that he never stops trying to be funny.

(A) Twain’s humor is so unexpected that he is able to see startling aspects about the behavior of people all over the world.

(B) Twain shares with the humorists of the past the tendency to attack foolishness whenever he sees it.

(C) Twain is limited by his inability to see things from anything other than an American perspective.

(D) Twain always uses his humor for a wise purpose and uses humor to emphasize the importance of people respecting each other.

(E) The effect of Twain’s probing beneath the surface is limited by the fact that he is not nearly as funny as many people claim he is.

    

Back to Score Report  

页码,20/21The Official SAT Online Course

2006-11-12file://E:\新建文件夹\b7.htm

UnRe

gistered

Page 93: SAT Online Course Test 2 Explanations

严禁用于商业用途!

Copyright © 2006 The College Board. All rights reserved. Privacy Policy Terms of Use Contact Us

 

页码,21/21The Official SAT Online Course

2006-11-12file://E:\新建文件夹\b7.htm

UnRe

gistered

Page 94: SAT Online Course Test 2 Explanations

严禁用于商业用途!

Help | Profile | My Organizer | My Bookmarks | Logout

Answers and Explanations

Test Sections

Section 1

Section 2

Section 3

Section 4

Section 6

Section 7

Section 8

Section 9

Section 10

Back to Score Report  

View Answers and Explanations     Online - Practice Test #2

1 Classified ads cost per words at City Newspaper. What is the least number of words that must be deleted from the text of a -word classified ad to reduce the cost to or less?

ANSWERS AND EXPLANATIONS Explanation for Correct Answer B : 

Choice (B) is correct. Since the ads cost per words, words would

cost exactly Therefore, at least words need to be deleted

from the text.

Explanation for Incorrect Answer A :  Choice (A) is not correct. Deleting words from the text leaves words. Since

an ad that is words long costs exactly an ad that is words long

costs more than

Explanation for Incorrect Answer C :  Choice (C) is not correct. Deleting words from the text would reduce the cost to

or less, but is the least number of words that need to deleted from the

text to reduce the cost to or less.

Explanation for Incorrect Answer D :  Choice (D) is not correct. Deleting words from the text would reduce the cost

to or less, but is the least number of words that need to deleted from

the text to reduce the cost to or less.

Explanation for Incorrect Answer E :  Choice (E) is not correct. Deleting words from the text would reduce the cost to

or less, but is the least number of words that need to deleted from the

text to reduce the cost to or less.

(A)

(B)

(C)

(D)

(E)

2

If then

(A)

(B)

(C)

(D)

(E)

页码,1/13The Official SAT Online Course

2006-11-12file://E:\新建文件夹\b8.htm

UnRe

gistered

Page 95: SAT Online Course Test 2 Explanations

严禁用于商业用途!

ANSWERS AND EXPLANATIONS Explanation for Correct Answer A :  Choice (A) is correct. If then Subtracting

from both sides gives and multiplying by gives

Explanation for Incorrect Answer B :  Choice (B) is not correct. If then substituting for in the equation

gives which simplifies to which is a false statement.

Explanation for Incorrect Answer C :  Choice (C) is not correct. If then substituting for in the equation

gives which simplifies to which is a false statement.

Explanation for Incorrect Answer D :  Choice (D) is not correct. If then substituting for in the equation

gives which simplifies to which is a false statement.

Explanation for Incorrect Answer E :  Choice (E) is not correct. If then substituting for in the equation

gives which simplifies to which is a false statement.

3

In the figure above, what is the value of

ANSWERS AND EXPLANATIONS Explanation for Correct Answer B :  Choice (B) is correct. For each of the right triangles, two of the three angle measures are given. The fact that the degree measures of the angles of a triangle add up to means and Therefore,

Explanation for Incorrect Answer A :  Choice (A) is not correct. The sum of the three angles of the large triangle is

which should add up to However, if were then would equal only

Explanation for Incorrect Answer C :  Choice (C) is not correct. The sum of the three angles of the large triangle is

which should add up to However, if were then the sum would equal

(A)

(B)

(C)

(D)

(E)

页码,2/13The Official SAT Online Course

2006-11-12file://E:\新建文件夹\b8.htm

UnRe

gistered

Page 96: SAT Online Course Test 2 Explanations

严禁用于商业用途!

Explanation for Incorrect Answer D :  Choice (D) is not correct. The sum of the three angles of the large triangle is

which should add up to However, if were then the sum would equal

Explanation for Incorrect Answer E :  Choice (E) is not correct. The sum of the three angles of the large triangle is

which should add up to However, if were then the sum would equal

4If which of the following is greater than

ANSWERS AND EXPLANATIONS Explanation for Correct Answer E : 

Choice (E) is correct. If and are both positive, then is positive. Therefore,

is greater than

Explanation for Incorrect Answer A :  Choice (A) is not correct. Since and are positive and dividing both

sides of the inequality by gives so

Explanation for Incorrect Answer B :  Choice (B) is not correct. Since and are positive and dividing both

sides of the inequality by gives . On the other hand, dividing both sides of

the inequality by gives So,

Explanation for Incorrect Answer C :  Choice (C) is not correct. Since and are positive and dividing both

sides of the inequality by gives while dividing both sides of the inequality

by gives So, Since and it follows that

Explanation for Incorrect Answer D : 

Choice (D) is not correct. Since and are positive,

(A)

(B)

(C)

(D)

(E)

页码,3/13The Official SAT Online Course

2006-11-12file://E:\新建文件夹\b8.htm

UnRe

gistered

Page 97: SAT Online Course Test 2 Explanations

严禁用于商业用途!

5 If what is the value of

ANSWERS AND EXPLANATIONS Explanation for Correct Answer B : 

Choice (B) is correct. so it follows that

Explanation for Incorrect Answer A : 

Choice (A) is not correct.

Explanation for Incorrect Answer C : 

Choice (C) is not correct.

Explanation for Incorrect Answer D : 

Choice (D) is not correct.

Explanation for Incorrect Answer E : 

Choice (E) is not correct.

(A)

(B)

(C)

(D)

(E)

6

In the figure above, If (not shown) is another line in the plane, what is the least number of points at which can intersect these four lines?

ANSWERS AND EXPLANATIONS Explanation for Correct Answer C :  Choice (C) is correct. If line is parallel to lines and then will not

intersect either or but must then intersect both lines and Moreover,

since lines and intersect only at line (and lines and are not the

same), line will intersect the lines and at two distinct points. If line is

not parallel to lines and it must intersect each of the lines and Therefore, the least possible number of points at which line can intersect these four lines is two.

(A) None

(B) One

(C) Two

(D) Three

(E) Four

页码,4/13The Official SAT Online Course

2006-11-12file://E:\新建文件夹\b8.htm

UnRe

gistered

Page 98: SAT Online Course Test 2 Explanations

严禁用于商业用途!

Explanation for Incorrect Answer A :  Choice (A) is not correct. The only way line could avoid intersecting any of these

lines is if were parallel to all four of them. However, only two of the four lines are

parallel to each other, so it is not possible for to be parallel to all four of the lines.

Explanation for Incorrect Answer B :  Choice (B) is not correct. The only way line could avoid intersecting any three of

these lines is if were parallel to three of them. However, only two of the four

lines are parallel to each other, so it is not possible for to be parallel to three of the lines.

Explanation for Incorrect Answer D :  Choice (D) is not correct. While it is possible that line can intersect the other lines at three points, two is also a possible number of intersections, and the question asks for the least number.

Explanation for Incorrect Answer E :  Choice (E) is not correct. While it is possible that line can intersect the other lines at four points, two is also a possible number of intersections, and the question asks for the least number.

7 If which of the following must be true?

ANSWERS AND EXPLANATIONS Explanation for Correct Answer C :  Choice (C) is correct. If then is between and This means that the distance between and is less than This can be written using

absolute value notation as or,

Explanation for Incorrect Answer A : 

Choice (A) is not correct. If then but which is

not less than

Explanation for Incorrect Answer B : 

Choice (B) is not correct. If then but which is not

greater than

Explanation for Incorrect Answer D : 

Choice (D) is not correct. If then but which is

not greater than

Explanation for Incorrect Answer E : 

Choice (E) is not correct. If then but

which is not less than

(A)

(B)

(C)

(D)

(E)

8

页码,5/13The Official SAT Online Course

2006-11-12file://E:\新建文件夹\b8.htm

UnRe

gistered

Page 99: SAT Online Course Test 2 Explanations

严禁用于商业用途!

The table above gives values of the linear function for selected values of Which of the following defines

ANSWERS AND EXPLANATIONS Explanation for Correct Answer D :  Choice (D) is correct. For each value of in the table, the value given by the

formula equals the value of in the table. In the table, is always

less than

Explanation for Incorrect Answer A : 

Choice (A) is not correct. When

Explanation for Incorrect Answer B : 

Choice (B) is not correct. When

Explanation for Incorrect Answer C : 

Choice (C) is not correct. When

Explanation for Incorrect Answer E : 

Choice (E) is not correct. When

(A)

(B)

(C)

(D)

(E)

9

The average (arithmetic mean) of and is equal to the median of and If which of the following must be equal to

ANSWERS AND EXPLANATIONS Explanation for Correct Answer A :  Choice (A) is correct. Since the median of and is Since the average of and is equal to the median of and it follows that

This simplifies to and further to and

(A)

(B)

(C)

(D)

(E)

页码,6/13The Official SAT Online Course

2006-11-12file://E:\新建文件夹\b8.htm

UnRe

gistered

Page 100: SAT Online Course Test 2 Explanations

严禁用于商业用途!

finally to

Explanation for Incorrect Answer B :  Choice (B) is not correct. Since and the average and median of the

three numbers and are equal, it follows that If were also

equal to then would have to be equal to This would imply

that This is impossible because

Explanation for Incorrect Answer C :  Choice (C) is not correct. Since and the average and median of the

three numbers and are equal, it follows that If were also

equal to then would have to be equal to This would imply

that This is impossible because

Explanation for Incorrect Answer D :  Choice (D) is not correct. Since and the average and median of the

three numbers and are equal, it follows that If were also

equal to then would have to be equal to This would imply

that This is impossible because

Explanation for Incorrect Answer E :  Choice (E) is not correct. Since and the average and median of the

three numbers and are equal, it follows that If were also

equal to then would have to be equal to This would imply that

was equal to so and would be equal. This is

impossible because

10 There are exposures on a certain roll of film. The cost of the roll of film is dollars, and the cost to develop the film is cents per exposure. What is the cost, in dollars, to purchase and develop the roll of film?

ANSWERS AND EXPLANATIONS Explanation for Correct Answer A :  Choice (A) is correct. The cost to purchase and develop the roll of film is the sum of the cost to purchase the roll, which is dollars, and the cost to develop the film, which is the number of exposures multiplied by the cost to develop each

exposure of dollars. Thus, the total cost, in dollars, is

(A)

(B)

(C)

(D)

(E)

页码,7/13The Official SAT Online Course

2006-11-12file://E:\新建文件夹\b8.htm

UnRe

gistered

Page 101: SAT Online Course Test 2 Explanations

严禁用于商业用途!

Explanation for Incorrect Answer B :  Choice (B) is not correct. This expression interchanges the cost of the roll of film and the cost per exposure to develop the film.

Explanation for Incorrect Answer C :  Choice (C) is not correct. This expression represents the product, not the sum, of the cost of the roll of film and the cost to develop the film.

Explanation for Incorrect Answer D :  Choice (D) is not correct. This expression gives the cost, in cents, to purchase and develop the roll of film.

Explanation for Incorrect Answer E :  Choice (E) is not correct. In this expression the roles and the units of and

have been interchanged. The multiplication by might convert from dollars to cents, but the question asks for the cost in dollars.

11 A circular piece of cardboard is cut in half along a diameter. If the diameter is inches, what is the perimeter, in inches, of one of the semicircular pieces?

ANSWERS AND EXPLANATIONS Explanation for Correct Answer B :  Choice (B) is correct. The perimeter of each of the semicircular pieces is made up of two parts: the diameter of the original circle and half of the circumference of the original circle. Since the diameter is inches, the circumference of the original circle was inches. This means that the perimeter, in inches, of one of the

semicircular pieces is which simplifies to

Explanation for Incorrect Answer A :  Choice (A) is not correct. The perimeter of a semicircular piece is made up of two parts: the diameter of the original circle and half of the circumference of the original circle. Though is half the perimeter of the circle, is not the full measure of the diameter.

Explanation for Incorrect Answer C :  Choice (C) is not correct. The perimeter of a semicircular piece is made up of two parts: the diameter of the original circle and half of the circumference of the original circle. is the full circumference the circle while is only half of the measure of the diameter.

Explanation for Incorrect Answer D :  Choice (D) is not correct. The perimeter of a semicircular piece is made up of two parts: the diameter of the original circle and half of the circumference of the original circle. Though is the diameter, is the full circumference of the circle, instead of half of it.

Explanation for Incorrect Answer E :  Choice (E) is not correct. The question asks for the perimeter of only one of the semicircular pieces, not the total perimeter for both.

(A)

(B)

(C)

(D)

(E)

12

页码,8/13The Official SAT Online Course

2006-11-12file://E:\新建文件夹\b8.htm

UnRe

gistered

Page 102: SAT Online Course Test 2 Explanations

严禁用于商业用途!

The figure above shows the remaining pieces of a paper chain from which a section has been removed. The first link of the original chain was red and the colors of the

links formed the repeating pattern of red yellow green white

and blue from left to right. Which of the following could be the number of links in

the section that was removed?

ANSWERS AND EXPLANATIONS Explanation for Correct Answer D :  Choice (D) is correct. The first piece of the chain ends with G, and for the chain to get to Y from G, it needs to go through three colors – W, B, and R. However, while

is a possibility for the number of missing links, it is not an answer choice. Since

the chain is set up so that the coloring system repeats itself every links, any

number that is exactly more than a multiple of is a possibility. The only

answer choice that has this property is because

Explanation for Incorrect Answer A :  Choice (A) is not correct. From the explanation for the correct answer, the number of missing links must be more than a multiple of The number does not

have that property, because

Explanation for Incorrect Answer B :  Choice (B) is not correct. From the explanation for the correct answer, the number of missing links must be more than a multiple of The number does not

have that property, because

Explanation for Incorrect Answer C :  Choice (C) is not correct. From the explanation for the correct answer, the number of missing links must be more than a multiple of The number does not

have that property, because

Explanation for Incorrect Answer E :  Choice (E) is not correct. From the explanation for the correct answer, the number of missing links must be more than a multiple of The number does not

have that property, because

(A)

(B)

(C)

(D)

(E)

13

In the figure above, the length of is and the length of each side of

quadrilateral is What is the area of quadrilateral

(A)

(B)

(C)

页码,9/13The Official SAT Online Course

2006-11-12file://E:\新建文件夹\b8.htm

UnRe

gistered

Page 103: SAT Online Course Test 2 Explanations

严禁用于商业用途!

ANSWERS AND EXPLANATIONS Explanation for Correct Answer D :  Choice (D) is correct. The area of quadrilateral is equal to the sum of the

area of quadrilateral and the area of right triangle Quadrilateral

is a rhombus, and therefore a parallelogram, because all four sides are equal. The area of a parallelogram is given by the formula where is the

length of the base and is the height. For the height is or and

the base is so the area of is Right triangle has or

as its height and the side as its base. Since the hypotenuse, of

has length using the Pythagorean theorem to find the length of gives

The area of is then Therefore,

the total area of quadrilateral is

Explanation for Incorrect Answer A :  Choice (A) is not correct. From the explanation for the correct answer, the length of

is and the height is However, the figure is not a rectangle

(a rectangle of length and height would require another piece congruent to

), so the area of must be less than

Explanation for Incorrect Answer B : 

Choice (B) is not correct. For the rhombus the height is or and

the base is so the area of is If the total area for were

then the area of would have to be instead of

Explanation for Incorrect Answer C : 

Choice (C) is not correct. For the rhombus the height is or and

the base is so the area of is If the total area for were

then the area of would have to be instead of

Explanation for Incorrect Answer E : 

Choice (E) is not correct. For the rhombus the height is or and

the base is so the area of is If the total area for were

then the area of would have to be instead of

(D)

(E)

14

A list consists of all possible three-letter arrangements formed by using the letters above such that the first letter is and one of the remaining letters is If no letter is used more than once in an arrangement in the list and one three-letter arrangement is randomly selected from the list, what is the probability that the arrangement selected will be

(A)

(B)

(C)

页码,10/13The Official SAT Online Course

2006-11-12file://E:\新建文件夹\b8.htm

UnRe

gistered

Page 104: SAT Online Course Test 2 Explanations

严禁用于商业用途!

ANSWERS AND EXPLANATIONS Explanation for Correct Answer D :  Choice (D) is correct. Since the three-letter arrangement must start with and must contain the letter there are 10 possible arrangements :

and So, the probability

of being randomly selected is

Explanation for Incorrect Answer A :  Choice (A) is not correct. If there were only three-letter arrangements meeting

the conditions of the problem, and if was one of those then the

probability of randomly selecting from the list would be However, there

are not arrangements meeting the conditions of the problem.

Explanation for Incorrect Answer B :  Choice (B) is not correct. If there were only three-letter arrangements meeting

the conditions of the problem, and if was one of those then the

probability of randomly selecting from the list would be However, there

are not arrangements meeting the conditions of the problem.

Explanation for Incorrect Answer C :  Choice (C) is not correct. If there were only three-letter arrangements meeting

the conditions of the problem, and if was one of those then the

probability of randomly selecting from the list would be However, there

are not arrangements meeting the conditions of the problem.

Explanation for Incorrect Answer E :  Choice (E) is not correct. If there were three-letter arrangements meeting the

conditions of the problem, and if was one of those then the probability

of randomly selecting from the list would be However, since no letter

can be used more than once, there are only not arrangements meeting the conditions of the problem.

(D)

(E)

15

The pyramid above has a square base and four congruent triangular faces. The height of the pyramid is inches, and each edge of the base is inches long. What is the length of in inches?

页码,11/13The Official SAT Online Course

2006-11-12file://E:\新建文件夹\b8.htm

UnRe

gistered

Page 105: SAT Online Course Test 2 Explanations

严禁用于商业用途!

ANSWERS AND EXPLANATIONS Explanation for Correct Answer B :  Choice (B) is correct. Since the base of the pyramid is square, its two diagonals intersect at the center of the square. The sides of the square have a length of so

the length of each diagonal of the square base is Therefore, the distance

from each vertex of the base to the center of the square base is Since the

triangular faces of the pyramid are identical, the point at which they meet is

directly above the center of the square base. Segment is the hypotenuse of a

right triangle, composed of the vertical line connecting to the center of the

base, and the segment from the center of the base to The height of the pyramid is so the vertical side of the triangle will have a length of By the Pythagorean

theorem,

Explanation for Incorrect Answer A :  Choice (A) is not correct. See the explanation for the correct answer (B).

Explanation for Incorrect Answer C :  Choice (C) is not correct. See the explanation for the correct answer (B).

Explanation for Incorrect Answer D :  Choice (D) is not correct. See the explanation for the correct answer (B).

Explanation for Incorrect Answer E :  Choice (E) is not correct. See the explanation for the correct answer (B).

(A) (approximately )

(B) (approximately )

(C) (approximately )

(D) (approximately )

(E) (approximately )

16

For all let the function be defined as Which of the following

is equal to

ANSWERS AND EXPLANATIONS Explanation for Correct Answer B : 

Choice (B) is correct. If then

Because

we have

Explanation for Incorrect Answer A : 

Choice (A) is not correct. This is not equal to

(A)

(B)

(C)

(D)

(E)

页码,12/13The Official SAT Online Course

2006-11-12file://E:\新建文件夹\b8.htm

UnRe

gistered

Page 106: SAT Online Course Test 2 Explanations

严禁用于商业用途!

Explanation for Incorrect Answer C : 

Choice (C) is not correct. This is not equal to

Explanation for Incorrect Answer D : 

Choice (D) is not correct. This is not equal to

Explanation for Incorrect Answer E : 

Choice (E) is not correct. This is not equal to

    

Back to Score Report  

Copyright © 2006 The College Board. All rights reserved. Privacy Policy Terms of Use Contact Us

 

页码,13/13The Official SAT Online Course

2006-11-12file://E:\新建文件夹\b8.htm

UnRe

gistered

Page 107: SAT Online Course Test 2 Explanations

严禁用于商业用途!

Help | Profile | My Organizer | My Bookmarks | Logout

Answers and Explanations

Test Sections

Section 1

Section 2

Section 3

Section 4

Section 6

Section 7

Section 8

Section 9

Section 10

Back to Score Report  

View Answers and Explanations     Online - Practice Test #2

1 Seeking to ------- what people view and read by determining what art and literature should be available, censorship laws directly ------- free expression.

ANSWERS AND EXPLANATIONS Explanation for Correct Answer D :  Choice (D) is correct. "Control" means to regulate. "Prohibit" means to prevent. If one were to insert these terms into the text, the sentence would read "Seeking to control what people view and read by determining what art and literature should be available, censorship laws directly prohibit free expression." By regulating what people read and view, censorship laws do in fact prevent free expression.

Explanation for Incorrect Answer A :  Choice (A) is incorrect. "Govern" means to control. "Liberate" means to free. If one were to insert these terms into the text, the sentence would read "Seeking to govern what people view and read by determining what art and literature should be available, censorship laws directly liberate free expression." Although censorship laws do seek to control what people read and view, these laws do not allow people to express themselves freely. In fact, they do the opposite.

Explanation for Incorrect Answer B :  Choice (B) is incorrect. "Juxtapose" means to place side by side. "Prescribe" means to dictate. If one were to insert these terms into the text, the sentence would read "Seeking to juxtapose what people view and read by determining what art and literature should be available, censorship laws directly prescribe free expression." Censorship laws do not seek to "juxtapose" two things. Censorship laws also do not "prescribe" free expression. In fact, these laws do just the opposite.

Explanation for Incorrect Answer C :  Choice (C) is incorrect. "Defer" means to put off or postpone. "Nullify" means to negate. If one were to insert these terms into the text, the sentence would read "Seeking to defer what people view and read by determining what art and literature should be available, censorship laws directly nullify free expression." Censorship laws do negate free expression and thus prevent people from viewing and reading certain things. However, censorship laws do not put off or postpone people's viewing and reading; they block it altogether.

Explanation for Incorrect Answer E :  Choice (E) is incorrect. "Balance" means to make equal. "Promote" means to advocate. If one were to insert these terms into the text, the sentence would read "Seeking to balance what people view and read by determining what art and literature should be available, censorship laws directly promote free expression." Censorship laws do not "balance" the type of art and literature that people view and read. Also, these laws do not advocate free expression. Instead, they discourage and prevent it.

(A) govern. . liberate

(B) juxtapose. . prescribe

(C) defer. . nullify

(D) control. . prohibit

(E) balance. . promote

2Myra laughed exuberantly and embraced her friends repeatedly, so ------- was she

页码,1/16The Official SAT Online Course

2006-11-12file://E:\新建文件夹\b9.htm

UnRe

gistered

Page 108: SAT Online Course Test 2 Explanations

严禁用于商业用途!

about having been selected.

ANSWERS AND EXPLANATIONS Explanation for Correct Answer C :  Choice (C) is correct. "Euphoric" means full of joy. If one were to insert this term into the text, the sentence would read "Myra laughed exuberantly and embraced her friends repeatedly, so euphoric was she about having been selected." The missing term describes Myra’s mood as evidenced by her actions, which include laughing and hugging her friends. It makes sense to describe Myra’s energetic displays of happiness as "euphoric."

Explanation for Incorrect Answer A :  Choice (A) is incorrect. "Ambivalent" means having contradictory feelings. If one were to insert this term into the text, the sentence would read "Myra laughed exuberantly and embraced her friends repeatedly, so ambivalent was she about having been selected." The missing term describes Myra’s mood as evidenced by her actions, which include laughing and hugging her friends. Such actions reflect pure joy, not contradictory feelings.

Explanation for Incorrect Answer B :  Choice (B) is incorrect. "Quizzical" means disbelieving or questioning. If one were to insert this term into the text, the sentence would read "Myra laughed exuberantly and embraced her friends repeatedly, so quizzical was she about having been selected." Myra’s energetic displays of happiness are not a reflection of questioning. Nothing in the sentence indicates that Myra is questioning anything.

Explanation for Incorrect Answer D :  Choice (D) is incorrect. "Jaded" means worn out or wearied. If one were to insert this term into the text, the sentence would read "Myra laughed exuberantly and embraced her friends repeatedly, so jaded was she about having been selected." Given her energetic display of emotion, it does not make sense to describe Myra as "jaded," or weary.

Explanation for Incorrect Answer E :  Choice (E) is incorrect. "Exacting" means severely demanding. If one were to insert this term into the text, the sentence would read "Myra laughed exuberantly and embraced her friends repeatedly, so exacting was she about having been selected." The missing term describes Myra’s mood as evidenced by her actions, which include laughing "exuberantly," or with high spirits, and hugging her friends. It does not make sense to describe Myra as severely demanding.

(A) ambivalent

(B) quizzical

(C) euphoric

(D) jaded

(E) exacting

3It would be a waste of time for any reviewer to bother ------- a book whose utter worthlessness is ------- to even the least discerning reader.

ANSWERS AND EXPLANATIONS Explanation for Correct Answer D :  Choice (D) is correct. "Condemning" means expressing strong disapproval. "Obvious" means evident or easily understood. If one were to insert these terms into the text, the sentence would read "It would be a waste of time for any reviewer to bother condemning a book whose utter worthlessness is obvious to even the

(A) enjoying. . doubtful

(B) mocking . . figurative

(C) assessing . . welcome

(D) condemning. . obvious

(E) ignoring . . obnoxious

页码,2/16The Official SAT Online Course

2006-11-12file://E:\新建文件夹\b9.htm

UnRe

gistered

Page 109: SAT Online Course Test 2 Explanations

严禁用于商业用途!

least discerning reader." This sentence makes sense: a reviewer should not bother to disapprove of a book whose "worthlessness," or poor quality, is evident to even the least "discerning," or perceptive, reader.

Explanation for Incorrect Answer A :  Choice (A) is incorrect. "Enjoying" means having fun. "Doubtful" means uncertain. If one were to insert these terms into the text, the sentence would read "It would be a waste of time for any reviewer to bother enjoying a book whose utter worthlessness is doubtful to even the least discerning reader." It makes little sense to advise a book reviewer not to bother having fun with a book whose quality is uncertain. It is the reviewer's job to judge the worth or quality of the book in question.

Explanation for Incorrect Answer B :  Choice (B) is incorrect. "Mocking" means making fun of. "Figurative" means metaphorical or not literal. If one were to insert these terms into the text, the sentence would read "It would be a waste of time for any reviewer to bother mocking a book whose utter worthlessness is figurative to even the least discerning reader." While a reviewer might make fun of a clearly worthless book, it does not make sense to say that a book's worthlessness is "figurative."

Explanation for Incorrect Answer C :  Choice (C) is incorrect. "Assessing" means evaluating. "Welcome" means giving pleasure. If one were to insert these terms into the text, the sentence would read "It would be a waste of time for any reviewer to bother assessing a book whose utter worthlessness is welcome to even the least discerning reader." A reader would be unlikely to consider the worthlessness of any book "welcome."

Explanation for Incorrect Answer E :  Choice (E) is incorrect. "Ignoring" means refusing to notice. "Obnoxious" means offensive or unpleasant. If one were to insert these terms into the text, the sentence would read "It would be a waste of time for any reviewer to bother ignoring a book whose utter worthlessness is obnoxious to even the least discerning reader." This sentence is illogical: "ignoring" something could never be a waste of time.

4

The new policy has been called a quiet revolution because, though introduced without -------, it is already producing ------- changes.

ANSWERS AND EXPLANATIONS Explanation for Correct Answer B :  Choice (B) is correct. "Fanfare" means showy display. "Momentous" means of the highest importance. If one were to insert these terms into the text, the sentence would read "The new policy has been called a quiet revolution because, though introduced without fanfare, it is already producing momentous changes." The correct answer builds on the idea that the policy is a "quiet revolution"-- something that, though understated, has a great effect. A policy that lacks showy display but produces important changes could definitely be called a quiet revolution.

Explanation for Incorrect Answer A :  Choice (A) is incorrect. A "warning" is an alert. "Specious" means having a false appearance. If one were to insert these terms into the text, the sentence would read "The new policy has been called a quiet revolution because, though introduced without warning, it is already producing specious changes." A policy that is introduced without a prior alert may or may not be called a quiet revolution. However, "specious changes," or apparent changes that are actually not changes at all, would not be part of a "quiet revolution."

(A) warning . . specious

(B) fanfare . . momentous

(C) concealment . . transient

(D) hesitation . . ostensible

(E) debate . . negligible

页码,3/16The Official SAT Online Course

2006-11-12file://E:\新建文件夹\b9.htm

UnRe

gistered

Page 110: SAT Online Course Test 2 Explanations

严禁用于商业用途!

Explanation for Incorrect Answer C :  Choice (C) is incorrect. "Concealment" means secrecy. "Transient" means remaining in place for only a brief time. If one were to insert these terms into the text, the sentence would read "The new policy has been called a quiet revolution because, though introduced without concealment, it is already producing transient changes." The correct answer builds on the idea that the policy is a quiet revolution, or something that, though understated or restrained, has a great effect. A policy that is introduced without secrecy would probably not be called quiet. Moreover, it does not make sense to refer to changes as "transient."

Explanation for Incorrect Answer D :  Choice (D) is incorrect. "Hesitation" means a pause due to uncertainty. "Ostensible" means seeming to be real. If one were to insert these terms into the text, the sentence would read "The new policy has been called a quiet revolution because, though introduced without hesitation, it is already producing ostensible changes." While a revolutionary policy might be introduced without "hesitation," one would not describe the changes produced by such a policy as only seeming to be real.

Explanation for Incorrect Answer E :  Choice (E) is incorrect. A "debate" is a discussion between two opposing sides. "Negligible" means extremely unimportant. If one were to insert these terms into the text, the sentence would read "The new policy has been called a quiet revolution because, though introduced without debate, it is already producing negligible changes." The changes produced by a policy called a quiet revolution would not be unimportant.

5When two chemical compounds are combined, a ------- effect can be achieved: the resulting combination can be more potent than either of the individual compounds alone.

ANSWERS AND EXPLANATIONS Explanation for Correct Answer A :  Choice (A) is correct. "Synergistic" refers to two or more things working together to create something powerful. If one were to insert this term into the text, the sentence would read "When two chemical compounds are combined, a synergistic effect can be achieved: the resulting combination can be more potent than either of the individual compounds alone." The missing term describes a combination that is "more potent," or stronger, than either of its "compounds," or parts, is alone. A mixture whose combination is more powerful than its individual parts is "synergistic" by definition.

Explanation for Incorrect Answer B :  Choice (B) is incorrect. "Naturalistic" refers to imitating nature. If one were to insert this term into the text, the sentence would read "When two chemical compounds are combined, a naturalistic effect can be achieved: the resulting combination can be more potent than either of the individual compounds alone." "Naturalistic" refers to something that is like or imitates nature, not something that is stronger than its individual parts.

Explanation for Incorrect Answer C :  Choice (C) is incorrect. "Competitive" refers to the enjoyment of contests or rivalry. If one were to insert this term into the text, the sentence would read "When two chemical compounds are combined, a competitive effect can be achieved: the resulting combination can be more potent than either of the individual compounds alone." A mixture whose combination is more powerful than its individual parts has nothing to do with being "competitive."

Explanation for Incorrect Answer D :  Choice (D) is incorrect. "Retroactive" means applying to a prior period. If one were to insert this term into the text, the sentence would read "When two chemical compounds are combined, a retroactive effect can be achieved: the resulting

(A) synergistic

(B) naturalistic

(C) competitive

(D) retroactive

(E) neutralizing

页码,4/16The Official SAT Online Course

2006-11-12file://E:\新建文件夹\b9.htm

UnRe

gistered

Page 111: SAT Online Course Test 2 Explanations

严禁用于商业用途!

combination can be more potent than either of the individual compounds alone." The fact that a combination is more powerful than its individual parts has nothing to do with it being "retroactive," or referring to a prior period.

Explanation for Incorrect Answer E :  Choice (E) is incorrect. "Neutralizing" means counteracting or making ineffective. If one were to insert this term into the text, the sentence would read "When two chemical compounds are combined, a neutralizing effect can be achieved: the resulting combination can be more potent than either of the individual compounds alone." "Neutralizing" the compounds would make them less potent rather than more potent.

6 Artists who are described as ------- are the first to experiment with new forms or concepts.

ANSWERS AND EXPLANATIONS Explanation for Correct Answer E :  Choice (E) is correct. "Avant-garde" means innovative or cutting-edge. If one were to insert this term into the text, the sentence would read "Artists who are described as avant-garde are the first to experiment with new forms or concepts." The missing term is defined by the final phrase "the first to experiment with new forms or concepts." This is precisely the definition of an "avant-garde" person.

Explanation for Incorrect Answer A :  Choice (A) is incorrect. "Aesthetic" means artistic. If one were to insert this term into the text, the sentence would read "Artists who are described as aesthetic are the first to experiment with new forms or concepts." An artistic person would not necessarily be "the first to experiment with new forms or concepts."

Explanation for Incorrect Answer B :  Choice (B) is incorrect. "Partisan" means supporting a certain group. If one were to insert this term into the text, the sentence would read "Artists who are described as partisan are the first to experiment with new forms or concepts." A "partisan" person would not necessarily be "the first to experiment with new forms or concepts."

Explanation for Incorrect Answer C :  Choice (C) is incorrect. "Decorous" means showing good manners. If one were to insert this term into the text, the sentence would read "Artists who are described as decorous are the first to experiment with new forms or concepts." A well-mannered person would not necessarily be "the first to experiment with new forms or concepts."

Explanation for Incorrect Answer D :  Choice (D) is incorrect. "Cerebral" means intellectual. If one were to insert this term into the text, the sentence would read "Artists who are described as cerebral are the first to experiment with new forms or concepts." Although artists who experiment with new forms and concepts can be cerebral, they do not have to be cerebral.

(A) aesthetic

(B) partisan

(C) decorous

(D) cerebral

(E) avant-garde

The following passage was adapted from a book about the Great Plains, a plateau extending westward from the prairies of central North America to the eastern base of the Rocky Mountains.

      Before the railroads were built, the way west followed

页码,5/16The Official SAT Online Course

2006-11-12file://E:\新建文件夹\b9.htm

UnRe

gistered

Page 112: SAT Online Course Test 2 Explanations

严禁用于商业用途!

 the rivers: west along the Platte into Wyoming, over South

 Pass, up the Snake River into the Oregon Territory; or up

Linethe Missouri through the Dakotas and into Montana, then

5west along the Yellowstone. It was the easiest but not the

 most accurate way to see the country. The country looked

 better or worse from the prospect of the river; I can’t say

 which, not having gone that way. But the country looked

 different, certainly, not at all like the Great Plains. There

10are many reasons why it could not have seemed the same.

      A river is an edge, for one thing, and not simply in the

 cartographer’s sense that it divides one piece of ground

 from another. It is a biological edge. There worlds collide,

 strangers meet, and much business, although not of the

15monetary kind, is transacted. Edges in the natural world

 are like cities in human cultures. They are the cosmopolitan

 places—populous, noisy, gaudy, rich, exciting—where one

 expects the unexpected and the extraordinary is ordinary.

 They are altogether unlike the provinces, where the surprises

20lie not in discovering what is odd or new but in appreciating,

 at last, what is routine and everyday, a larger accomplish-

 ment than one might imagine. The rivers of the earliest

 westward travelers passed through the provinces, but these

 rivers revealed a world that was not, in itself, characteristic

25 of those regions.

      Rivers carry water, for instance, but the region of the

 Great Plains is by its nature arid—not so arid as the deserts, although for a long time the Great Plains were

页码,6/16The Official SAT Online Course

2006-11-12file://E:\新建文件夹\b9.htm

UnRe

gistered

Page 113: SAT Online Course Test 2 Explanations

严禁用于商业用途!

  regarded

 as a desert, but arid enough to inhibit the growth of trees,

30except along rivers. You could no more know the Great

 Plains by canoeing up the Missouri than you could see

 the Sonoran Desert by rafting down the Colorado. River

 travelers poled or steamed up the channels by day and

 fished for supper by twilight. The Blackfeet, the lords of

35the Great Plains and the prairie’s most serious students,

 would no sooner have dined on catfish than we would on

 a dish of fricasseed sewer rat. The mucus-covered creatures

 of the muddy river bottoms, the Blackfeet thought, were

 simply not the best the plains had to offer; far from being

40 palatable, catfish were repulsive, disgusting.

      The rivers, moreover, seek the level, that is, the low

 ground, but the plains are the province of the big sky. The

 rivers are always running away to the sea, but the plains

 are always rising toward the mountains. They are contrary

45forces working in contrary directions. The rivers dig in; the

 plains surmount. A river closes in, rounds the bend, runs

 between banks, hides shallows and snags, tumbles over

 rapids, skirts islands, is forever calling attention to itself,

 like a trail, which a river inevitably becomes. The plains,

50on the other hand, open out, unfold, beg the long and track-

 less view. The river draws a line; the plains reveal a space.

      It is like the difference between an interstate expressway

 and a county road. An interstate is broad and swift and, in

 its own way, keeps to the level. You can drive on an inter-

页码,7/16The Official SAT Online Course

2006-11-12file://E:\新建文件夹\b9.htm

UnRe

gistered

Page 114: SAT Online Course Test 2 Explanations

严禁用于商业用途!

55state across the most endlessly enchanting of countrysides

 and encounter only an unrelieved monotony. The express-

 way exists in its own world, an unwalled tunnel, and moves

 at its own urgent pace. It has a rhythm and a rigidly regular

 time quite distinct from the landscape it crosses. It would

60 not serve its purposes in any other way.

      The county road, on the other hand, moves in and

 through the landscape and exists as one more feature of

 it. Where there is a tree and a sun, the road falls under

 shadow; where there is a stream, the road follows down

65one bank, across the water, and up the other bank; where

 a tall hill intervenes, the road goes around rather than

 through it and the traveler enjoys the sensation of having

 moved among hills; at the village, the road unhesitatingly

 takes it in, and your own pace slows to accommodate the

70taking in, rather than swooping to the right or left around

 the settlement at a curve calculated for high-speed safety.

 A stray cow might cross in front of you and you will be

 obliged to stop to let it pass, and so you will chance to

 hear the song of the meadowlark on the fence post. The

75hay wagon ahead cannot be gotten around; you are forced

 to reduce your own speed to the local standard, and so you

  see the marsh hawk circling above a pothole.

      The best way to go west, therefore, is the slowest way

 possible and across country rather than along the rivers,

80avoiding both the old watery rivers and the new ones of

  asphalt and cast concrete.

7 The author’s primary purpose in the passage is to

页码,8/16The Official SAT Online Course

2006-11-12file://E:\新建文件夹\b9.htm

UnRe

gistered

Page 115: SAT Online Course Test 2 Explanations

严禁用于商业用途!

ANSWERS AND EXPLANATIONS Explanation for Correct Answer C :  Choice (C) is correct. The author's central thesis – that one must travel across the land in order to know the richness of the prairie – is meant to encourage an increased appreciation or understanding of the Great Plains. As the author states, "the best way to go west, therefore, is the slowest way possible" (lines 78-79).

Explanation for Incorrect Answer A :  Choice (A) is incorrect. The author's main purpose is to encourage travelers to explore and better understand the prairie. The author is not writing a "narrative."

Explanation for Incorrect Answer B :  Choice (B) is incorrect. Although the author mentions the region's earliest travelers, relating the excitement of their exploration is not the main point of the passage.

Explanation for Incorrect Answer D :  Choice (D) is incorrect. In this passage, the author does not appear to be interested in whether or not the western rivers are beautiful, or whether they are less beautiful than they used to be.

Explanation for Incorrect Answer E :  Choice (E) is incorrect. The author's purpose is not to make the Great Plains seem mysterious, but rather to encourage readers to know and understand the prairie on its own terms.

(A) introduce the narrative figure of the traveler

(B) convey the excitement felt by the earliest explorers

(C) encourage an appreciation of the Great Plains

(D) establish the vanished beauty of western rivers

(E) confirm the mysterious nature of the Great Plains

8For the author, a “biological edge” (line 13) represents a

ANSWERS AND EXPLANATIONS Explanation for Correct Answer A :  Choice (A) is correct. The passage describes a biological edge as a place where “worlds collide, strangers meet” (lines 13-14). A biological edge is a "place where communities mingle," or mix.

Explanation for Incorrect Answer B :  Choice (B) is incorrect. The opposite is actually true: in line 13, a biological edge is a place where “worlds collide,” not a structure or limit that keeps different groups apart.

Explanation for Incorrect Answer C :  Choice (C) is incorrect. The author actually claims that a biological edge is similar to an urban environment, or city: "edges in the natural world are like cities in human cultures" (lines 15-16). They are places were different groups are thrown together with often unexpected results.

Explanation for Incorrect Answer D : 

(A) place where communities mingle

(B) barrier that separates different groups

(C) contrast to an urbanized environment

(D) change over time in the geography of the land

(E) clash between incompatible ways of life

页码,9/16The Official SAT Online Course

2006-11-12file://E:\新建文件夹\b9.htm

UnRe

gistered

Page 116: SAT Online Course Test 2 Explanations

严禁用于商业用途!

Choice (D) is incorrect. A biological edge does not represent a change over time, but a situation in which different kinds and systems of life interact.

Explanation for Incorrect Answer E :  Choice (E) is incorrect. According to the passage, the ways of life that exist at a biological edge are not incompatible. On the contrary, such an edge is a productive place of exchange among different groups.

9 In line 20, “appreciating” most nearly means

ANSWERS AND EXPLANATIONS Explanation for Correct Answer C :  Choice (C) is correct. The passage explains that edges "are altogether unlike the provinces, where the surprises lie not in discovering what is odd or new but in appreciating, at last, what is routine and everyday" (lines 19-21). Since surprises come about when one understands the routine aspects of life better, "appreciating" most nearly means "understanding better" in this context.

Explanation for Incorrect Answer A :  Choice (A) is incorrect. Nothing in the passage indicates that the provinces are in danger of being harmed or that they require preservation.

Explanation for Incorrect Answer B :  Choice (B) is incorrect. While "appreciating" can mean increasing in value over time, it does not make sense to say that the surprises of the provinces lie in increasing the value of the routine and everyday. Rather, "appreciating" means understanding the value that is already there.

Explanation for Incorrect Answer D :  Choice (D) is incorrect. In this context, "appreciating" means understanding the meaning or importance of something. It does not make sense to say that the surprises lie in "praising" the routine and everyday.

Explanation for Incorrect Answer E :  Choice (E) is incorrect. Although one may give thanks for something that one appreciates, it does not make sense to say that the surprises of the provinces lie in "thanking" what is routine and everyday.

(A) preserving from harm

(B) increasing in value

(C) understanding better

(D) praising

(E) thanking

10In lines 26-32 (“Rivers carry . . . Colorado”), the author implies that viewing the plains from the prospect of a river might lead one to conclude that the plains are

ANSWERS AND EXPLANATIONS Explanation for Correct Answer A :  Choice (A) is correct. The passage states that the Great Plains are "arid enough to inhibit the growth of trees, except along rivers" (lines 29-30). It makes sense to say that if one only saw the Great Plains from a river, where trees grow, one might get the mistaken impression that the plains are not arid.

(A) less arid than they actually are

(B) less populous than they actually are

(C) rising more rapidly toward the mountains than they actually do

(D) not fertile enough to sustain the growth of trees

(E) not as arid as the desert

页码,10/16The Official SAT Online Course

2006-11-12file://E:\新建文件夹\b9.htm

UnRe

gistered

Page 117: SAT Online Course Test 2 Explanations

严禁用于商业用途!

Explanation for Incorrect Answer B :  Choice (B) is incorrect. Nothing in the passage indicates that the perspective of river travel would lead one to think of the plains as less "populous," or crowded, than they actually are.

Explanation for Incorrect Answer C :  Choice (C) is incorrect. The passage indicates that "the rivers are always running away to the sea, but the plains are always rising toward the mountains" (lines 42-44). The passage implies that river travel would lead one to miss the fact that the plains are rising. It does not suggest that one would see the plains to be "rising more rapidly toward the mountains than they actually do."

Explanation for Incorrect Answer D :  Choice (D) is incorrect. The opposite is most likely true. The passage states that the Great Plains are "arid enough to inhibit the growth of trees, except along rivers" (lines 29-30). From the riverbanks, where trees grow, one would wrongly conclude that the plains are "fertile enough to sustain the growth of trees."

Explanation for Incorrect Answer E :  Choice (E) is incorrect. The point of lines 26-32 is that viewing the plains from the river gives one a mistaken impression. The plains are in fact "not as arid as the desert," so a traveler having this impression would be correct. Lines 26-32 focus instead on travelers' mistakes: "you could no more know the Great Plains by canoeing up the Missouri than you could see the Sonoran Desert by rafting down the Colorado."

11 The author mentions the Blackfeet (lines 34-40) primarily because

ANSWERS AND EXPLANATIONS Explanation for Correct Answer A :  Choice (A) is correct. The author holds up the Blackfeet as “the prairie’s most serious students” as an example of people who knew and appreciated the plains, and who had little interest in the very different life on the river. The fact that the Blackfeet refused to eat catfish from the rivers is offered as support for the author's claim that life on the rivers is very different from life on the open prairie.

Explanation for Incorrect Answer B :  Choice (B) is incorrect. The passage actually states the opposite: the Blackfeet did not use the resources of the rivers, even refusing to eat the catfish that swam there. Instead, they focused their energies on making use of the land.

Explanation for Incorrect Answer C :  Choice (C) is incorrect. While the Blackfeet probably cared about the "ecology" of the plains, that does not explain why the author mentions them. They are used to support the passage’s broader claims about the very different worlds one encounters on the river and on land.

Explanation for Incorrect Answer D :  Choice (D) is incorrect. Nothing in the passage indicates that river travelers learned from the Blackfeet, who stayed on the land and knew little of the waters.

Explanation for Incorrect Answer E :  Choice (E) is incorrect. The passage does not discuss the relationship between the Blackfeet and any other local people.

(A) they appreciated the plains

(B) they were experts in using the resources of the rivers

(C) they cared about the ecology of the plains

(D) river travelers learned a lot from them

(E) local people were in awe of them

页码,11/16The Official SAT Online Course

2006-11-12file://E:\新建文件夹\b9.htm

UnRe

gistered

Page 118: SAT Online Course Test 2 Explanations

严禁用于商业用途!

12 The author indicates that the view offered by the interstate is similar to that offered by the river in that both

ANSWERS AND EXPLANATIONS Explanation for Correct Answer E :  Choice (E) is correct. According to the passage, both the interstate and the river give an "erroneous," or incorrect, impression of the land. As the author explains, "you can drive on an interstate across the most endlessly enchanting of countrysides and encounter only an unrelieved monotony" (lines 54-56). Likewise, "the river draws a line; the plains reveal a space" (line 51). Interstates and rivers both separate the traveler from the true nature of the places they cross.

Explanation for Incorrect Answer A :  Choice (A) is incorrect. The author does not describe the view from the interstate as teeming with, or full of, the unexpected, but rather as dull and monotonous.

Explanation for Incorrect Answer B :  Choice (B) is incorrect. The author describes the river as "calling attention to itself" (line 48) and the interstate as existing "in its own world" (line 57). In other words, both the interstate and the river are cut off from the land, and are thus not reflective of changes in the environment.

Explanation for Incorrect Answer C :  Choice (C) is incorrect. It is not the river and highway that are likely to involve obstructions: the interstate, for example, is decribed as "broad and swift" (line 53).

Explanation for Incorrect Answer D :  Choice (D) is incorrect. The view from the river and interstate is not "unobstructed," or free. On the contrary, the passage argues that the only way to freely and immediately experience the plains is to travel across the land itself, via county roads.

(A) are teeming with unexpected surprises

(B) are reflective of environmental changes

(C) are subjected to minor obstructions

(D) give the traveler an unobstructed look at nature

(E) give an erroneous impression of the countryside

13

In context, the word “tunnel” (line 57) most directly emphasizes the expressway’s

ANSWERS AND EXPLANATIONS Explanation for Correct Answer C :  Choice (C) is correct. The passage describes the expressway as "exist[ing] in its own world, an unwalled tunnel, and mov[ing] at its own pace." This suggests that the expressway is independent of its surroundings.

Explanation for Incorrect Answer A :  Choice (A) is incorrect. Nothing in the passage emphasizes the expressway as a means of escape from dullness; on the contrary, the view from the expressway is described as being one of "unrelieved monotony" (line 56).

(A) function as a means of escape from provincial dullness

(B) role as a connection between two points

(C) independence from its surroundings

(D) intrusion into the landscape

(E) destruction of wildlife

页码,12/16The Official SAT Online Course

2006-11-12file://E:\新建文件夹\b9.htm

UnRe

gistered

Page 119: SAT Online Course Test 2 Explanations

严禁用于商业用途!

Explanation for Incorrect Answer B :  Choice (B) is incorrect. The passage does not emphasize a tunnel as something that connects points, but as something that is isolated from the landscape in which it exists.

Explanation for Incorrect Answer D :  Choice (D) is incorrect. The opposite is actually true. The author describes the expressway as remaining distinct and separate from the countryside through which it passes.

Explanation for Incorrect Answer E :  Choice (E) is incorrect. The author’s description of the expressway as an "unwalled tunnel" does not suggest any connection between the expressway and the destruction of wildlife.

14 The description of the county road (lines 61-77) suggests that it is most like the Great Plains in the way that it

ANSWERS AND EXPLANATIONS Explanation for Correct Answer B :  Choice (B) is correct. Like the Great Plains, the county road allows the traveler to interact with different elements of the country. The county road "moves in and through the landscape" (lines 61-62). It makes sense to say that the experience of driving on this road "fosters," or encourages, a sense of the world around one.

Explanation for Incorrect Answer A :  Choice (A) is incorrect. While the author celebrates the Great Plains and the county road for the pleasures they provide to the traveler, the passage does not argue that either "encourages a feeling of satisfaction with one’s lot in life."

Explanation for Incorrect Answer C :  Choice (C) is incorrect. The passage does not mention stillness or silence. Rather, the county road encourages the traveler to immerse himself or herself in the local rhythms of life.

Explanation for Incorrect Answer D :  Choice (D) is incorrect. Neither the county road nor the Great Plains "evokes," or causes one to feel, a sense of power and mastery over the land. On the contrary, both demand that the traveler adjust to suit the landscape, changing his or her speed and route as necessary.

Explanation for Incorrect Answer E :  Choice (E) is incorrect. The author does not discuss "an intuitive awareness of the complexity of all life" in this passage.

(A) encourages a feeling of satisfaction with one’s lot in life

(B) fosters a sense of the world around one

(C) creates a desire to cultivate stillness and silence

(D) evokes a sense of power and mastery over the land

(E) imparts an intuitive awareness of the complexity of all life

15

Both the “stray cow” (line 72) and the “hay wagon” (line 75) are presented as

(A) features of a rural landscape that are depicted sentimentally in American art

(B) objects that present unexpected dangers to the unwary

(C) memories of the author’s childhood that are typical of the Great Plains

(D) obstacles that force people to pay attention to the beauty of their surroundings

(E) symbols of an agrarian ideal that attracted settlers to the west

页码,13/16The Official SAT Online Course

2006-11-12file://E:\新建文件夹\b9.htm

UnRe

gistered

Page 120: SAT Online Course Test 2 Explanations

严禁用于商业用途!

ANSWERS AND EXPLANATIONS Explanation for Correct Answer D :  Choice (D) is correct. The author presents both the cow and the wagon as examples of "obstacles," or things that prevent passage and force travelers to slow or stop. As the passages describes it, "A stray cow might cross in front of you and you will be obliged to stop to let it pass, and so you will chance to hear the song of the meadowlark" (lines 72-74). The key thing about these obstacles is that they give travelers the opportunity to notice the beauty that surrounds them.

Explanation for Incorrect Answer A :  Choice (A) is incorrect. The passage does not discuss the depiction of rural landscapes in American art.

Explanation for Incorrect Answer B :  Choice (B) is incorrect. The cow and wagon are not described as dangers. Rather, they are presented as obstacles encountered on county roads that force one to stop and enjoy the environment.

Explanation for Incorrect Answer C :  Choice (C) is incorrect. The passage makes no mention of the author’s childhood or memories.

Explanation for Incorrect Answer E :  Choice (E) is incorrect. While the cow and wagon may be symbols of the countryside, the passage is not concerned with an "agrarian ideal," or dream of a life of farming, that may have attracted settlers.

16The opinion expressed in lines 78-81 (“The best . . . concrete”) assumes that the westward traveler

ANSWERS AND EXPLANATIONS Explanation for Correct Answer E :  Choice (E) is correct. The passages states that "the best way to go west, therefore, is the slowest way possible and across country rather than along the rivers" (lines 78-79). This advice assumes that a traveler actually wants to spend time in the countryside.

Explanation for Incorrect Answer A :  Choice (A) is incorrect. The author’s opinion that it is best to travel slowly across the Great Plains assumes that one is unfamiliar with the area and interested in learning about it.

Explanation for Incorrect Answer B :  Choice (B) is incorrect. Nothing in the passage suggests that enjoyment of the countryside is necessarily related to a dislike of cities.

Explanation for Incorrect Answer C :  Choice (C) is incorrect. The passage actually urges the traveler to avoid "both the old watery rivers and the new ones of asphalt and cast concrete" (lines 80-81).

Explanation for Incorrect Answer D :  Choice (D) is incorrect. The author encourages westward travelers to go slowly through the prairies so that they may enjoy and appreciate the countryside. Nothing in the passage mentions the study of different species of life, however.

(A) is familiar with the Great Plains

(B) dislikes cities

(C) appreciates the slow pace of river travel

(D) is interested in studying different species in nature

(E) wants to savor the countryside

页码,14/16The Official SAT Online Course

2006-11-12file://E:\新建文件夹\b9.htm

UnRe

gistered

Page 121: SAT Online Course Test 2 Explanations

严禁用于商业用途!

17 How is the way referred to in line 78 “best”?

ANSWERS AND EXPLANATIONS Explanation for Correct Answer B :  Choice (B) is correct. In the final sentence of the passage, the author states, "the best way to go west, therefore, is the slowest way possible." The author suggests that the slowest way across the Great Plains is the best way because it provides the greatest understanding of the land.

Explanation for Incorrect Answer A :  Choice (A) is incorrect. The passage does not describe county roads as "safe and easy," but rather as slow trails that require the traveler to adjust his or her schedule to the shape and rhythm of the landscape.

Explanation for Incorrect Answer C :  Choice (C) is incorrect. The opposite is actually true: far from being convenient, the land route is described as slow and roundabout, requiring that the traveler adapt to the plains and to obstacles on the road.

Explanation for Incorrect Answer D :  Choice (D) is incorrect. Given that the earliest explorers traveled west by river, it is likely that the water routes, rather than the overland routes, have the most historical significance. Further, there is no discussion of the historical significance of the Great Plains.

Explanation for Incorrect Answer E :  Choice (E) is incorrect. Nothing in the passage indicates that traveling overland is best because it illustrates the author’s "fund," or store, of knowledge. On the contrary, the prairie adds to and expands the author's fund of knowledge.

(A) It is safe and easy.

(B) It provides the most insight.

(C) It is the most convenient.

(D) It has the most historical significance.

(E) It illustrates the author’s fund of knowledge.

18Which of the following best expresses the author’s central point about river travel?

ANSWERS AND EXPLANATIONS Explanation for Correct Answer D :  Choice (D) is correct. According to the passage, river travelers did not see much of the Great Plains. Cut off from the land, the world they experienced looked "not at all like the Great Plains" (line 9). The author emphasizes that river travel was "not the most accurate way to see the country" (lines 5-6) because river travelers saw little of the Great Plains' celebrated landscapes.

Explanation for Incorrect Answer A :  Choice (A) is incorrect. The passage does not discuss whether or not river travel

(A) It was often hazardous.

(B) It caused disputes between travelers and the residents of the Northwest Territory.

(C) It was an inefficient way to travel.

(D) It offered little opportunity to learn about the landscape of the Great Plains.

(E) It offered the safest route to the West Coast.

页码,15/16The Official SAT Online Course

2006-11-12file://E:\新建文件夹\b9.htm

UnRe

gistered

Page 122: SAT Online Course Test 2 Explanations

严禁用于商业用途!

was dangerous.

Explanation for Incorrect Answer B :  Choice (B) is incorrect. The passage does not include any information on disputes, or fights, between travelers on the river and residents of the Northwest Territory.

Explanation for Incorrect Answer C :  Choice (C) is incorrect. The author does not desribe river travel as "inefficient," or something that wastes time. On the contrary, the passage states that traveling by the river was "the easiest but not the most accurate way to see the country" (lines 5-6).

Explanation for Incorrect Answer E :  Choice (E) is incorrect. The passage does not address which westward route offered the safest passage to the coast. Rather, it contrasts what one learns about the prairie from river travel with what one learns about the prairie by traveling overland.

19 The passage as a whole most fully answers which question?

ANSWERS AND EXPLANATIONS Explanation for Correct Answer D :  Choice (D) is correct. In the passage, the author compares the information travelers gather about the country on different western routes, ultimately concluding that only by going on slow county roads can one really learn about the Great Plains. As the last sentence of the passage points out, "the best way to go west, therefore, is the slowest way possible and across country rather than along the rivers" (lines 78-79).

Explanation for Incorrect Answer A :  Choice (A) is incorrect. The passage mentions several river routes but does not single any out as being the best route to the mountains.

Explanation for Incorrect Answer B :  Choice (B) is incorrect. Although the author encourages travelers to take the time to learn about and appreciate the prairie, the passage does not discuss which Great Plains ecosystems visitors find most exciting.

Explanation for Incorrect Answer C :  Choice (C) is incorrect. Although the passage mentions the experience of earlier explorers on the western rivers, it does not make broad claims about what these rivers can teach us about the past.

Explanation for Incorrect Answer E :  Choice (E) is incorrect. While the passage includes descriptions of the countryside, it does so to compare different ways of traveling across the prairie, not to "identify the best scenery." The author states that one gets an inaccurate impression of the prairie by traveling along a river, but the passage does not claim that river scenery is better or worse than other scenery.

(A) Which western river provides the best access to the mountains?

(B) Which ecosystems of the Great Plains are most exciting for visitors?

(C) What do the rivers of the West teach us about our past?

(D) What route west provides most information about the Great Plains?

(E) How can one identify the best scenery in the Great Plains?

    

Back to Score Report  

Copyright © 2006 The College Board. All rights reserved. Privacy Policy Terms of Use Contact Us

 

页码,16/16The Official SAT Online Course

2006-11-12file://E:\新建文件夹\b9.htm

UnRe

gistered

Page 123: SAT Online Course Test 2 Explanations

严禁用于商业用途!

Help | Profile | My Organizer | My Bookmarks | Logout

Answers and Explanations

Test Sections

Section 1

Section 2

Section 3

Section 4

Section 6

Section 7

Section 8

Section 9

Section 10

Back to Score Report  

View Answers and Explanations     Online - Practice Test #2

1 The amount of garbage produced in the United States could be reduced by recycling trash, minimizing packaging, and developing new technology for incinerators and landfills.

ANSWERS AND EXPLANATIONS Explanation for Correct Answer A :  Choice (A) is correct. It uses three parallel verbal phrases ("recycling trash," "minimizing packaging," and "developing new technology") to identify three actions that might reduce the amount of garbage.

Explanation for Incorrect Answer B :  Choice (B) fails to maintain parallelism. The clause "they develop" (used to state the third action in a series) is not parallel with the verbals "recycling" and "minimizing" (used to name the two earlier actions).

Explanation for Incorrect Answer C :  Choice (C) displays wordiness. Since the conjunction "and" already implies that additional information will follow, the word "also" is not needed; since the preposition "by" is not repeated before the second verbal in the series ("minimizing"), it is not needed before the third ("developing").

Explanation for Incorrect Answer D :  Choice (D) has a flaw in parallelism. Although two verbals in the series ("recycling" and "minimizing") are in active voice, the third verbal ("being developed") shifts awkwardly to passive voice.

Explanation for Incorrect Answer E :  Choice (E) violates parallelism. The clause "there was" is not parallel with the earlier verbals "recycling" and "minimizing."

(A) and developing new technology

(B) and if they develop new technology

(C) and also by developing new technology

(D) and new technology being developed

(E) and if there was new technology

2Tickets are available at the box office they can be picked up one hour before the performance.

ANSWERS AND EXPLANATIONS

(A) at the box office they can be picked up one hour before the performance

(B) at the box office; they can be picked up one hour before the performance

(C) one hour before the performance, they can be picked up at the box office

(D) and that can be picked up at the box office one hour before the performance

(E) at the box office, one hour before the performance is when they can be picked up

页码,1/9The Official SAT Online Course

2006-11-12file://E:\新建文件夹\b10.htm

UnRe

gistered

Page 124: SAT Online Course Test 2 Explanations

严禁用于商业用途!

Explanation for Correct Answer B :  Choice (B) is correct. It avoids the error of the original by using a semicolon to join two complete thoughts.

Explanation for Incorrect Answer A :  Choice (A) displays improper coordination. Two complete thoughts ("Tickets . . . at the box office" and "they can . . . before the performance") are fused together without any punctuation to indicate their boundaries.

Explanation for Incorrect Answer C :  Choice (C) uses improper coordination. Two complete thoughts ("Tickets . . . before the performance" and "they can . . . at the box office") are linked with only a comma.

Explanation for Incorrect Answer D :  Choice (D) uses an inappropriate pronoun. The singular pronoun "that" does not agree with the plural noun "tickets."

Explanation for Incorrect Answer E :  Choice (E) exhibits improper coordination. It uses only a comma to link two complete thoughts ("Tickets . . . at the box office" and "one hour before the performance . . . picked up").

3 Medical research now emphasizes that the best treatment for many diseases is the body’s own defenses being stimulated.

ANSWERS AND EXPLANATIONS Explanation for Correct Answer B :  Choice (B) is correct. It avoids the error of the original by linking the subject of the clause,"treatment," with the predicate noun, "stimulation."

Explanation for Incorrect Answer A :  Choice (A) involves logical disagreement. The resulting sentence links two nouns ("treatment" and "defenses") of different logical types, illogically suggesting that the body’s defenses are a medical treatment rather than the object of such a treatment.

Explanation for Incorrect Answer C :  Choice (C) results in wordiness. The phrase "there should be" is unnecessary.

Explanation for Incorrect Answer D :  Choice (D) results in wordiness and creates an improper idiom. The phrase "there should be" is unnecessary. The phrase "a stimulation" improperly suggests a discrete event rather than an ongoing effort.

Explanation for Incorrect Answer E :  Choice (E) involves an improper idiom. The phrase "should have stimulation" improperly suggests that stimulation is a form of treatment for the defenses rather than for the body.

(A) the best treatment for many diseases is the body’s own defenses being stimulated

(B) the best treatment for many diseases is stimulation of the body’s own defenses

(C) there should be stimulation of the body’s own defenses for the best treatment for many diseases

(D) there should be a stimulation of the body’s own defenses to achieve the best treatment for many diseases

(E) the body’s own defenses should have stimulation for the best treatment for many diseases

4 Some political scientists are convinced that the major conflict toward democracy and

页码,2/9The Official SAT Online Course

2006-11-12file://E:\新建文件夹\b10.htm

UnRe

gistered

Page 125: SAT Online Course Test 2 Explanations

严禁用于商业用途!

communism is moral.

ANSWERS AND EXPLANATIONS Explanation for Correct Answer E :  Choice (E) is correct. It avoids the error of the original by replacing the preposition “toward” with the idiomatic preposition “between.”

Explanation for Incorrect Answer A :  Choice (A) results in an improper idiom. The noun “conflict” is followed by the preposition “toward,” rather than by the preposition “between.”

Explanation for Incorrect Answer B :  Choice (B) creates an illogical sentence. The sentence identifies “the major conflict” as “a moral one” rather than as one between “democracy and communism.”

Explanation for Incorrect Answer C :  Choice (C) involves an improper idiom. The phrase “between democracy” is followed by the phrase “compared to communism” rather than by “and communism.”

Explanation for Incorrect Answer D :  Choice (D) results in an illogical sentence. The conflict that political scientists have identified exists between democracy and communism, not “for democracy, compared to communism.”

(A) toward democracy and communism is moral

(B) is a moral one between democracy and communism

(C) between democracy, compared to communism, is a moral one

(D) is a moral one for democracy, compared to communism

(E) between democracy and communism is a moral one

5Until just recently many students would take lengthy trips during spring break rather than temporary jobs like now.

ANSWERS AND EXPLANATIONS Explanation for Correct Answer C :  Choice (C) is correct. It avoids the error of the original by providing a clause, "now they take temporary jobs," whose structure parallels that of the first clause. It thus clarifies the comparison between what students used to do and what they do now.

Explanation for Incorrect Answer A :  Choice (A) involves an illogical comparison. The word "like" indicates that "temporary jobs" are illogically being compared to "now."

Explanation for Incorrect Answer B :  Choice (B) results in an illogical comparison. By the use of the word "like," it illogically compares a noun, "jobs," to an adverb, "now."

Explanation for Incorrect Answer D :  Choice (D) is unsatisfactory because of vague wording. It provides the adverb "instead" to introduce the second clause ("students are . . . jobs") instead of the

(A) break rather than temporary jobs like now

(B) break instead of temporary jobs like now

(C) break; now they take temporary jobs

(D) break; instead, students are taking temporary jobs

(E) break, but now it is temporary jobs

页码,3/9The Official SAT Online Course

2006-11-12file://E:\新建文件夹\b10.htm

UnRe

gistered

Page 126: SAT Online Course Test 2 Explanations

严禁用于商业用途!

adverb "now" that is needed, thus obscuring the comparison between what students used to do and what they do now.

Explanation for Incorrect Answer E :  Choice (E) involves the use of a vague pronoun. There is nothing in the sentence to which the pronoun "it" can logically refer.

6 To meet the college’s requirement of service to her community, 40 hours of her time to Habitat for Humanity were volunteered by Laurie.

ANSWERS AND EXPLANATIONS Explanation for Correct Answer B :  Choice (B) is correct. It avoids the error of the original by placing the introductory phrase "To meet . . . community" directly next to "Laurie," the noun it modifies.

Explanation for Incorrect Answer A :  Choice (A) involves improper modification. The introductory phrase, "To meet . . . community," improperly modifies "40 hours" rather than "Laurie."

Explanation for Incorrect Answer C :  Choice (C) creates improper modification. The opening phrase, "To meet . . . community," improperly modifies "40 hours" rather than "Laurie."

Explanation for Incorrect Answer D :  Choice (D) results in an incomplete thought. The introductory phrase "To meet . . . community" is followed by another phrase, "through . . . Humanity," rather than by an independent clause.

Explanation for Incorrect Answer E :  Choice (E) creates improper modification. The adverb phrase "to Habitat for Humanity" modifies "Laurie" rather than the verb "was volunteering."

(A) 40 hours of her time to Habitat for Humanity were volunteered by Laurie

(B) Laurie volunteered 40 hours of her time to Habitat for Humanity

(C) 40 hours of her time to Habitat for Humanity was what Laurie volunteered

(D) through 40 hours of volunteering by Laurie to Habitat for Humanity

(E) Laurie, to Habitat for Humanity, was volunteering 40 hours of her time

7 Reviews of books and films have generally been exempt from the standards of libel that are applied to news stories, but they are now questioning the distinction between reporting and criticism.

ANSWERS AND EXPLANATIONS Explanation for Correct Answer C :  Choice (C) is correct. It avoids the error of the original by providing a passive verb phrase ("is . . . being questioned") and by removing the vague pronoun "they."

Explanation for Incorrect Answer A : 

(A) they are now questioning the distinction between reporting and criticism

(B) questions are now being asked by them as to the distinction between reporting and criticism

(C) the distinction between reporting and criticism is now being questioned

(D) the question they are asking now is whether there is a distinction between reporting and criticism

(E) the distinction they make between reporting and criticism is now questioned

页码,4/9The Official SAT Online Course

2006-11-12file://E:\新建文件夹\b10.htm

UnRe

gistered

Page 127: SAT Online Course Test 2 Explanations

严禁用于商业用途!

Choice (A) involves the use of a vague pronoun. There is nothing in the sentence to which the pronoun "they" can logically refer.

Explanation for Incorrect Answer B :  Choice (B) includes the use of a vague pronoun. The pronoun "they" cannot logically refer to any of the plural nouns in the sentence.

Explanation for Incorrect Answer D :  Choice (D) results in an awkward sentence, combining unnecessary emphasis ("the question they are asking now") with the use of a vague pronoun, "they," which cannot logically refer to anything in the sentence.

Explanation for Incorrect Answer E :  Choice (E) creates a confusing sentence. The pronoun "they" apparently refers to "reviews," but it is odd to say that reviews, which are criticism, could make a distinction between reporting and criticism.

8 A writer who well understood the plight of the underprivileged, many acclaim Richard Wright as the novelist of the downtrodden.

ANSWERS AND EXPLANATIONS Explanation for Correct Answer D :  Choice (D) is correct. It avoids the error of the original by following the introductory adjective clause (“A writer who . . . underprivileged” ) with “Richard Wright,” the noun it modifies.

Explanation for Incorrect Answer A :  Choice (A) results in improper modification. The introductory noun clause “A writer . . . underprivileged” improperly modifies “many” when it should modify “Richard Wright.”

Explanation for Incorrect Answer B :  Choice (B) creates improper modification. The noun clause “A writer . . . underprivileged” improperly modifies the pronoun “many” rather than the proper noun “Richard Wright.”

Explanation for Incorrect Answer C :  Choice (C) results in an improper sequence of verb tenses. The verb “understood” (past tense) is improperly followed by the verb “is being acclaimed” (present progressive tense).

Explanation for Incorrect Answer E :  Choice (E) involves improper modification. The introductory adjective clause (“A writer . . . underprivileged”) is improperly followed by an adjective phrase, “widely acclaimed,” rather than by “Richard Wright,” the noun it modifies.

(A) many acclaim Richard Wright as

(B) many have acclaimed Richard Wright as

(C) Richard Wright is being acclaimed by many as

(D) Richard Wright has been widely acclaimed as

(E) widely acclaimed is Richard Wright as

9Having no additional funds to spend, the meeting of the budget committee was promptly concluded.

(A) the meeting of the budget committee was promptly concluded

(B) the meeting of the budget committee concluded promptly

(C) the conclusion of the budget committee meeting was prompt

(D) the budget committee promptly concluded its meeting

(E) the budget committee’s meeting was promptly concluded

页码,5/9The Official SAT Online Course

2006-11-12file://E:\新建文件夹\b10.htm

UnRe

gistered

Page 128: SAT Online Course Test 2 Explanations

严禁用于商业用途!

ANSWERS AND EXPLANATIONS Explanation for Correct Answer D :  Choice (D) is correct. It avoids the error of the original by providing an appropriate subject, "the budget committee," for the modifying phrase "Having . . . spend."

Explanation for Incorrect Answer A :  Choice (A) involves improper modification. It improperly makes "the meeting" the subject of the modifying phrase "Having . . . spend."

Explanation for Incorrect Answer B :  Choice (B) results in improper modification. The phrase "Having no additional funds to spend" cannot logically modify "the meeting of the budget committee."

Explanation for Incorrect Answer C :  Choice (C) is unsatisfactory because it involves improper modification. The phrase "Having . . . spend" cannot logically modify "the conclusion of the budget committee meeting."

Explanation for Incorrect Answer E :  Choice (E) results in an illogical sentence because it involves improper modification. It illogically makes "the budget committee's meeting" the subject of the modifying phrase "Having . . . spend."

10 In the early songs of the Beatles, one hears plaintive Blues-inspired melodies that would seem to be more a product of rural southern America than an English industrial city.

ANSWERS AND EXPLANATIONS Explanation for Correct Answer E :  Choice (E) is correct. It avoids the error of the original by providing a phrase, "of an English industrial city," that is grammatically parallel to the phrase "of rural southern America," thus making clear that two locations are being compared.

Explanation for Incorrect Answer A :  Choice (A) involves an illogical comparison. A product is being illogically compared to a city.

Explanation for Incorrect Answer B :  Choice (B) is wordy; the pronoun "that" is not necessary. Also, there is an improper idiom: the phrase "a product from . . . America" is used where it would be more idiomatic to say "a product of . . . America."

Explanation for Incorrect Answer C :  Choice (C) results in the use of an improper idiom. The phrase "more produced in . . . America" is used where it would be more idiomatic to say "more a product of . . . America."

Explanation for Incorrect Answer D :  Choice (D) involves the use of an improper idiom. The phrase "a product out of . . . America" is used where it would be more idiomatic to say "a product of . . . America."

(A) a product of rural southern America than an English industrial city

(B) a product from rural southern America than that of an English industrial city

(C) produced in rural southern America than by an English industrial city

(D) a product out of rural southern America than from an English industrial city

(E) a product of rural southern America than of an English industrial city

页码,6/9The Official SAT Online Course

2006-11-12file://E:\新建文件夹\b10.htm

UnRe

gistered

Page 129: SAT Online Course Test 2 Explanations

严禁用于商业用途!

11 In 1968 air pollution from automobile exhaust, particularly like that in the Los Angeles environs, became of an increasingly more urgent concern to environmentalists.

ANSWERS AND EXPLANATIONS Explanation for Correct Answer B :  Choice (B) is correct. It avoids the error of the original by changing the overly fancy word "environs" to the simpler "area" and by changing the phrase "became of an increasingly more urgent concern" to the more idiomatic "became an increasingly urgent concern."

Explanation for Incorrect Answer A :  Choice (A) results in an awkward and wordy sentence. The word "environs" is unnecessarily fancy; also, the phrase "became of an increasingly more urgent concern" is used where it would be more idiomatic to say "became an increasingly urgent concern."

Explanation for Incorrect Answer C :  Choice (C) involves an illogical sequence of tenses. The present tense of the verb in the phrase "such as you have" does not make sense in a sentence describing something that happened in 1968.

Explanation for Incorrect Answer D :  Choice (D) involves wordiness. The word "environs" means the area around something, so it is repetitive to use both it and the phrase "in and around."

Explanation for Incorrect Answer E :  Choice (E) involves an illogical comparison. Air pollution is illogically being compared to a geographical area.

(A) like that in the Los Angeles environs, became of an increasingly more urgent

(B) of the sort found in the Los Angeles area, became an increasingly urgent

(C) such as you have in the area of Los Angeles, became of increasingly more urgent

(D) like the kind in and around Los Angeles and its environs, became an increasingly urgent

(E) the kind similar to the Los Angeles area, became an increasingly more urgent

12Although long considered a vestigial organ that has no function in humans, the appendix, some scientists believe, may have a significant role as part of the body’s immune system.

ANSWERS AND EXPLANATIONS Explanation for Correct Answer A :  Choice (A) is correct. It provides an appropriate subject, "the appendix," for the modifying phrase, "Although long considered . . . in humans."

Explanation for Incorrect Answer B :  Choice (B) results in a sentence fragment. The sentence is grammatically incomplete because it has no main verb.

(A) the appendix, some scientists believe, may have

(B) the appendix, which, some scientists believe, may have

(C) the appendix, in the belief of some scientists, is possibly having

(D) the appendix is believed by some scientists as perhaps having

(E) some scientists believe the appendix may have

页码,7/9The Official SAT Online Course

2006-11-12file://E:\新建文件夹\b10.htm

UnRe

gistered

Page 130: SAT Online Course Test 2 Explanations

严禁用于商业用途!

Explanation for Incorrect Answer C :  Choice (C) is unsatisfactory because it involves the use of improper idioms. The phrase "in the belief of" is used where it would be more idiomatic to say "in the view of," and the verb phrase "is possibly having" is used where it would be more idiomatic to say "may have."

Explanation for Incorrect Answer D :  Choice (D) involves the use of an improper idiom. It provides the phrase "is believed . . . as perhaps having" where it would be more idiomatic to use a phrase such as "is believed . . . to have."

Explanation for Incorrect Answer E :  Choice (E) involves improper modification. It improperly makes "some scientists" the subject of the modifying phrase "Although long considered . . . in humans."

13 Failing to anticipate the sharp downturn in the economy, millions of dollars were lost through the investment fund’s managers’ slowness to act.

ANSWERS AND EXPLANATIONS Explanation for Correct Answer D :  Choice (D) is correct. It avoids the error of the original by providing an appropriate subject, "managers," for the modifying phrase "Failing to . . . the economy."

Explanation for Incorrect Answer A :  Choice (A) results in improper modification. It improperly makes the noun phrase "millions of dollars" the subject of the modifying phrase "Failing to anticipate . . . the economy."

Explanation for Incorrect Answer B :  Choice (B) involves improper modification. The noun phrase "millions of dollars" cannot logically be modified by the phrase "Failing to anticipate . . . the economy."

Explanation for Incorrect Answer C :  Choice (C) results in improper coordination. The clause beginning with the word "millions" is linked to the previous clause by only a comma.

Explanation for Incorrect Answer E :  Choice (E) results in an illogical sentence because it involves improper modification. It illogically makes the noun phrase "the investment fund" the subject of the modifying phrase "Failing to anticipate . . . the economy."

(A) millions of dollars were lost through the investment fund’s managers’ slowness to act

(B) millions of dollars were lost due to the slowness of the investment fund’s managers to act

(C) the investment fund’s managers were slow to act, millions of dollars were lost as a result

(D) the investment fund’s managers were slow to act and thus lost millions of dollars

(E) the investment fund lost millions of dollars because its managers acted too slowly

14 Most often defenders of art have justified its existence with their pointing out a function that nothing but art itself could perform.

ANSWERS AND EXPLANATIONS

(A) with their pointing out a function that nothing but art itself could perform

(B) by them pointing out a function that only art could perform

(C) through the pointing out of a function performed by art alone

(D) by pointing to some function that art alone can perform

(E) through their having pointed to a function performed by art itself

页码,8/9The Official SAT Online Course

2006-11-12file://E:\新建文件夹\b10.htm

UnRe

gistered

Page 131: SAT Online Course Test 2 Explanations

严禁用于商业用途!

Explanation for Correct Answer D :  Choice (D) is correct. It avoids the error of the original by using the proper preposition, "by," and by eliminating the unnecessary pronoun "their."

Explanation for Incorrect Answer A :  Choice (A) results in an improper idiom. The phrase "with their pointing out" uses the preposition "with" and the pronoun "their" where the preposition "by" would be idiomatic.

Explanation for Incorrect Answer B :  Choice (B) involves an improper idiom. The phrase "by them pointing" uses the pronoun "them" where no pronoun is required.

Explanation for Incorrect Answer C :  Choice (C) involves an improper idiom. The phrase "through the pointing out" uses the preposition "through" where "by" would be idiomatic.

Explanation for Incorrect Answer E :  Choice (E) creates an improper idiom. The sentence uses the preposition "through" and the phrase "their having pointed" where the preposition "by" and the noun "pointing" would be idiomatic.

  

Back to Score Report  

Copyright © 2006 The College Board. All rights reserved. Privacy Policy Terms of Use Contact Us

 

页码,9/9The Official SAT Online Course

2006-11-12file://E:\新建文件夹\b10.htm

UnRe

gistered